You are on page 1of 177

HOSPITAL AND CLINICAL PHARMACY

9. The following are factors that may exacerbate


1. Which of the following is NOT included in the established chronic renal failure, EXCEPT:
treatment algorithm for allergic rhinitis?
A. Hypotension D. Nephrotoxins
A. Implement appropriate environmental controls. B. Prostatic hypertrophy E. Hypertension
B. Consider Cromolyn if with sneezing, itching and C. Increased renal blood flow
rhinorrhea.
C. Administer intranasal steroids if with sneezing, 10. Another term for nearsightedness:
itching, rhinorrhea, and nasal congestion.
D. Give intranasal anticholinergics if with sneezing and A. astigmatism D. Presbyopia
ocular symptoms. B. hyperopia E. Myopia
E.I f symptoms still not controlled, consider Montelukast. C. exophthalmos
2. Clinical presentation of Acute Renal Failure includes: 11. Which of the following may cause “hairy tongue”?
A. Edema D. A, B, C A. Hypercholesterolemia D. Liver disease
B. Orthostatic Hypotension E. A and B B. Antibiotic therapy E. Aging
C. Foamy urine C. Measles
3. Nail and nail beds are evaluated for which of the 12. Which strategy may be advised to a patient who
following? considers Beta blockers unpleasant to take?
A. Clubbing D. All of these A. Change the dosage regimen. D. A and B
B. Cyanosis E. None of these B. Consider verapamil. E. All of these
C. Trauma C. Encourage patient to use regularly.
4. Why is the Rinne Test performed? 13. Which of the following is NOT TRUE regarding
Captopril?
A. To assess equilibrium.
B. To hearing acuity of high-frequency sounds. A. Taken once daily.
C. To evaluate hearing in a person who hears better in B. First-dose hypotension may occur
one ear than in the other. C. May worsen renal failure
D. To compare air and bone conduction. D. Cough and taste disturbance may occur
E. All of these E. Hyperkalemia may be expected
5. Blistering of the skin due to second-degree burns is 14. The following drugs may increase anticoagulant
an example of a: effect, EXCEPT:
A. Excoriation D. Nevus A. Ketoconazole D. Erythromycin
B. Lichenification E. Bulla B. Estrogen E. Zafirlukast
C. Millium C. Gemfibrozil
6. Method of assessing the sinuses: 15. Drug-induced bronchospasm due to reflex
bronchoconstriction:
A. Inspection D. Expression
B. Percussion E. Auscultation A. Pilocarpine D. Penicillin
C. Transillumination B. Aspirin E. Timolol
C. Lactose dry powder
7. Which body position allows longer retention times for
enemas? 16. Physical assessment technique of creating a note by
tapping a finger placed on the body:
A. Lithotomy D. Erect
B. Supine E. Prone A. Palpation D. Direct percussion
C. Sitting B. Auscultation E. Indirect percussion
C. Ballotement
8. Grade for hepatic encephalopathy with experience of
drowsiness and grossly impaired calculation ability. 17. Koilonychia is due to deficiency of ___________?
A. Grade 0 D. Grade 3 A. Iron D. Water
B. Grade 1 E. Grade 4 B. Vitamin B12 E. Vitamin C
C. Grade 2 C. Calcium
18. Example of a Scale skin lesion: A. Research D. Patient care
B. Wellness E. Expansion
A, Hives D. Mouth sore C. Education
B. Dandruff E. Wart
C. Measles rash 27. Type of questioning: “What medication do you take
for a headache?”
19. Which benzodiazepine is LEAST to be used among
elderly patients? A. Direct Questioning D. Open-ended
B. Indirect Questioning E. Close-ended
A. Diazepam D. Lorazepam C. Probing
B. Oxazepam E. Clonazepam
C. Alprazolam 28. “Spell WORLD backwards” is an example of a test
for what area of mental state?
20. First-line treatment for Myoclonic seizures:
A. Orientation D. Registration
A. Phenobarbital D. Valproate B. Attention E. Language
B. Acetazolamide E. Topiramate C. Recall
C. Phenytoin
29. Which information is under the Demographic
21. A patient is diagnosed with parkinsonian syndrome if database?
patient has bradykinesia, plus one or more of the
following: A. Patient Identification Number
B. Physician and prescribers
A. Classic rest tremor D. B and C C. Occupation
B. Muscular rigidity E. A, B, and C D. Personality type
C. Postural instability E. Ethnic background

22. These are orders for in-patients in hospitals and 30. Volunteer service belongs to which department in the
other institutions written by physicians on forms: hospital?

A. Physician’s Order Sheet D. Recipe A. Pharmacy service


B. Prescription E. Patient’s Chart B. Medical Library
C. Medication Order C. Business office
D. Medical Social Service
23. The following are types of information under social/ E. Administrative
economic database, EXCEPT:
31. When using the standardized outline approach,
A. Daily activities D. Sexual History which of the following is NOT followed when writing
B. Ethnic background E. A and D Progress Notes?
C. Living arrangement
A. Subjective data
24. “Health is a state of complete physical, mental, and B. Objective data
social well-being and not merely the absence of disease C. Recommendation
or infirmity.” This condition is according to which model? D. Impression
E. None of these
A. Environmental model D. Medical model
B. Wellness model E. All of these 32. The question “Are agents available in commercial
C. World Health Organization model formulation?” represents which request classification for
drug information?
25. Patients of psychiatric conditions usually stay in
which type of hospital? A. Adverse Drug Reaction
B. Availability
A. Non-governmental C. Compounding/ Formulation
B. General D. Identification
C. Long-term E. Compatibility/ Stability
D. 400-499 bed-capacity
E. Short-term 33. When looking for information regarding method of
administration, which of the following drug information
26. Which function of the hospital is geared towards the sources may be the most useful?
improvement of hospital services and advancement of
medical knowledge against disease? A. Drug Facts and Comparisons
B. RPS
C. Goodman and Gilman’s Pharmacological Basis of 40. The USP contains nomograms for estimating body
Therapeutics surface area (BSA) for both children and adults. Which
D. Martindale: The Complete Drug Reference of the following measurements must be known in order
E. USP DI to use the nomogram?

34. If the answer to the question “Do you cough up A. Age and height D. Height and weight
blood?” is negative, which of the following is to be noted B. Age and weight E. Weight and sex
that the patient denied of? C. Height and creatinine clearance

A. Paroxysmal nocturnal dyspnea 41. Which of the following is a comprehensive, unbiased


B. Hemoptysis source of current information on drugs often used as
C. Shortness of breath hospital nursing stations, and serves as a basis for
D. Exertional SOB pharmacists to extend their role as pharmaceutical
E. Rales consultants to the medical profession?

35. This is an important tool for assuring the quality of A. Minimum Standard for Pharmacies in Hospitals
drug use and controlling its cost in an institution such as B. Mirror to Hospital Pharmacy
the hospital. C. International Pharmaceutical Abstracts
D. American Journal of Health-System Pharmacy
A. Essential drug list E. Drug Information
B. Formulary system
C. Formulary 42. Which class of stock system consists of a
D. P&T Committee predetermined list of medications that are usually used
E. Core list in large quantities and are prepackaged in standardized
containers?
36. A facility where ambulatory patients are seen for
appointments and treated by a group of physicians A. Unit-dose system
practicing together. B. Charge floor stock
C. Free floor stock
A. Long-term health care facility D. Inpatient prescription system
B. Clinic E. Coordinated control system
C. Hospital
D. County hospital 43. Aside from Policy Development, what is the other
E. Resident treatment facility primary purpose of the P & T committee?

37. Which of the following belongs to the business A. Operation D. Education


management or administrative side of the hospital? B. Organization E. Drug Administration
C. Community Health
A. Medical Records
B. Medical Library 44. Water for injection is an example of drugs dispensed
C. Biomedical Engineering in which type of stock system?
D. Pharmacy Service
E. Dietary and Nutrition Service A. Unit dose system
B. Charge floor stock
38. Which of the following statements regarding C. Free floor stock
questioning strategies for the pharmacist-patient D. Inpatient prescription system
encounter is CORRECT? E. Coordinated control system

A. Probing questions are assertive and aggressive. 45. Which of the following drug information may not be
B. Precise information may be obtained using the found in Physician’s Desk Reference?
close-ended questions.
C. Open-ended questions are for narrow-focused A. Lactation D. Dosage
answers. B. Drug of choice E. Product availability
D. Direct questions encourages patient to elaborate. C. ADR
E. All except A
46. Which of the following journals cover medical news
39. Which of the following body areas is last in the and original articles?
sequence followed for physical examination?
A. Drug Information Journal
A. HEENT D. Neurological System B. Archives of Internal Medicine
B. Genitals E. Lower extremities C. New England Journal of Medicine
C. Rectum D. Medical Letter
E. Annals of Pharmacotherapy A. Subjective D. Plan
B. Objective E. Either A or B
47. Which of the following describes a medium intensity C. Assessment
of feeling, which can be used in a medical interview?
55. Which one of the following is useful in the diagnosis
A. Marvelous D. Blue and monitoring of liver disease?
B. Upset E. Gloomy
C. Distraught A. Amylase
B. Hematocrit
48. The following statements regarding arterial blood C. Hemoglobin
gases are true, EXCEPT: D. Human Chorionic Gonadotropin
E. Bilirubin
A. Alveolar hyperventilation if PaCo 2 <4.5 kPa
B. Oxygen saturation is measured by pulse oximetry 56. What color of the stool is generally associated with
C. Alveolar ventilation may regulate PaCO 2 upper gastrointestinal tract bleeding?
D. Low PaCO2 implies alveolar hypoventilation.
E. Arterial oxygen tension < 12 kPa may indicate A. Black D. Brown
hypoxaemia B. Gray E. Red
C. Light Gray
49. Which term refers to a decreased number of cell
types is synonymous with aplastic anemia? 57. Which one of the following may indicate longer
prothrombin time?
A. Microcytic D. Thrombocytopenia
B. Pancytopenia E. Neutrophilia A. Hepatic dysfunction
C. Aplastic B. Increased Vitamin K
C. Increased production of clotting factors
50. Which of the following is an invasive test or D. All of these
procedure? E. None of these

A. Insertion of a central venous catheter 58. Which of the following is TRUE regarding patient
B. Taking of chest x-ray films chart preparation?
C. Collection of spontaneously voided urine
D. Comparative measurements of body parts A. Abbreviations of any kind should not be used.
E. All except B B. Write in ink, felt, or pencil.
C. Blue ink is conventional.
51. Which one of the following uses high frequency D. The patient should be identifiable by at least one (1)
sound waves imperceptible to the human ear to create demographic characteristic.
images of organs? E. All of the choices except B

A. Standard Radiography 59. Which one of the following may be an indication of


B. Echocardiography the presence of white blood cells in the stool?
C. Echography
D. Echocardiogram A. Bile duct obstruction
E. Radionuclide studies B. Gastrointestinal infection
C. Measles
52. Sexual history is under what patient information type/ D. Dehydration
section? E. Rapid gastrointestinal transit

A. Demographic D. Behavioral 60. Which dosage form is utilized in barium studies?


B. Medical E. Administrative
C. Social A. Troches D. Ointment
B. Enema E. Dressing
53. Assessment of pancreatic function may be C. Implant
determined by the following, EXCEPT:
61. Using the Scratch Test, which one of the following is
A. Glucose D. Insulin not a criterion for evaluation of the skin indicating the
B. Amylase E. None of these patient’s sensitivity to an allergen?
C. Thyroxine
A. Itchiness D. No change/ reaction
54. Mini-mental status exam (MMSE) is an example of B. Swelling E. B and C
what data type? C. Redness
62. A bronchoscope is introduced through the following
openings, EXCEPT: A. Edrophonium tensilon test D. Romberg Test
A. Nose D. Mouth B. Ventilation Scanning E. Stress Test
B. Endotracheal tube E. None of these C. Scratch and Patch Test
C. Rectum
71. Test of choice in patients suspected of having iron
63. Which one of the following is a noninvasive test or deficiency anemia:
procedure?
A. Coomb’s Test D. C-reactive protein
A. Venipuncture D. Ultrasonography B. Serum ferritin E. Haptoglobulin
B. Radionuclide studies E. pH stimulation tests C. ZPP fluorescence
C. Paracentesis
72. This refers to the technique of manipulating
64. Parameter of fecalysis that is used in the diagnosis unconscious or involuntary bodily processes made
of intestinal hemorrhage: perceptible to the senses by conscious mental control:

A. Consistency D. Amount A. Naturopathy D. Biofeedback


B. Fecal parasites E. pH B. Auscultation E. Holistic Health
C. Occult blood C. Acupuncture

65. Drug that may cause brown-colored urine: 73. BMI category indicating a Possible likelihood of
chronic protein-energy under nutrition:
A. Methyldopa D. Doxorubicin
B. Rifampicin E. Metronidazole A. <10 kg/ m2 D. 20-35 kg/ m2
C. Indomethacin B. <18 kg/ m2 E. >35 kg/ m2
C. 18.5-20 kg/ m2
66. Major component of plasma proteins and indicates
severity in chronic liver disease 74. In parenteral nutrition, which supplies Chloride the
most per mL?
A. Albumin D. Glucose
B. Creatinine E. Bilirubin A. Sodium chloride D. Iron chloride
C. Hemoglobin B. Potassium chloride E. None of these
C. Calcium chloride
67. Test to evaluate the absorption of Vitamin B12:
75. Which of the following statements is INCORRECT
A. Schilling Test when handling cytotoxic drugs:
B. Rinne Test
C. Weber’s Test A. Syringes and IV sets with luer-lock fittings are
D. Romberg Test recommended.
E. Schamroth Technique B. Syringes should be large enough to accommodate the
total dose needed.
68. Which of the following pairs of clinical findings and C. Syringes should not be three-quarters full when
laboratory values is TRUE? containing the total drug dose.
D. B and C
A. Proteus mirabilis infection – urine pH 9.0 E. None of these
B. Hypothyroidism – FTI4: 6.4
C. Warfarin therapy – Bleeding time: 2 min 76. Which among the following is NOT a characteristic of
D. Primary hyperparathyroidism – Serum chloride: 128 the therapeutic relationship between the patient and the
mEq/L pharmacist?
E. Previous myocardial infarction – CK- MB: 5.7 ng/mL
A. Equality D. Authenticity
69. Which of the following pairs of clinical findings and B. Trust E. Responsiveness
laboratory values is FALSE? C. Empathy

A. Hepatocellular damage – Female patient: ALT: 50 U/L 77. Example of metabolic complications during
B. Folic acid deficiency – Male patient: 76% parenteral nutrition, EXCEPT:
C. Lithium carbonate therapy – Serum Potassium: 2.1
mEq/L A. Thrombocytopenia
D. Hemolytic disease – Total Bilirubin: 0.8 mg/dL B. Lipid intolerance
E. Diabetes mellitus – GIT: 180 mg/dL at 2 hrs C. Thrombophlebitis
D. Essential fatty acid deficiency
70. Test used to diagnose myasthenia gravis: E. Refeeding syndrome
78. An important clinical skill that refers to the 87. Which of the following databases is published by the
continuous process of deliberation and reflection during American Society of Health-System Pharmacists
problem solving: (ASHP)?

A. Metacognition D. Genuineness A. MEDLARS


B. Ethnocentricity E. Manipulation B. International Pharmaceutical Abstracts
C. Adherence C. Index Medicus
D. Toxline
79. Which clinical finding is associated to Measles? E. EMBASE

A. Hairy tongue D. Koplik spots 88. What is deficiency in scurvy?


B. Chvostek’s sign E. Xanthelesma
C. Osler’s node A. Vitamin B1 D. Iron
B. Vitamin C E. Phosphorus
80. A small, circumscribed, elevated solid lesion: C. Calcium

A. Papule D. Macule 89. Blistering of the skin due to second-degree burns is


B. Vesicle E. Comedo an example of a:
C. Patch
A. Excoriation D. Vesicle
81. What body organ is associated with fetor hepaticus? B. Lichenification E. Bulla
C. Millium
A. Kidney D. Skin
B. Uterus E. Liver 90. Volume of air inspired or expired with normal
C. Oral mucosa breathing:

82. Practice sites for Ambulatory Care Setting: A. Tidal volume


B. Residual volume
A. Nursing homes D. A, B, C C. Peak expiratory flow
B. Patient’s residence E. B and C D. Forced tidal capacity
C. Clinic environment E. A and B

83. Conditions that can affect vision, EXCEPT: 91. Comparative measurements of parts of the body are
used to assess nutritional status:
A. Astigmatism D. Corneal arcus
B. Muddy sclera E. All except B A. Manometry D. EMG
C. Conical cornea B. Anthropometrics E. ESR
C. Schilling Test
84. Which of the following describes a macule?
92. Which of the following is NOT a primary literature for
A. Wart D. A and C drug information?
B. Colored lesion E. B and C
C. Less than 1cm A. Morbidity and Mortality Weekly Report
B. Annals of Internal Medicine
85. Which among the given characteristics is NOT true C. Clin-Alert
regarding a Social interaction as to space boundaries D. Lancet
with patient? E. Pharmacotherapy

A. Patient is 4-12 feet away. 93. Electrolyte level is increased due to spironolactone
B. Body contact is described as out of reach. use:
C. Eye contact is important.
D. Low and frequent vocalization A. Potassium D. Calcium
E. None of these B. Magnesium E. Chloride
C. Sodium
86. Herpes zoster is an example of ________
94. Drugs that may cause dyspepsia
A. Patch D. Papule
B. Scale E. Macule A. Corticosteroids D. Either A or B
C. Vesicle B. Iron E. A, B, C
C. Antibiotics
95. The following are precipitating factors of hepatic 103. Which of the following is a qualification for a Senior
encephalopathy, EXCEPT: Pharmacist in a hospital?

A. Constipation D. Bacterial peritonitis A. With at least 3 years work experience


B. High-protein diet E. Benzodiazepines B. With at least 1 year pharmacy practice
C. Hyperkalemia C. Have passed sub professional Career Service
Eligibility examination
96. Contraindications to thrombolysis as an intervention D. MS in Pharmacy or its equivalent
to acute coronary syndrome: E. With at least 18 units of post-graduate studies

A. One week postpartum D. A, B, C 104. Which of the following is a qualification for a


B. CNS Neoplasm E. A and C Supervising Pharmacist in a hospital?
C. Oral anticoagulant therapy
A. With at least 1 year pharmacy practice
97. When organizing a patient’s Problem-Oriented B. With at least 18 units of post-graduate studies
Medical Record (POMR), which of the following may be C. MS in Pharmacy or its equivalent
found on the last pages/ sections? D. With at least 3 years work experience
E. Have passed sub professional Career Service
A. Graphic charts D. Medication List Eligibility examination
B. Laboratory data E. Physicians Orders
C. Progress notes 105. Who among the following personnel prepares
reports of monthly consumption and balance of stocks?
98. Which of the following is NOT a clinical manifestation
of heart failure? A. Senior Pharmacist
B. Staff Pharmacist
A. Bradycardia D. Confusion C. Administrative Aide
B. Pallor E. Hepatomegaly D. Chief Pharmacist
C. Fatigue E. Laboratory Pharmacy Aide

99. What is the interaction of Tetracycline and milk? 106. Which of the following drugs is deemed
inappropriate for use in elderly patients?
A. Potentiation D. Enzyme inhibition
B. Drug displacement E. Addition A. Secobarbital D. Only A
C. Complexation B. Meprobamate E. A, B, C
C. Gluthetimide
100. In what section of the medical chart can one find
the patient’s Past Medical History (PMH)? 107. In the pharmacy functional chart, Poison Control is
part of which service?
A. Flow sheet D. Progress notes
B. Admit summary E. Patient interview A. Dispensing D. Research
C. Admission form B. Clinical E. Administrative
C. Compounding
101. Which of the following decreases theophylline
clearance? 108. Investigational Drug Study is part of which service
among the hospital pharmacy services?
A. Cigarette smoking
B. Viral pneumonia A. Dispensing D. Research
C. Phenytoin B. Clinical E. Administrative
D. Age (1-12 years old) C. Compounding
E. High-protein, low carbohydrate diet
109. Physical Examination results may be found in which
102. Who among the following is under the Chief section of the medical chart?
Pharmacist but above the other positions based on the
organizational chart of a hospital? A. Admit summary D. A, B, C
B. Progress Notes E. B and C
A. Staff Pharmacist C. Flow sheet
B. Chief Administrative Officer
C. Senior Pharmacist 110. Who is considered as the head of the Nutrition
D. Supervising Pharmacist Support Team?
E. Associate Pharmacist
A. Pharmacist D. Dietitian
B. Physician E. All of these
C. Nurse C. Error, With Harm

111. Carcinogenesis is an example of what type of ADR? 120. The green color coding for the medication cards is
for drugs administered:
A. Type A D. Type D
B. Type B E. Type E A. O.D D. Q.I.D
C. Type C B. B.I.D E. H.S
C. T.I.D
112. Syndromes that are typical of drug-induced
pulmonary disease, EXCEPT: 121. To which government agency should the hospital
submit the report on empty vials and ampoules crushed
A. Cardiogenic pulmonary edema and used as specimen container?
B. Asthma
C. Plural effusions A. DOH D. DTI
D. Interstitial fibrosis B. PDEA E. A and B
E. Pulmonary vascular disease C. FDA

113. Ethambutol- optic neuropathy is an example of what 122. Prescriptions for Phenobarbital ampule should be
type of ADR? filed for how long?

A. Ending of use D. Failure of Efficacy A. 6 months D. 5 years


B. Bizarre E. Augmented B. 1 year E. It is not filed.
C. Continuous B. 2 years

114. Deteriorated or outdated medication is a cause of 123. Effects more pronounced in Haloperidol than in
what type of medication error? Benperidol as antipsychotic agent:

A. Prescribing error D. B and C A. Sedation


B. Dispensing error E. A, B, C B. Reduce sexual drive
C. Administration error C. Tardive dyskinesia with long-term use
D. Hormonal effects
115. Which among the categories of medication error E. All except B
has the presence of error but poses no harm?
124. Which of the following may cause Extrapyramidal
A. Type A D. Type G Symptoms as its adverse effect?
B. Type C E. Type I
C. Type F A. Atropine D. Streptomycin
B. Codeine E. Nitroglycerin
116. Prescriptions for Phenobarbital tablet is filed for C. Phenothiazine
how long?
125. Which of the following age groups is considered as
A.6 months D. 3 years High-risk factor for atrial fibrillation?
B. 1 year E. 5 years
C. 2 years A. 20 to 35 years old
B. Below 65 years old
117. Drugs that are implicated in disorders of mood: C. 65 to 75 years old
D. Above 75 years old
A. Oral contraceptives D. B and C E. No risk seen or proven
B. Analgesics E. A, B, C
C. Steroids 126. Benzodiazepine- caused rebound insomnia is an
example of what category of Adverse Drug Reaction?
118. The following are found on the Flow Sheet section
of the medical chart, EXCEPT: A. Augmented D. Delayed
B. Bizarre E. Ending of Use
A. Physical Examination D. Laboratory tests C. Continuous
B. Pulmonary Function Test E. None of these
C. Chief Complaint 127. Which of the following is NOT considered as Class
A bioterrorism agent?
119. What does a Category I Medication Error mean?
A. Anthrax
A. No error D. Error, Death B. Boil
B. Error, No Harm E. No Error, With Harm C. Tularemia
D. Viral hemorrhagic fever C. Schilling Test
E. Botulinum toxin D. Serum ferritin
E. Total Reticulocyte Count
128. Which antiepileptic drug has the smallest starting
dose (in mg)? 136. Which of the following is recommended for severe
refractory atopic dermatitis condition?
A. Carbamazepine D. Gabapentin
B. Clonazelam E. Phenytoin A. Emollient D. Tacrolimus BID
B. Valproate B. Topical calcineurin inhibitor E. Phototherapy
C. Topical corticosteroid
129. This reflects the expansion of the role of the
hospital as to care programs and facilities: 137. Which is NOT to be followed when given a
prescription of “Gentamicin 80 mg IV PB q 8 hr”?
A. Organized health-care D. Patient care
B. Heath-system E. Pharmaceutical care A. Gentamicin is added to amni-bottle containing 100 mL
C. Institutional care 5% Dextrose Inj.
B. Dose is given every 8 hours
130. Headache as an adverse effect of nitroglycerin C. Drug administration is by piggyback technique.
belongs to what category of ADR? D. Flow rate required is at least 30 minutes.
E. None of these
A. Augmented D. Delayed
B. Bizarre E. End of Use 138. Which of the following symptoms is specifically
C. Continuous controlled with intranasal anticholinergics?

131. Which is an add-on therapy to the standard A. Nasal congestion D. Rhinorrhea


pharmacotherapy to hypertension with diabetes B. Sneezing E. Ocular symptoms
mellitus? C. Itching

A. Diuretic 139. This is the most common associated symptom for


B. ACE-inhibitor Osteoarthritis.
C. Calcium Channel Blocker
D. Angiotensin II Receptor Blocker A. Joint stiffness D. Joint deformity
E. Aldosterone Antagonist B. Crepitus E. Joint enlargement
C. Pain
132. Blood Coagulation factor XII is also known as
_____________ 140. Which of the following is LEAST considered as a
risk factor for Osteoporosis?
A. Proaccelerin D. Stuart-Power factor
B. Christmas factor E. Von Willebrand factor A. Low calcium intake
C. Hageman factor B. Male sex
C. Current cigarette smoking
133. The following are indications for TPN because D. Minimal sun exposure
enteral intake is inadvisable, EXCEPT: E. Estrogen deficiency before age 45

A. Short gut syndrome D. Acute Pancreatitis 141. Which of the following is NOT true regarding Drug
B. Renal failure E. Severe cachexia Usage Evaluation?
C. Acute hepatitis
A. It is the same as the Drug Utilization Review.
134. Possible solution when pain and immobility oocurs B. It is the same as drug therapy monitoring.
during the night in a Parkinson’s disease patient: C. It can be a research tool.
D. It is a justification of both distributive and clinical
A. Reduce antiparkinsonian therapy functions.
B. Use of slow-release levodopa E. It considers patterns of drug usage.
C. Consider therapy
D. Drug holiday for 1 week 142. Which of the following drugs/ agents will cause skin
E. More frequent doses of levodopa reactions the most?

135. A positive finding for which test may indicate A. Penicillin G D. Erythromycin
antibody-mediated hemolysis? B. Amoxicillin E. Ampicillin
C. Blood
A. Indirect Coomb’s Test
B. Direct Coomb’s Test
143. Which among the following chemotherapeutic drugs D. Optic Clearance Tomography
would LEAST induce emesis? E. Indocyamine Green Angiography

A. Fluorouracil D. Paclitaxel 151. What is the standard time to be followed for “q12”?
B. Cisplatin E. Bleomycin
C. Carboplatin A. 6 am – 6 pm D. 9 am- 9 pm
B. 7 am – 7 pm E. 12 nn – 12 mn
144. When the medication card is color coded violet, the C. 8 am – 8 pm
administration of the drug is:
152. Which among the following utensils/ equipment/
A. Per orem D. Stat apparatus is found in a 100-bed capacity, but not in a 10-
B. B.I.D E. Round the clock or 15-bed capacity hospital?
C. H.S
A. Mixer 2-L capacity D. Ring stand
145. Which among the following is NOT a required B. Calculating machine E. Ribbed funnel
utensil/ apparatus/ equipment in a 25-bed capacity C. Rough balance
hospital?
153. Which may be considered as an alternative drug for
A. Filing cabinet D. Tablet counter social anxiety disorder?
B. Funnel E. Analytical balance
C. Hot plate A. Buspirone D. Paroxetine
B. Fluvoxamine E. Clonazepam
146. This refers to the direct, responsible provision of C. Sertraline
medication-related care for the purpose of achieving
definite outcomes that improve a patient’s quality of life: 154. Which is NOT a category under the Laboratory
Data of the Patient Medical Chart?
A. Clinical Pharmacy
B. Pharmaceutical Care A. Microbiology D. Consults
C. Patient Care B. Urinalysis E. Type and Screens
D. Pharmaceutical Service C. Arterial Blood Gases
E. Pharmacy
155. This refers to a radiograph examination of kidneys,
147. The following drugs/ agents may increase blood ureters, urinary bladder and urethra with the use of
glucose level, EXCEPT: contrast media:

A. Oral contraceptives D. Nicotinic acid A. Barium Enema


B. Phenytoin E. Diuretics B. Barium Swallow
C. Salicylates C. Cholangiogram
D. Intravenous Pyelography
148. For chronic illnesses, what is the maximum number E. Retrograde Pyelography
of months to be supplied in a single dispensing of
medicines? 156. Drugs used in the treatment of Pneumocystis
jiroveci pneumonia (PCP) infection in HIV patient,
A. 1 D. 12 EXCEPT:
B. 3 E. Nothing specified
C. 6 A. Azithromycin D. Primaquine
B. Flucytosine E. Co-trimoxazole
149. Which of the following parenteral antibiotics used C. Dapsone
for pyelonephritis may cause nephrotoxicity and
ototoxicity? 157. Which of the following is NOT a dietary restriction
for patients using monoamine oxidase inhibitors?
A. Co-amoxiclav D. Ciprofloxacin
B. Cefuroxime E. Meropenem A. Ripe avocado D. Celery
C. Gentamicin B. Soy sauce E. Beer
C. Snails
150. What procedure provides information about the
circulatory system at the back of the eye not attainable 158. Which of the following characteristics is FALSE
with a routine examination? regarding Tourette’s Disorder?

A. Fundus photographs A. Echolalia D. Shoulder shrugging


B. Ocular Ultrasound B. Facial twitching E. Throat clearing
C. Fluorescein Angiography C. Age of onset after 60 years
159. This procedure is used to evaluate the size, shape, B. PNDF
and motion of the valves, septum and walls and changes C. Goodman and Gilman’s Pharmacological Basis of
in chamber size during the cardiac cycle. Therapeutics
D. USP-NF
A. Echo Cardiography E. Martindale’s Extra Pharmacopeia
B. Electrocardiogram
C. Coronary Angiography 167. Example of macrovascular complication in Type 2
D. Fluorescein Angiography Diabetes?
E. Indocyamine Green angiography
A. Nephropathy D. Abnormal ECG
160. What is the best definition for the abbreviation B. Ischemic skin changes E. Erectile dysfunction
AUC? C. Retinopathy

A. Area under the time curve 168. Recommended drug therapy for Stage 2
B. Area under the curve Hypertension:
C. Area under the serum- or blood- concentration-
versus- time curve A. Thiazide diuretic
D. Area under blood concentration curve B. ACE inhibitor + ARB agent
E. Area under serum- or blood- concentration curve C. ACE inhibitor or ARB agent
D. ACE inhibitor or ARB + CCB
161. Which of the following is TRUE regarding Type 2 E. Thiazide diuretic + ACE inhibitor or ARB or CCB
Diabetes Mellitus?
169. Clinical circumstance where hospitalization should
A. Predominantly secretory defect be considered and not just recommended for patient
B. Beta cell destruction presenting with acute decompensated heart failure?
C. Autoimmune
D. Idiopathic A. Dyspnea at rest
E. Absolute insulin deficiency B. Acute coronary syndromes
C. Hypotension
162. A platelet count of 1200 x 10 3 /mm3 may indicate D. Altered mentation
which condition? E. Major electrolyte disturbance

A. Vit B12 deficiency 170. Risk factors for major bleeding while on
B. Leukemia anticoagulant factor, EXCEPT:
C. Hemorrhage
D. Aplastic bone marrow A. Renal failure
E. Immune disorder B. Age <60 years
C. Heavy alcohol use
163. Which among the following is a SIGN of cardio D. Malignancy
pulmonary arrest? E. Recent history of surgery

A. Anxiety D. Nausea and vomiting 171. The following are just suggested references for
B. Cyanosis E. Dyspnea pharmacists in the hospital pharmacy, EXCEPT:
C. Diaphoresis
A. Cross Reference Index
164. Which of the following is NOT an indication of a B. Clinical Laboratory Medicine
WBC Count of 25 x 103/mm3? C. Medical Dictionary
D. Hospital formulary
A. Bone marrow suppression D. Typhoid fever E. None of these
B. Tissue necrosis E. Influenza
C. Rubella 172. A female patient presents a 55% Hct value. What
does this mean?
165. Characteristic of a Stage 1 Hypertension:
A. Polycythemia D. Hemodilution
A. DPB 80-89 D. DBP >100 B. Hemoconcentration E. A and/or B
B. SBP >160 E. SBP 120-139 C. Anemia
C. DBP 90-99
173. Which of the following required record books is
166. The following are required books in a hospital DOH hospital pharmacies exempted of?
pharmacy, EXCEPT:
A. Dangerous Drug Book
A. RPS B. Prescription Book
C. Poison Book 181. Which among the following is NOT part of the
D. Red Book Administrative Service of the hospital pharmacy?
E. Exempt Dangerous Drugs Book
A. Internship Program
174. What is elevated in a stage IV B. Financial Management
Hyperlipoproteinemia? C. Compliance to regulatory requirements
D. Drug supply management
A. Chylomicrons D. VLDL E. Records and reports
B. LDL E. IDL
C. HDL 182. Which is NOT a contraindication for
cardiopulmonary exercise testing?
175. Which oral hypoglycaemic agent will be
administered the MOST frequent? A. Uncontrolled hypertension
B. Congestive heart failure
A. Gliquidone D. Chlorpropamide C. Severe aortic stenosis
B. Pioglitazone E. Rosiglitazone D. Acute pericarditis
C. Glimepride E. Dyspnea upon exertion

176. This term is defined as “the extent to which a 183. Thiazolidinedione


person’s behavior-taking medication…corresponds with
agreed recommendations from a health care provider.” A. TZLD D. TLD4
B. TLD E. TADD
A. Patient compliance C. TZD
B. Medication adherence
C. Medication consumption 184. Forced expiratory volume in 1 second
D. Drug compliance
E. Medication counselling A. FEV1S D. FEV1
B. FEV E. FExV1
177. Which patient classification makes the patient pay C. FexpV1sec
only 50% of the charges of medicines given and ancillary
services rendered? 185. Which of the following is NOT true regarding the
gastrointesitinal adverse effects of sulfonylureas?
A. Class C-1 D. Class D
B. Class C-2 E. Class E A. Most commonly nausea and vomiting
C. Class C-3 B. Dose-related
C. Usually occurs in the first 2-6 weeks of therapy
178. The following are monitored when patient is under D. Affects approximately 2% of users
erythropoietis stimulant therapy, EXCEPT: E. May be avoided when taken with food

A. Blood pressure D. Blood sugar levels 186. At what stage of stable chronic obstructive
B. Iron level E. Ferritin level pulmonary disorder is a short acting bronchodilator
C. Complete blood count added in the therapy?

179. Which of the following is NOT a symptom of heart A. 0 D. III


failure? B. I E. IV
C. II
A. Hemoptysis D. Bloating
B. S2 gallop E. Exercise intolerance 187. AAT stands for __________
C. Nocturia
A. Amino Acid Treatment
180. What form of economic analyses estimates the ratio B. Alpha Acid Tolerance
between the cost of health-related intervention and the C. Alpha 1- antitrypsin
benefit it produces in terms of number oy years lived in D. Active Aortic Tachycardia
full health by the beneficiaries? E. Alveolar Activity Test

A. Cost Minimization Analysis 188. What form of economic analysis compares the
B. Cost Utility Analysis relative expenditure and outcomes of two or more
C. Cost Benefit Analysis courses of action?
D. Cost Interventional Analysis
E. Cost Effectiveness Analysis A. Cost Benefit Analysis
B. Cost Utility Analysis
C. Cost Effectiveness Analysis
196. Which of the following proton pump inhibitors is
D. Cost Interventional Analysis NOT commercially available as a tablet?
E. Cost Minimization Analysis A. Omeprazole
B. Pantoprazole
189. Pharmaceutical care requires a direct relationship C. Lansoprazole
between ___________. D. Esomeprazole
E. None of the given choices
A. Medical doctor and pharmacist
B. Pharmacist and patient 197. This compares the cost of therapies that achieve
C. Pharmacist and colleagues the same outcome:
D. Patient and Heath care team
E. Patient, Doctor, and Pharmacist A. Cost Minimization Analysis
B. Cost Operational Analysis
190. Anemia may be possible condition for the following C. Cost Effectiveness Analysis
values, EXCEPT: D. Cost Benefit Analysis
E. Cost Utility Analysis
A. Male patient 30% Hct
B. Platelet count 70 x 109L 198. Which of the following antiemetic preparations may
C. Female patient 6g/dL Hb NOT be given OTC?
D. Male patient 2.2 mill/mm3 RBC
E. Hb 80 g/L A. Cimetidine D. Diphenhydramine
B. Dimenhydrinate E. Cyclizine
191. Laboratory investigations as part of annual review C. Famotidine
for diabetes, EXCEPT:
199. Which of the following drugs may cause diarrhea?
A. Fasting lipid profile D. Creatinine
B. HbA1C E. Amylase test A. Tetracycline D. A and B
C. Liver function tests B. Methyldopa E. A, B, and C
C. Neostigmine
192. Which of the following drugs may NOT cause
gastrointestinal mucosa injury? 200. The following are Ex-Officio members of the
Pharmacy and Therapeutics Committee, EXCEPT:
A. Estrogen D. Reserpine
B. Ethanol E. Warfarin A. Chief Pharmacist
C. Aspirin B. Chief Nurse
C. Chief Administrative Officer
193. What is fundamental to the pharmacy profession’s D. PHIC Representative
purpose of helping people make the best use of E. None of the choices
medication?
201. Which is NOT a clinical manifestation of liver
A. Clinical Pharmacy cirrhosis?
B. Medication therapy management
C. Pharmaceutical care A. Pruritus D. Hyperpigmentation
D. Commitment B. Increased libido E. Spider angioma
E. Professional responsibility C. Weight loss

194. Endoscopy-negative reflux disease 202. The patient has a medication indication but is taking
the wrong medication. What category of
A.ENRD D. (-) ERD medication-related problem is indicated?
B. E (-) E. ERD-Neg.
C. E-RD A. Failure to receive medication
B. Medication use without indication
195. An increase in Sodium values may mean the C. Subtherapeutic dosage
following conditions, EXCEPT: D. Improper drug selection
E. Untreated indications
A. Adrenal insufficiency
B. Diabetes insipidus 203. Which is NOT included in the treatment approach
C. Aldosteronism for ascites?
D. Impaired renal function
E. None of these A. Diuretics D. Antibiotic therapy
B. Diet E. Intrahepatic shunt
C. Paracentesis
204. The following are possible medical conditions B. Decline in GFR
related to a triglyceride value of 185 mg/dL, EXCEPT: C. Increased BUN
D. B and C
A. Diabetes E. A, B, and C
B. Overconsumption of alcohol
C. Nephritic syndrome 212. Which of the following is a Nephritic sign?
D. Biliary obstruction
E. Pancreatitis A. Weight gain D. Hematuria
B. Edema E. All except D
205. Which of the following drugs has a definite C. Fatigue
association with acute pancreatitis?
213. Of the following steps of drug procurement cycle,
A. Furosemide D. Paclitaxel which comes first?
B. Propofol E. Ciprofloxacin
C. Rifampin A. Locate and select suppliers
B. Reconcile needs and funds
206. Cushing’s syndrome may be a possible condition C. Choose procurement method
as indicated by an increased value of which diagnostic D. Review drug selection
test? E. Collect consumption information

A. FBS D. Calcium 214. Which of the following has the highest Potassium
B. HDL E. Potassium content?
C. Triglyceride
A. Avocado D. Carrots
207. Somatic diagnostic criteria for Premenstrual B. Molasses E. Ground beef
Syndrome (PMS), EXCEPT: C. Banana

A. Headache 215. Gout may be possible for a male patient with a Uric
B. Depression Acid Value of?
C. Abdominal bloating
D. Swelling of extremities A. 0.13 mg/dL D. 8 mg/dL
E. Breast tenderness B. 1 mg/dL E. 11 mg/dL
C. 3 mg/dL
208. Para-amino hippuric acid
216. Drugs with negligible risk when breastfeeding
A.pHa D. PAHA full-term healthy babies:
B. p-aha E. pAHA
C. PAH A. Paracetamol D. Insulin
B. Digoxin E. All of these
209. Which of the following is NOT included in a hospital C. Codeine
formulary?
217. The patient has a medical problem that requires
A. Table of sodium content of antacids medication therapy but is not receiving a medication for
B. Metric conversion scales or tables that indication. What category of medication-related
C. Table of drug interactions problem is involved?
D. Poison antidote charts
E. None of these A. Failure to receive medication
B. Improper drug selection
210. The most appropriate initial therapy for the C. Untreated indications
management of dyslipidemia of patients with chronic D. Medication use without indication
kidney disease? E. Subtherapeutic dosage

A. Maximum-dose statin 218. For fast moving drugs, the preparation of request is
B. Niacin at ______ stock level.
C. Fibrate
D. Low-dose statin A. 25% D. 100%
E. Therapeutic lifestyle changes B. 50% E. Any
C. 75%
211. The most common manifestation of drug-induced
kidney disease: 219. Which of the following is NOT associated with a
high Total Cholesterol value?
A. Decrease in serum creatinine
A. Obstructive jaundice D. Hyperthyroidism 227. Which of the following is a CORRECT measure
B. Pancreatitis E. Hepatitis followed regarding proper storage in the hospital
C. CAD pharmacy?

220. Pharmaceutical care is applicable and achievable A. Heavier items should be placed in smaller pile.
by pharmacists in which practice setting? B. Varnish wood.
C. Bind sharp corners with tape.
A. Community setting D. A only D. A and C
B. Marketing E. A, B, C E. A, B, C
C. Research
228. The following drugs may cause secondary
221. In the procurement of drugs in the hospital, who parkinsonism, EXCEPT:
approves the purchase request?
A. Haloperidol D. α-methyldopa
A. Chief Pharmacist B. Olanzapine E. Carbidopa
B. Chief Administrative Officer C. Cinnarizine
C. Chief Medical Professional Staff
D. Chief of Hospital 229. Which of the following is a characteristic of acute
E. Supply Officer pain?

222. Which is NOT an example of Microcytic Anemia? A. Insomnia


B. Depression
A. Thalassemia C. Organic cause
B. IDA D. Dependence to medication
C. Sideroblastic anemia E. Functionality treatment goal
D. Anemia of chronic disease
E. A and C 230. Which of the following is NOT TRUE about classic
migraine?
223. Which of the following indicates a favorable
prognosis for multiple sclerosis? A. Migraine with aura
B. Basilar-type aura
A. <40 years old C. At least two attacks
B. Male gender D. Not attributed to another disorder
C. Cerebellar initial symptoms E. Headache attacks lasts to 72 hours
D. Progressive course of disease
E. High attack frequency 231. What drug classification usually serves as a starting
point for Drug Usage Evaluation in a hospital?
224. In the procurement of drugs in the hospital, who
prepares the Purchase Order? A. Anticoagulants D. Cardiac drugs
B. Antibiotics E. Insulin therapy
A. Chief of Hospital C. CNS drugs
B. Chief Pharmacist
C. Staff Pharmacist 232. Who is responsible for arranging dangerous drugs
D. Supply Officer systematically in a secured cabinet?
E. Pharmacy Administrative Aide
A. Laboratory Aide D. Senior Pharmacist
225. Which of the following is NOT a type of generalized B. Pharmacy technician E. Pharmacy Aide
seizure? C. Staff Pharmacist

A. Absence D. Infantile spasms 233. Features of megaloblastic anemia, EXCEPT:


B. Atonic E. Myoclonic
C. Status epilepticus A. Sterility D. Angular stomatitis
B. Anorexia E. Diarrhea
226. Basis of Glassgow Coma Scale in defining level of C. Severe jaundice
consciousness:
234. Which of the following is NOT TRUE regarding
A. Motor response D. A and B Rivastigmine as drug for Alzheimer’s Disease?
B. Verbal response E. A, B, C
C. Eye opening A. Available as tablet D. 40% protein binding
B. Take with food E. Available as Patch
C. 1.5 hours half-life
235. Which of the following pharmacologic agents is C. Penicillamine
NOT used in the treatment of alcohol withdrawal? D. Cyclosporine + methotrexate
E. Methotrexate + sulfasalazine + hydroxychloroquine
A. Clonidine D. Multivitamins
B. Fibrate E. Alcohol IV 244. Which is NOT TRUE regarding Type 1 Diabetes
C. Labetalol Mellitus?

236. Drug NOT associated with autoimmune hemolytic A. B-cell destruction


anemia: B. Idiopathic
C. Relative insulin deficiency
A. Cyclosporine D. Clarithromycin D. Immune-mediated
B. Quinine E. Mefenamic acid E. Absolute insulin deficiency
C. Levodopa
245. Which of the following is the most sensitive to heat?
237. Pharmaceutical care outcomes include:
A. Tetanus toxoid D. Hepatitis B
A. Cure of patient’s disease B. DTP E. A and C
B. Reduction of patient’s symptomatology C. BCG
C. Prevention of the disease
D. A and C 246. What model defines Health as “the absence of
E. A, B, C disease or disability”?

238. In a hospital setting, who among the following is A. Medical model


responsible for the detection of TPN complications? B. World Health Organization model
C. Wellness model
A. Gastroenterologist D. Nurse D. Environmental model
B. Primary care physician E. Dietitian E. All except D
C. Pharmacist
247. Which type of ADR may be prevented by
239. Which of the following is considered as a positive adjustment of dosage regimen?
symptom of schizophrenia?
A. Type A D. Type D
A. Hallucinations D. Alogia B. Type B E. Type E
B. Affective flattening E. Anhedonia C. Type C
C. Avolition
248. Which of the following is TRUE regarding Adrenal
240. Which of the following is NOT included among the Carcinoma Cushing’s syndrome?
contents of Cytotoxic Spill Kit?
A. Slow course
A. Overshoes D. Head covering B. Dominant among children
B. Absorbent material E. None of these C. Atypical symptoms
C. Detergent D. Mild to moderate symptoms
E. Dominant among males
241. A dopamine agonist used to treat extrapyramidal
side effects: 249. “I hear you saying it bothers you that your
headaches don’t respond to aspirin,” is an example of
A. Benztropine D. Amantadine what type of questioning strategy?
B. Biperiden E. Trihexyphenidyl
C. Lorazepam A. Open-ended D. Indirect
B. Close-ended E. Probe
242. Which of the following vaccines is ideally stored in a C. Direct
-15 degree celcius to -25 degree celcius?
250. Which of the following is NOT considered as a
A.DTP D. Hepatitis B pharmacotherapeutic option in patients with urinary
B. Oral Polio E. Tetanus Toxoid incontinence due to stress?
C. BCG
A. Alpha adrenergic agonist
243. Which among the following therapies may pose B. Tricyclic antidepressant
more toxicity? C. Estrogen
D. Imipramine
A. Methotrexate + sulfasalazine E. Duloxetine
B. Cyclosporine + hydroxychloroquine
251. Which method is used for the computation of dose C. Diaphoresis
for infants up to 2 years old?
260. The following medicines are maintained stocks in
A. Clark’s Rule D. All of these the emergency room, EXCEPT:
B. Fried’s Rule E. A and C
C. Young’s Rule A. Naloxone D. Regular Insulin
B. Furosemide E. Calcium gluconate
252. An ADR that has little or no relation to usual C. Amiloride
pharmacological effects of the drug is of what type?
261. Which of the following is an add-on
A. Type A D. Type D pharmacotherapy to hypertension with post-myocardial
B. Type B E. Type E infarction condition?
C. Type C
A. Diuretic
253. Which of the following is a clinical presentation of B. Aldosterone antagonist
systemic sclerosis? C. Calcium channel blocker
D. Angiotensin II receptor blocker
A. Dyspepsia D. Diarrhea E. Beta blocker
B. Constipation E. All of these
C. Steatorrhea 262. This alternative antihypertensive agent is to be
used only once weekly:
254. Type II Allergic Drug Reaction:
A. Doxazosin D. Clonidine
A. Cytotoxic D. Anaphylactic B. Minoxidil E. Aliskiren
B. Immune complex E. Tuberculin reaction C. Methyldopa
C. Cell-mediated
263. Which of the following information is classified
255. Which of the following may decrease Cyclosporine under the Medical Section?
levels when given concomitantly?
A. Past medical history D. Home remedies
A. Verapamil D. Erythromycin B. Medication regimen E. Medication allergies
B. Ketoconazole E. Nicardipine C. Health plan
C. Phenytoin
264. Which antihypertensive agent is administered in
256. Constipation is an ADR of which drug? most hypertensive emergencies except in acute heart
failure?
A. Tetracycline D. Captopril
B. Propanolol E. Streptomycin A. Labetalol HCl
C. Codeine B. Sodium nitroprusside
C. Fenoldopam mesylate
257. The following are clinical presentation of acute D. Hydralazine HCl
acetaminophen poisoning, EXCEPT: E. Esmolol HCl

A. Jaundice 265. An error occurred that resulted in permanent patient


B. Oliguria harm; this belongs to which category of medication
C. Abdominal discomfort error?
D. Left upper abdominal quadrant tenderness
E. Nausea and vomiting A. Category A D. Category G
B. Category D E. Category I
258. Which is NOT a test for cardiac electrical rhythm? C. Category E

A. Angioscopy 266. Which of the following can cause negative inotropic


B. Holter monitoring effect and may precipitate heart failure?
C. Nuclear imaging
D. Electrophysiologic studies A. Ethanol D. Rosiglitazone
E. ECG B. Cocaine E. Glucocorticoids
C. Terbinafine
259. Which of the following is NOT included in the
clinical presentation of cardiopulmonary arrest? 267. The maximum number of vials of Demerol that may
be prescribed in a single applicable prescription by a
A. Hypertension D. Cyanosis licensed physician:
B. Dyspnea E. Anxiety
A. 3 D. 20 C. Shifting body position
B. 10 E. 30 D. Frequent throat clearing
C. 14 E. Steepling the hands

268. Which is NOT a good source of protein? 276. Who among the following is an allied health care
professional?
A. Cheese D. Fish
B. Peas E. Potatoes A. Athletic trainer D. A and B
C. Egg B. Medical illustrator E. A, B, C
C. Exercise scientist
269. Which of the following Amiodarone side effects may
NOT be managed by discontinuation of the drug? 277. What kind of questions should be asked early
during a patient interview?
A. Hypothyroidism D. Tremors
B. Pulmonary fibrosis E. Optic neuropathy A. Questions that can be answered by Yes or No
C. Bradycardia B. Multiple questions
C. Long, complex questions
270. Hospital reports on Adverse Drug Reactions are D. Leading questions
submitted to which government agency? E. Open-ended questions

A. PDEA D. B and C 278. What is the alcohol use category of a male patient
B. DOH E. A, B, C who drinks three drinks on average each week?
C. FDA
A. Binge D. Infrequent
271. What is considered as the gold standard for the B. Moderate E. Current
diagnosis of pulmonary embolism? C. Light

A. Computed tomography 279. A patient states that he has smoked 3 packs of


B. Complete Blood Count cigarettes a day for the past 20 years. What is the
C. Chest X-Ray patient’s pack-year smoking history?
D. Pulmonary angiography
E. Ventilation-perfusion A. 3 pk-yr D. 20 pk-yr
B. 6 pk-yr E. 60 pk-yr
272. Which is NOT found on the flow sheets and graphic C. 10 pk-yr
chart section of the medical record?
280. A patient is troubled by shortness of breath when
A. Blood pressure hurrying on a level surface. What does he have?
B. Respiratory rate
C. Fluid intake and output A. No dyspnea
D. Heart rate B. Slight dyspnea
E. Arterial blood gas analysis C. Moderate dyspnea
D. Severe dyspnea
273. Which of the following is an example of an objective E. Very Severe Dyspnea
data?
281. Which of the following drugs will most likely induce
A. Decreased swelling of feet apnea?
B. Decreased edema on CXR
C. Increased SOB A. Anesthetics D. Antihistamines
D. Increase exercise tolerance B. Ketamine E. Digitalis
E. Sleeps with fewer pillows C. Barbiturates

274. For medication cards, what is the color coding for 282. Which of the following drugs would least likely
“at bedtime” drugs? induce bronchospasm?

A. White D. Pink A. Cimetidine D. Papain


B. Yellow E. Red B. Penicillin E. Bisulfite
C. Orange C. Cephalosphorin

275. Which body language implies receptiveness? 283. Which of the following would LEAST worsen GERD
symptoms?
A. Leaning toward the speaker
B. Rising the hand A. Dopamine D. Milk
B. Onions E. Peppermint B. New heart mjurmur E. A, B, and C
C. Tetracycline C. Roth spots

284. For intermittent, mild heartburn, which of the 292. Which of the following may NOT indicate a chronic
following approaches is NOT appropriate? kidney disease?

A. Ranitidine twice daily A. Weight gain D. Gout


B. Gaviscon 2 tabs before meals B. Edema E. Hyperkalemia
C. Maalox 30 mL as needed C. GERD
D. Omeprazole once daily
E. Calcium carbonate 2 tabs as needed 293. Which of the following is characteristic of
hyponatremia?
285. Thiopurine S-methyltransferase
A. Increased skin turgor
A. TS-M D. TMT B. Dry mucous membranes
B. TPSMT E. TSMT C. Bradycardia
C. TPMT D. Increased JVP
E. Noncardiogenic pulmonary edema
286. Which of the following is a cause of Ascites?
294. Which of the following statements regarding
A. Hyperdynamic circulation hypokalemia is FALSE?
B. Sodium and water retention
C. Renal vasoconstriction A. Mild hypokalemia is often asymptomatic.
D. Gravity B. Moderate hypokalemia is associated with malaise.
E. A, B and C C. In severe hypokalemia, ECG often changes.
D. Digitalis-induced arrhythmia may be caused by
287. Which of the following drug information does not severe hypokalemia.
belong to the group? E. Hypermagnesemia is also present.

A. Current symptoms 295. In which domain of the neurologic examination is


B. Compliance with therapy myasthenia gravis tested?
C. Laboratory information
D. Weight and height\ A. Gait D. Mental status
E. Past medical history B. Reflexes E. Sensory function
C. Cranial nerves
288. Which of the following is present in a severe acute
pancreatitis? 296. Which of the following pairs of patient information
and category is CORRECT?
A. Pseudocyst D. B and C
B. Necrosis E. A, B, and C A. Consent Forms – Administrative
C. Abscess B. Vital Signs – Demographic
C. Living arrangement – Lifestyle
289. Which is NOT true regarding steatorrhea? D. Medication regimen – Medical
E. Occupation – Economic
A. It is mostly seen among chronic pancreatitis patients.
B. It is the loss of excess proteins in feces. 297. Type of Question: “Does your headache respond to
C. It is associated with diarrhea. aspirin?”
D. Bloating is associated with such.
E. It is the loss of excess fat in the feces. A. Open-ended question D. Indirect question
B. Close-ended question E. Probe
290. Hepatitis C Virus genotype 6 is mostly distributed in C. Direct question
which region?
298. Which interview questions may be used to assess
A. Africa D. Middle East mania?
B. India E. Southeast Asia
C. Worldwide A. Do you cry without any reason?
B. Do you have a lot of extra energy?
291. Which of the following physical examination C. Do you have any guilty feelings?
findings in person with acute renal failure may indicate D. Has your weight changed recently?
endocarditis? E. Do you still enjoy the same hobbies that you once
did?
A. Splinter hemorrhages D. Rales
299. When a patient has difficulty remembering recent
events, what stage of Alzheimer’s disease does he
exhibit?

A. Mild D. Very Severe


B. Moderate E. Relapsing
C. Severe

300. Color coding for medication card for STAT drugs:

A. White D. Pink
B. Red E. Orange
C. Yellow
Subject:

General Instructions:

HOSPITAL AND CLINICAL PHARMACY

Select the letter of the BEST answer.

_____1. These are preparations intended for injection through the skin or other external boundary
tissue, rather than the alimentary canal.

A) Parenteral articles
B) Large-volume intravenous solutions
C) Small-volume injections
D) Intravenous push
E) Intravenous piggyback
Thompson JE and Davidow LW (2004). A Practical Guide to Contemporary Pharmacy Practice,
Second Edition. Lippincott Williams and Wilkins p 32.4

_____2. These are single dose injections containing more than 100mL of solution that are intended
for intravenous use.

A) Parenteral articles
B) Large-volume intravenous solutions
C) Small-volume injections
D) Intravenous push
E) Intravenous piggyback
Thompson JE and Davidow LW (2004). A Practical Guide to Contemporary Pharmacy Practice,
Second Edition. Lippincott Williams and Wilkins p 32.4

_____3. These are injections of 100mL or less and may either be single-dose and multi-dose
products.

A) Parenteral articles
B) Large-volume intravenous solutions
C) Small-volume injections
D) Intravenous push
E) Intravenous piggyback
Thompson JE and Davidow LW (2004). A Practical Guide to Contemporary Pharmacy Practice,
Second Edition. Lippincott Williams and Wilkins p 32.4
_____4. This term is used to designate the probability of finding a non-sterile unit following a
sterilization step or procedure.

A) Sterility
B) Sterilization assurance level
C) Sterilization
D) Terminal sterilization
E) Aseptic processing
Thompson JE and Davidow LW (2004). A Practical Guide to Contemporary Pharmacy Practice,
Second Edition. Lippincott Williams and Wilkins p 32.5

______5. This is a procedure carried out at the end of processing, when a product is in its final
sealed container that destroys all viable organisms.

A) Sterility
B) Sterilization assurance level
C) Sterilization
D) Terminal sterilization
E) Aseptic processing
Thompson JE and Davidow LW (2004). A Practical Guide to Contemporary Pharmacy Practice,
Second Edition. Lippincott Williams and Wilkins p 32.5

_____6. This term refers to processing operations involving sterile products that are carried out in a
laminar airflow workbench or barrier isolator.

A) Sterility
B) Sterilization assurance level
C) Sterilization
D) Terminal sterilization
E) Aseptic processing
Thompson JE and Davidow LW (2004). A Practical Guide to Contemporary Pharmacy Practice,
Second Edition. Lippincott Williams and Wilkins p 32.5

______7. These are workbenches that provide an environment of specially filtered air that sweeps
the work area and provides an aseptic work area.

A) HEPA filter
B) Laminar flow
C) Laminar flow hood
D) Barrier isolator
E) Critical surfaces
Thompson JE and Davidow LW (2004). A Practical Guide to Contemporary Pharmacy Practice,
Second Edition. Lippincott Williams and Wilkins p 32.5

______8. This area is designed and maintained to prevent particulate and microbiological
contamination of drug products as they are being prepared or processed.

A) Anteroom
B) Clean room
C) Critical surface
D) Controlled area
E) Critical area
Thompson JE and Davidow LW (2004). A Practical Guide to Contemporary Pharmacy Practice,
Second Edition. Lippincott Williams and Wilkins p 32.7

______9. In this type of intravenous administration, the drug is added to a large volume parenteral
solution and the solution is then slowly and continuously dripped into a vein.

A) Continuous IV infusion
B) Intermittent IV infusion
C) IV push
D) IV injection
E) IV bolus
Thompson JE and Davidow LW (2004). A Practical Guide to Contemporary Pharmacy Practice,
Second Edition. Lippincott Williams and Wilkins p 32.15

______10. In this type of intravenous administration, the drug is added to an intermediate volume
(25-100mL) and given in an intermediate period of time (15-60minutes), at spaced interval, such as
every 6 hours.

A) Continuous IV infusion
B) Intermittent IV infusion
C) IV push
D) IV injection
E) IV bolus
Thompson JE and Davidow LW (2004). A Practical Guide to Contemporary Pharmacy Practice,
Second Edition. Lippincott Williams and Wilkins p 32.16

______11. It is the feeding of a patient by the intravenous infusion of fluids and basic nutrients.

A) Total parenteral nutrition


B) Partial parenteral nutrition
C) Parenteral infusion
D) Enteral nutrition
E) IV bolus
Ansel HC and Stoklosa MJ (2006). Pharmaceutical Calculations, 12th Edition. Lippincott Williams and
Wilkins p 215

_____12. It is a method of providing nutrition support via tubes inserted into the stomach or small
intestine.

A) Total parenteral nutrition


B) Partial parenteral nutrition
C) Parenteral infusion
D) Enteral nutrition
E) IV bolus
Ansel HC and Stoklosa MJ (2006). Pharmaceutical Calculations, 12th Edition. Lippincott Williams and
Wilkins p 213

_____13. This nutritional support supplements oral intake and provides only part of daily nutritional
requirements.

A) Total parenteral nutrition


B) Partial parenteral nutrition
C) Parenteral infusion
D) Enteral nutrition
E) IV bolus
Ansel HC and Stoklosa MJ (2006). Pharmaceutical Calculations, 12th Edition. Lippincott Williams and
Wilkins p 215

_____14. These preparations consist of one or more sterile drug products, added to an IV fluid,
generally dextrose or sodium chloride solution alone or in combination.

A) Total parenteral nutrition


B) Partial parenteral nutrition
C) Parenteral infusion
D) Enteral nutrition
E) IV bolus
Comprehensive Pharmacy Review 6th ed., Shargel, Leon, Mutnick, Alan, Souney, Paul and
Swanson, Larry. Lippincott Williams and Wilkins p 567

______15. It is a therapeutic agent whose place in therapy is well-established.

A) Nonformulary drug
B) Formulary drug
C) Restricted drug
D) Investigational drug
E) Clinical evaluation drug
Durgin JM and Hanan ZI (1994). Pharmacy Practice for Technicians. Delmar Publishers Inc. p 433

______16. It is a commercially available, nonformulary agent that is temporarily made available to a


particular physician for the purpose of evaluation.

A) Nonformulary drug
B) Formulary drug
C) Restricted drug
D) Investigational drug
E) Clinical evaluation drug
Durgin JM and Hanan ZI (1994). Pharmacy Practice for Technicians. Delmar Publishers Inc. p 433

_____17. It is a therapeutic agent, admitted to the formulary, the use of which is authorized by a
specific group of physicians designated by the committee.

A) Nonformulary drug
B) Formulary drug
C) Restricted drug
D) Investigational drug
E) Clinical evaluation drug
Durgin JM and Hanan ZI (1994). Pharmacy Practice for Technicians. Delmar Publishers Inc. p 433

_____ 18.It is a therapeutic agent undergoing clinical investigation.

A) Nonformulary drug
B) Formulary drug
C) Restricted drug
D) Investigational drug
E) Clinical evaluation drug
Durgin JM and Hanan ZI (1994). Pharmacy Practice for Technicians. Delmar Publishers Inc. p 434

_____19. This is a mistake in prescribing, dispensing, or planned medication administration that is


detected and corrected through intervention (by another health-care provider or patient) before
actual medication administration.

A) Medication error
B) Potential error
C) Prescribing error
D) Omission error
E) Wrong time error
ASHP Guidelines on Preventing Medication Errors in Hospitals p 155

_____ 20. This includes incorrect drug selection, dose, dosage form, quantity, route, concentration,
rate of administration, or instructions for use of a drug product ordered or authorized by a physician.

A) Medication error
B) Potential error
C) Prescribing error
D) Omission error
E) Wrong time error
ASHP Guidelines on Preventing Medication Errors in Hospitals p 156

______21. This includes illegible prescriptions or medication orders that lead to errors that reach the
patient.

A) Medication error
B) Potential error
C) Prescribing error
D) Omission error
E) Wrong time error
ASHP Guidelines on Preventing Medication Errors in Hospitals p 156

_______22. The failure to administer an ordered dose to a patient before the next scheduled dose, if
any.

A) Medication error
B) Potential error
C) Prescribing error
D) Omission error
E) Wrong time error
ASHP Guidelines on Preventing Medication Errors in Hospitals p 156

_______ 23. Administration of medication outside a predefined time interval from its scheduled
administration time (this interval should be established by each individual health care facility).

A) Medication error
B) Potential error
C) Prescribing error
D) Omission error
E) Wrong time error
ASHP Guidelines on Preventing Medication Errors in Hospitals p 156

______ 24. Administration to the patient of medication not authorized by a legitimate prescriber.

A) Omission error
B) Unauthorized drug error
C) Wrong dosage form error
D) Improper dose error
E) Wrong time error
ASHP Guidelines on Preventing Medication Errors in Hospitals p 156

______ 25. Administration to the patient of a dose that is greater than or less than the amount
ordered by the prescriber or administration of duplicate doses to the patient i.e., one or more dosage
units in addition to those that were ordered.

A) Omission error
B) Unauthorized drug error
C) Wrong dosage form error
D) Improper dose error
E) Wrong time error
ASHP Guidelines on Preventing Medication Errors in Hospitals p 156

______26. Administration to the patient of a drug product in a different dosage form than ordered by
the prescriber.

A) Omission error
B) Unauthorized drug error
C) Wrong dosage form error
D) Improper dose error
E) Wrong time error
ASHP Guidelines on Preventing Medication Errors in Hospitals p 156

______ 27. Drug product incorrectly formulated or manipulated before administration.

A) Deteriorated drug error


B) Wrong administration-technique error
C) Wrong drug preparation error
D) Monitoring error
E) Wrong drug prescription
ASHP Guidelines on Preventing Medication Errors in Hospitals p 156

______ 28. Inappropriate procedure or improper technique in the administration of a drug.

A) Deteriorated drug error


B) Wrong administration-technique error
C) Wrong drug preparation error
D) Monitoring error
E) Wrong drug prescription
ASHP Guidelines on Preventing Medication Errors in Hospitals p 156

______ 29. Administration of a drug that has expired or for which the physical or chemical dosage
form integrity has been compromised.

A) Deteriorated drug error


B) Wrong administration-technique error
C) Wrong drug preparation error
D) Monitoring error
E) Wrong drug prescription
ASHP Guidelines on Preventing Medication Errors in Hospitals p 156

______ 30. Failure to review a prescribed regimen for appropriateness and detection of problems, or
failure to use appropriate clinical or laboratory data for adequate assessment of patient response to
prescribed therapy.

A) Deteriorated drug error


B) Wrong administration-technique error
C) Wrong drug preparation error
D) Monitoring error
E) Wrong drug prescription
ASHP Guidelines on Preventing Medication Errors in Hospitals p 156

______31. Inappropriate patient behavior regarding adherence to a prescribed medication regimen.

A) Omission error
B) Unauthorized drug error
C) Compliance error
D) Improper dose error
E) Wrong time error
ASHP Guidelines on Preventing Medication Errors in Hospitals p 156

______ 32. This medication error severity level indicates that an error occurred did not result in
patient harm.

A) Level 0
B) Level 1
C) Level 2
D) Level 3
E) Level 4
ASHP Guidelines on Preventing Medication Errors in Hospitals p 162

______ 33. This medication error severity level indicates that an error occurred resulted in the need
for increased patient monitoring but no change in vital signs and no patient harm.

A) Level 0
B) Level 1
C) Level 2
D) Level 3
E) Level 4
ASHP Guidelines on Preventing Medication Errors in Hospitals p 162

_____ 34. Non medication error occurred.

A) Level 0
B) Level 1
C) Level 2
D) Level 3
E) Level 4
ASHP Guidelines on Preventing Medication Errors in Hospitals p 162

______ 35. The medication error severity level indicates that an error occurred resulted in the need
for increased patient monitoring with a change in vital signs but to ultimate patient harm, or any error
that resulted in the need for increased laboratory monitoring.

A) Level 0
B) Level 1
C) Level 2
D) Level 3
E) Level 4
ASHP Guidelines on Preventing Medication Errors in Hospitals p 163

______ 36. This medication error severity level indicates that an error occurred resulted in the need
for treatment with another drug or an increased length of stay or that affected patient participation in
an investigational study.

A) Level 2
B) Level 3
C) Level 4
D) Level 5
E) Level 6
ASHP Guidelines on Preventing Medication Errors in Hospitals p 162
______ 37. This medication error severity level indicates that an error occurred resulted in
permanent patient harm.

A) Level 2
B) Level 3
C) Level 4
D) Level 5
E) Level 6
ASHP Guidelines on Preventing Medication Errors in Hospitals p 163

______ 38. This medication error severity level indicates that an error occurred resulted in patient
death.

A) Level 2
B) Level 3
C) Level 4
D) Level 5
E) Level 6
ASHP Guidelines on Preventing Medication Errors in Hospitals p 163

______ 39. This is a continually updated list of medications and related information, representing the
clinical judgment of physicians, pharmacists, and other experts in the diagnosis, prophylaxis, or
treatment of disease and promotion of health.

A) Formulary system
B) Formulary
C) Drug monograph
D) Policy and procedure manual
E) Operating manual
ASHP Guidelines on the Pharmacy and Therapeutics Committee and the Formulary System

______ 40. An on-going process whereby a healthcare organization, through its physicians,
pharmacists, and other healthcare professionals, establishes policies on the use of drug products
and therapies and identifies drug products and therapies that are most medically appropriate and
cost-effective to best serve the health interests of a given patient population.

A) Formulary system
B) Formulary
C) Drug monograph
D) Policy and procedure manual
E) Operating manual
ASHP Guidelines on the Pharmacy and Therapeutics Committee and the Formulary System
_______ 41. The substitution of drug products that contain the same active ingredient or ingredients
and are chemically identical in strength, concentration, dosage form, and route of administration to
the drug product prescribed.

A) Formulary system
B) Therapeutic Interchange
C) Therapeutic Alternatives
D) Generic Substitution
E) Therapeutic Substitution
ASHP Guidelines on the Pharmacy and Therapeutics Committee and the Formulary System

_________42. The act of dispensing a therapeutic alternative for the drug product prescribed without
prior authorization of the prescriber. This is an illegal act because only the prescriber may authorize
an exchange of therapeutic alternatives.

A) Formulary system
B) Formulary
C) Drug monograph
D) Policy and procedure manual
E) Operating manual
ASHP Guidelines on the Pharmacy and Therapeutics Committee and the Formulary System

________43. Authorized exchange of therapeutic alternatives in accordance with previously


established and approved written guidelines or protocols within a formulary system.

A) Formulary system
B) Formulary
C) Drug monograph
D) Policy and procedure manual
E) Operating manual
ASHP Guidelines on the Pharmacy and Therapeutics Committee and the Formulary System

________44. Drug products with different chemical structures but of the same pharmacologic or
therapeutic class and usually have similar therapeutic effects and adverse-reaction profiles when
administered to patients in therapeutically equivalent doses.

A) Formulary system
B) Formulary
C) Drug monograph
D) Policy and procedure manual
E) Operating manual
ASHP Guidelines on the Pharmacy and Therapeutics Committee and the Formulary System

________45. The use of a drug prescribed for an indication nor specifically approved by FDA is
often referred to as:

A) Formulary system
B) Therapeutic Interchange
C) Off-label use
D) Generic Substitution
E) Therapeutic Substitution
ASHP Guidelines on the Pharmacy and Therapeutics Committee and the Formulary System

________46.An advisory committee that is responsible for developing, managing, updating and
administering a formulary system.

A) Formulary system
B) Therapeutic Interchange
C) Off-label use
D) Pharmacy and Therapeutics Committee
E) Therapeutic Substitution
ASHP Guidelines on the Pharmacy and Therapeutics Committee and the Formulary System

________47. It is an immunologic hypersensitivity that occurs as the result of unusual sensitivity to a


drug formulary system.

A) Allergic reaction
B) Idiosyncratic reaction
C) Adverse drug reaction
D) Serious adverse event
E) Side effect
ASHP Guidelines on Adverse Drug Reaction Monitoring and Reporting p 126

_____ 48. It is an abnormal susceptibility to a drug that is peculiar to an individual.

A) Allergic reaction
B) Idiosyncratic reaction
C) Adverse drug reaction
D) Serious adverse event
E) Side effect
ASHP Guidelines on Adverse Drug Reaction Monitoring and Reporting p 126

_____ 49. Any response to a drug which is noxious and unintended, and which occurs at doses
normally used in man for prophylaxis, diagnosis, or therapy of disease, or for the modification of
physiological function.

A) Allergic reaction
B) Idiosyncratic reaction
C) Adverse drug reaction
D) Serious adverse event
E) Side effect
ASHP Guidelines on Adverse Drug Reaction Monitoring and Reporting p 126

____50. It is expected, well-known reaction resulting in little or no change in patient management.


A) Allergic reaction
B) Idiosyncratic reaction
C) Adverse drug reaction
D) Serious adverse event
E) Side effect
ASHP Guidelines on Adverse Drug Reaction Monitoring and Reporting p 126

____51. The daily intravenous requirement of sodium is ____________.

A) 0.1-0.15 mmol/Kg
B) 0.1-0.2 mmol/Kg
C) 0.5-0.7 mmol/Kg
D) 1-2 mmol/Kg
E) 2-4 mmol/Kg
Winfield AJ and Richards RME (1998). Pharmaceutical Practice, Second Edition. Churchill Living
stone p 278

___52. The daily intravenous requirement of potassium is ____________.

A) 0.1-0.15 mmol/Kg
B) 0.1-0.2 mmol/Kg
C) 0.5-0.7 mmol/Kg
D) 1-2 mmol/Kg
E) 2-4 mmol/Kg
Winfield AJ and Richards RME (1998). Pharmaceutical Practice, Second Edition. Churchill Living
stone p 278

___53. The daily intravenous requirement of magnesium is ____________.

A) 0.1-0.15 mmol/Kg
B) 0.1-0.2 mmol/Kg
C) 0.5-0.7 mmol/Kg
D) 1-2 mmol/Kg
E) 2-4 mmol/Kg
Winfield AJ and Richards RME (1998). Pharmaceutical Practice, Second Edition. Churchill Living
stone p 278

___54. The daily intravenous requirement of calcium is ____________.

A) 0.1-0.15 mmol/Kg
B) 0.1-0.2 mmol/Kg
C) 0.5-0.7 mmol/Kg
D) 1-2 mmol/Kg
E) 2-4 mmol/Kg
Winfield AJ and Richards RME (1998). Pharmaceutical Practice, Second Edition. Churchill Living
stone p 278
______55. The daily intravenous requirement of phospate is ____________.

A) 0.1-0.15 mmol/Kg
B) 0.1-0.2 mmol/Kg
C) 0.5-0.7 mmol/Kg
D) 1-2 mmol/Kg
E) 2-4 mmol/Kg
Winfield AJ and Richards RME (1998). Pharmaceutical Practice, Second Edition. Churchill Living
stone p 278

___56. The daily intravenous requirement of chloride is ____________.

A) 0.1-0.15 mmol/Kg
B) 0.1-0.2 mmol/Kg
C) 0.5-0.7 mmol/Kg
D) 1-2 mmol/Kg
E) 2-4 mmol/Kg
Winfield AJ and Richards RME (1998). Pharmaceutical Practice, Second Edition. Churchill Living
stone p 278

_____57. This occurs when the effects of one drug is altered by the effects of another drug.

A) Medication error
B) Adverse drug reactions
C) Drug interactions
D) Pharmacodynaminc interactions
E) Pharmacokinetic interactions
Winfield AJ and Richards RME (1998). Pharmaceutical Practice, Second Edition. Churchill Living
stone p 361

_____ 58. This occurs when one drug alters the effect of another by acting at the same site of action,
on the same pharmacological receptors on the same pharmacological system.

A) Medication error
B) Adverse drug reactions
C) Drug interactions
D) Pharmacodynaminc interactions
E) Pharmacokinetic interactions
Winfield AJ and Richards RME (1998). Pharmaceutical Practice, Second Edition. Churchill Living
stone p 361

___59. This occurs when one drug alters the absorption, distribution, metabolism or excretion of
another drug.

A) Medication error
B) Adverse drug reactions
C) Drug interactions
D) Pharmacodynaminc interactions
E) Pharmacokinetic interactions
Winfield AJ and Richards RME (1998). Pharmaceutical Practice, Second Edition. Churchill Living
stone p 361

___60. It is the advancement of medical knowledge against diseases.

A) Education
B) Patient care
C) Public health
D) Research
E) All of the above
Philip P. Gerbino. Remington: The Science and Practice of Pharmacy, 21st ed., p. 2251

___61. This is where ambulatory patients are seen by appointment and where patient is not
confined.

A) Ambulatory surgery center


B) Clinic
C) Long-term health care facility
D) Resident treatment facility
E) None of the above
Philip P. Gerbino. Remington: The Science and Practice of Pharmacy, 21st ed., p. 2249

____62. Independent, voluntary organization that maintains and elevates the standards of health
care.

A) ASHP
B) AHA
C) JCAHO
D) ISO
E) PSHP
Philip P. Gerbino. Remington: The Science and Practice of Pharmacy, 21st ed., p. 2251

___63. Represents an investment interest of their owners and profits are legally shared among the
owners.

A) Federal hospital
B) Non-profit hospital
C) Profit hospital
D) State hospital
E) County hospital
Philip P. Gerbino. Remington: The Science and Practice of Pharmacy, 21st ed., p. 2250

___64. Function of hospital where it can provide practical learning experiences dealing with saving
human lives.

A) Education
B) Patient care
C) Public health
D) Research
E) All of the above
Philip P. Gerbino. Remington: The Science and Practice of Pharmacy, 21st ed., p. 2250

___65. Must produce a two-way channel of communication between the board, the hospital staff and
personnel.

A) Chairman
B) Chief executive officer
C) Governing body
D) Medical staff
E) Hospital staff
Philip P. Gerbino. Remington: The Science and Practice of Pharmacy, 21st ed., p. 2252

___66. Patients with chronic cases are confined in:

A) Governmental hospital
B) Long-term hospital
C) Private hospital
D) Short-term hospital
E) General hospital
Philip P. Gerbino. Remington: The Science and Practice of Pharmacy, 21st ed., p. 2250

___67. Public health activities include all except:

A) Poison prevention measures


B) Disease detection measures
C) Smoking cessation measures
D) Teaching routine hygienic practices
E) Clinical evaluation of investigational drugs
Philip P. Gerbino. Remington: The Science and Practice of Pharmacy, 21st ed., p. 2251

____68. Internal factor that affects the practice of hospital pharmacy.

A) Accreditation agency
B) Licensing agency
C) Organizational structure of the hospital
D) Third-party payer
E) Federal government
Philip P. Gerbino. Remington: The Science and Practice of Pharmacy, 21st ed., p. 2247

____ 69. Third-party payers exert their influence on:

A) Hospital operations
B) Hospital reimbursements
C) Services provided to indigent patients
D) Standards of practice
E) Hospital practices
Philip P. Gerbino. Remington: The Science and Practice of Pharmacy, 21st ed., p. 2247

____70. Fundamental functions of hospitals are education, public health, research, and:

A) Community development
B) Diagnostic procedures
C) Health maintenance
D) Training
E) Patient care
Philip P. Gerbino. Remington: The Science and Practice of Pharmacy, 21st ed., p. 2250

_____71. Requirements for registration of hospitals include all except:

A) Accreditation certificate
B) Identifiable governing body
C) With CEO
D) With food services
E) At least six in-patient beds
Philip P. Gerbino. Remington: The Science and Practice of Pharmacy, 21st ed., p. 2247

_____ 72. The following are sources of income of hospitals except:

A) Patients
B) Government
C) Hospitalization insurance
D) International accreditation agencies
E) Voluntary contributions
Philip P. Gerbino. Remington: The Science and Practice of Pharmacy, 21st ed., p. 2253

____73. The first hospital on the American continent built by the Spaniards in 1524 was:

A) Pennsylvania Hospital
B) Hospital of Jesus Nazareth
C) Bristol Royal Hospital
D) Massachusetts General Hospital
E) Johns Hopkins Hospital
Philip P. Gerbino. Remington: The Science and Practice of Pharmacy, 21st ed., p. 2249

____74. He is the Greek god of Medicine.

A) Kos
B) Hippocrates
C) Hygea
D) Fabiola
E) Aesculapius
Philip P. Gerbino. Remington: The Science and Practice of Pharmacy, 21st ed., p. 2249

____75. This act provided federal funds for hospital construction on a matching basis with local
communities.

A) Prospective Payment System


B) Hospital Standardization Program
C) Hospital Survey and Construction Act
D) Social Security Amendments of 1965
E) Generics Act
Philip P. Gerbino. Remington: The Science and Practice of Pharmacy, 21st ed., p. 2250

____76. Factors in the development and expansion of hospitals except:

A) Activities of Florence Nightingale during Crimean War


B) Civil War
C) Flexner report on medical education
D) Military influence
E) Establishment of religious orders
Philip P. Gerbino. Remington: The Science and Practice of Pharmacy, 21st ed., p. 2250

____77. Factors in the development and expansion of hospitals except

A) Activities of Florence Nightingale during Crimean War


B) Civil War
C) Flexner report on medical education
D) Military influence
E) Establishment of religious orders
Philip P. Gerbino. Remington: The Science and Practice of Pharmacy, 21st ed., p. 2249

_____78. The prime objective of this hospital function is to assist community in reducing the illness
and improving the general health of the population.

A) Patient Care
B) Research
C) Education
D) Public Health
E) None of the above
Philip P. Gerbino. Remington: The Science and Practice of Pharmacy, 21st ed., p. 2251

_____79.. Parenteral products with an osmotic pressure less than that of blood or 0.9% sodium
chloride are referred to as:
A) Isotonic solutions
B) Hypertonic solutions
C) Hypotonic solutions
D) Iso-osmotic solutions
E) Neutral solutions
Shargel, L., et al., Comprehensive Pharmacy Review, 4th ed., p. 475-487

_____80. Aseptic technique should be used in the preparation of all of the following medications with
the exception of:

A) Neomycin irrigation solution


B) Ganciclovir intraocular injection
C) Phytonadione subcutaneous
D) Ampicillin IV admixture piggyback
E) Bacitracin ointment
Shargel, L., et al., Comprehensive Pharmacy Review, 4th ed., p. 475-487

_____81. Which needle has the smallest diameter?

A) 25-gauge 3 ¼”
B) 24-gauge 3 ½”
C) 22-gauge 3”
D) 20-gauge 3 3/8”
E) 26-gauge 3 5/8”
Shargel, L., et al., Comprehensive Pharmacy Review, 4th ed., p. 475-487

_____82. Intra-articular injection refers to injection into the:

A) Muscle mass
B) Subcutaneous tissue
C) Spinal fluid
D) Superficial skin layer
E) Joint space
Shargel, L., et al., Comprehensive Pharmacy Review, 4th ed., p. 475-487

_____83. Advantages of intravenous route include:

A) Ease of removal of the dose


B) A depot effect
C) Low incidence of phlebitis
D) Rapid onset of action
E) A localized effect
Shargel, L., et al., Comprehensive Pharmacy Review, 4th ed., p. 475-487

_____84. A central vein, either subclavian or internal jugular, may be considered a suitable route for
intravenous in which of the following situations:
A) When an irritating drug is given
B) When hypertonic drugs are given
C) For long-term therapy
D) For administering dextrose 35% as parenteral nutrition
E) All of the above
Shargel, L., et al., Comprehensive Pharmacy Review, 4th ed., p. 475-487

_____85. To prepare a TPN that requires 10 mEq of Calcium Gluconate and 15 mcg of Potassium
Phosphate, the appropriate action would be to:

A) Add the calcium first, add the other additives, then add the phosphate last,
thoroughly mixing the solution after addition
B) Add the calcium gluconate and potassium phosphate consecutively
C) Not combine the agents together but give them as separate infusions
D) All are correct
E) None of the above
Shargel, L., et al., Comprehensive Pharmacy Review, 4th ed., p. 475-487

_____86. Which needle gauge would be most likely used as a subcutaneous injection of Epoetin?

A) 25 gauge 5/8 inches


B) 16 gauge 1 inch
C) 18 gauge 1 ½ inches
D) 22 gauge 1 ½ inches
E) None of the above
Shargel, L., et al., Comprehensive Pharmacy Review, 4th ed., p. 475-487

_____87. Which of the following drugs should not be prepared in a horizontal laminar flow hood?

A) Ampicillin
B) Dopamine
C) Cis-Platinum
D) Nitroglycerin
E) Bretyllium tosylate
Shargel, L., et al., Comprehensive Pharmacy Review, 4th ed., p. 475-487

_____88. All of the following statements about D5W are true, except;

A) Its pH range is 3.5-6.5


B) It is hypertonic
C) It is a 5% solution of D-glucose
D) It should be used with caution in diabetic patients
E) It is often used in intravenous admixtures
Shargel, L., et al., Comprehensive Pharmacy Review, 4th ed., p. 475-487
_____89. All of the following are potential hazards of parenteral therapy, except:

A) Hypothermia
B) Phlebitis
C) Extravasation
D) Allergic reactions
E) Ileus
Shargel, L., et al., Comprehensive Pharmacy Review, 4th ed., p. 475-487

_____90. Procedures for the safe handling of antineoplastic agents include all of the following,
except;

A) Use of Luer-Lok syringe fittings


B) Wearing double-layered latex gloves
C) Use of negative-pressure technique when medication is being withdrawn from vials
D) Wearing closed-front, surgical type gowns with cuffs
E) Use of horizontal laminar flow hood
Shargel, L., et al., Comprehensive Pharmacy Review, 4th ed., p. 475-487

_____91. In preparation of an intraspinal dose of bupivacaine, the best pore size filter for cold
sterilization would be:

A) 8mm filter
B) 5µg filter
C) 0.45µg filter
D) 0.22µg filter
E) None of the above
Shargel, L., et al., Comprehensive Pharmacy Review, 4th ed., p. 475-487

_____92. Process stimulation is:

A) A method of quality assurance


B) Evaluates the adequacy of a practitioner’s aseptic technique
C) Requires the use of a microbial growth medium
D) Is carried out in a manner identical to normal sterile admixture production
E) All of the above
Shargel, L., et al., Comprehensive Pharmacy Review, 4th ed., p. 475-487

_____93. The following definition, “one which is noxious and unintended, and which occurs at does
normally used in man for the prophylaxis, diagnosis, or therapy of disease, or for the modification of
physiological function” describes:

A) A side effect
B) An adverse drug reaction
C) An adverse drug event
D) A drug interaction
E) All of the above
Shargel, L., et al., Comprehensive Pharmacy Review, 4th ed., p. 416-421
_____94. Type A ADRs are characterized by which of the following?

A) Idiosyncratic reactions
B) A function of patient susceptibility
C) Caused by drug-drug interaction
D) All of the above
E) None of the above
Shargel, L., et al., Comprehensive Pharmacy Review, 4th ed., p. 416-421

_____95. Preventable ADRs

A) Generally display mild symptoms


B) Are always easily recognized
C) Are problems that are easily medically managed
D) All of the above
E) None of the above
Shargel, L., et al., Comprehensive Pharmacy Review, 4th ed., p. 416-421

_____96. Based on ASHP Guidelines: Minimum Standard for Pharmacies in Hospitals, immunization
programs of hospitals are under what standard?

A) Facilities and equipment


B) Medication distribution and control
C) Optimizing medication therapy
D) Research
E) Drug information and education
ASHP Guidelines: Minimum Standard for Pharmacies in Hospitals, Am J Health Syst Pharm. 1995
Dec 1;52(23):2711-7

_____97. Medical emergency services of the pharmacy department belong to:

A) Standard I
B) Standard II
C) Standard III
D) Standard IV
E) Standard V
ASHP Guidelines: Minimum Standard for Pharmacies in Hospitals, Am J Health Syst Pharm. 1995
Dec 1;52(23):2711-7

_____98. Medication therapy monitoring includes an assessment of the following, except;

A) Appropriateness of the route


B) Medication interactions
C) Therapeutic safety
D) Physical signs and cinical symptoms
E) Therapeutic appropriateness
ASHP Guidelines: Minimum Standard for Pharmacies in Hospitals, Am J Health Syst Pharm. 1995
Dec 1;52(23):2711-7
_____99. The director of pharmacy service is required to have an advanced management degree
such as the following, except;

A) M.B.A
B) M.H.A
C) M.S.
D) M.PEG.
E) None of the above
ASHP Guidelines: Minimum Standard for Pharmacies in Hospitals, Am J Health Syst Pharm. 1995
Dec 1;52(23):2711-7

_____100. The following services are under drug information and education standard, except;

A) Clinical care plans


B) Medication therapy monographs
C) Patient education
D) Dissemination of drug information
E) Drug information requests
ASHP Guidelines: Minimum Standard for Pharmacies in Hospitals, Am J Health Syst Pharm. 1995
Dec 1;52(23):2711-7

_____101. Which of the following belongs to leadership and practice management standard for
pharmacies in hospitals?

A) Disaster services
B) Education and training
C) Patient education
D) Microbial resistance
E) Medication orders
ASHP Guidelines: Minimum Standard for Pharmacies in Hospitals, Am J Health Syst Pharm. 1995
Dec 1;52(23):2711-7

_____102. This is equipment that is designed to reduce the risk of airborne contamination during the
preparation of IV admixtures by providing an ultraclean environment.

A) Sterile area
B) Clean room
C) Hypodermic needle
D) Biorefridgerator
E) Laminar flow hood
Hunt, Max., Training Manual for Intravenous Admixture Personnel, 5 th ed., p. 70

_____103. What are the two basic parts of a needle?

A) Bevel and lumen


B) Lumen and shaft
C) Hub and shaft
D) Bevel and hub
E) Hub and lumen
Hunt, Max., Training Manual for Intravenous Admixture Personnel, 5 th ed., p. 73

_____104. The part of a syringe that is a piston-type rod with slightly cone-shaped tip that passes
inside the barrel of the syringe.

A) Flange
B) Plunger
C) Rim
D) Barrel
E) None of the above
Hunt, Max., Training Manual for Intravenous Admixture Personnel, 5 th ed., p. 75

_____105. The following are examples of macronutrients that make-up most of the volume of a
parenteral solution, except;

A) IV fat emulsion
B) Dextrose solution
C) Amino acid solution
D) Vitamins and minerals
E) None of the above
Hunt, Max., Training Manual for Intravenous Admixture Personnel, 5 th ed., p. 105

_____106. Subclavian vein, to which most parenteral solutions are infused with, is located:

A) Under the collar bone


B) Beside the sternum
C) At the side of the neck
D) At the base of the heart
E) Near the liver
Hunt, Max., Training Manual for Intravenous Admixture Personnel, 5 th ed., p. 105

_____107. Hypersensitivity reactions are examples of what type of ADR?

A) Type A
B) Type B
C) Type C
D) Type D
E) Type E
Philip P. Gerbino. Remington: The Science and Practice of Pharmacy, 21st ed., p. 1222

_____108. Which is not a risk factor for ADR?

A) Age
B) Genetics
C) Duration of therapy
D) Concurrent medications
E) Social status
Philip P. Gerbino. Remington: The Science and Practice of Pharmacy, 21st ed., p. 1223

_____109. These are ADRs that are uncommon, dose and time related, and associated with the
culminative dose of the drug.

A) Type A
B) Type B
C) Type C
D) Type D
E) Type E
Philip P. Gerbino. Remington: The Science and Practice of Pharmacy, 21st ed., p. 1223

_____110. These devices are used for controlling the delivery of small volumes of intravenous
infusions over a predetermined period of time.

A) Infusion controllers
B) Infusion pumps
C) Elastomeric infusers
D) Syringe infusers
E) Heplock
Winfield AJ and Richards RME (1998). Pharmaceutical Practice, Second Edition, Churchill
Livingstone p 251

_____111. This medical staff is composed of physicians who have been active in the hospital but
who are retired and those whom it is desired to honor because of outstanding contributions.

A) Consulting
B) Honorary
C) Active
D) Courtesy
E) Associate
Philip P. Gerbino. Remington: The Science and Practice of Pharmacy, 21st ed.

_____112. This medical staff is composed of specialists who are recognized as such by right of
passing specialty boards or belonging to the national organization of their specialty and who serve
as consultants to other members of the medical staff when called upon.

A) Consulting
B) Honorary
C) Active
D) Courtesy
E) Associate
Philip P. Gerbino. Remington: The Science and Practice of Pharmacy, 21st ed.
_____113. This medical staff is most actively involved in the hospital counseling.

A) Consulting
B) Honorary
C) Active
D) Courtesy
E) Associate
Philip P. Gerbino. Remington: The Science and Practice of Pharmacy, 21st ed.

_____114. This group is composed of junior or less-experienced members of the staff.

A) Consulting
B) Honorary
C) Active
D) Courtesy
E) Associate
Philip P. Gerbino. Remington: The Science and Practice of Pharmacy, 21st ed.

_____115, This group consists of those physicians who desire the privilege of attending to private
patients, but who do not desire active staff membership.

A) Consulting
B) Honorary
C) Active
D) Courtesy
E) Associate
Philip P. Gerbino. Remington: The Science and Practice of Pharmacy, 21st ed.

_____116. This group is composed of junior or less-experienced members of the staff.

A) Consulting
B) Honorary
C) Active
D) Courtesy
E) Associate
Philip P. Gerbino. Remington: The Science and Practice of Pharmacy, 21st ed.

_____117. This group is composed of residents who are full-time employees of the hospital.

A) Consulting
B) Honorary
C) Active
D) Courtesy
E) Associate
Philip P. Gerbino. Remington: The Science and Practice of Pharmacy, 21st ed.
_____118. Small volumes of drug that are injected into the vein for a short period of time.

A) Volume control sets


B) Direct IV injection
C) Piggyback administration
D) IV stat or push
E) Continuous IV infusion
Philip P. Gerbino. Remington: The Science and Practice of Pharmacy, 21st ed.

_____119. These units consist of calibrated, plastic, fluid chambers placed in a direct line under an
established primary IV container or more often attached to an independent fluid supply.

A) Volume control sets


B) Direct IV injection
C) Piggyback administration
D) IV stat or push
E) Continuous IV infusion
Philip P. Gerbino. Remington: The Science and Practice of Pharmacy, 21st ed.

_____120. Method of intravenous administration by which solution from two containers flow into the
patient’s vein through a common tubing and a common injection site (venipuncture).

A) Volume control sets


B) Direct IV injection
C) Piggyback administration
D) IV stat or push
E) Continuous IV infusion
Philip P. Gerbino. Remington: The Science and Practice of Pharmacy, 21st ed.

_____121. It refers to the responsible of drug therapy to achieve definite outcomes that are intended
to improve a patient’s way of life.

A) Professional-patient relationship
B) Therapeutic drug monitoring
C) Drug therapy assessment
D) Pharmaceutical care
E) Formal documentation
There are several component of pharmaceutical care. The first is a belief and commitment by the
practitioner that he or she shares equal responsibility with the patient and the prescriber for optimal
drug therapy outcomes. Second is the establishment of a trusting professional-patient relationship.
The third critical component is formal documentation of not only the pharmaceutical care plan, but
also of all clinical interventions and therapeutic outcomes. Koda-Kimble 1-2

_____122. It involves measuring direct and indirect cost attributable to a specific disease.

A) Cost-of-illness evaluation
B) Cost-of-effectiveness analysis
C) Cost-minimization analysis
D) Cost-utility analysis
E) Cost-benefit analysis
A cost-of illness evaluation identifies and estimates the overall cost of a particular disease for a
defined population. By the successful identification of all cost of illness, be it direct or indirect, the
relative value of a treatment or prevention strategy can be determined. Dipiro 3

_____123. This method allows the identification, measurement, and comparison of the costs of a
program or treatment alternative.

A) Cost-of-illness evaluation
B) Cost-of-effectiveness analysis
C) Cost-minimization analysis
D) Cost-utility analysis
E) Cost-benefit analysis
Both costs and benefits are expressed in equivalent monetary value in the year in which they will
occur, these costs and benefits are expressed as a ratio, a net cost. This is an appropriate method to
use in documenting the value of an existing program or the potential worth of a new one.

_____124. The method that involves the determination of the least costly alternatives when
comparing two or more treatment alternatives.

A) Cost-of-illness evaluation
B) Cost-of-effectiveness analysis
C) Cost-minimization analysis
D) Cost-utility analysis
E) Cost-benefit analysis
With cost minimization Analysis, the alternatives must have an assumed or demonstrated
equivalency in safety and efficacy. This method only shows a “cost savings” of one program of
treatment over another. Dipiro 3

_____125. The method that integrates the patient preferences and health-related quality of life.

A) Cost-of-illness evaluation
B) Cost-of-effectiveness analysis
C) Cost-minimization analysis
D) Cost-utility analysis
E) Cost-benefit analysis
In Cost-Utility analysis, the cost is measured in monetary units, and therapeutic outcome is
measured in patient-weighted utilities. This method is the most appropriate in comparing programs
and alternatives that are life extending with serious side effects, those which produce reductions in
morbidity rather that mortality. And when quality of life is the most important outcome being
considered. Dipiro 4

_____126. A way of summarizing the health benefits and resources used by competing health care
programs so that policy makers can choose from them.
A) Cost-of-illness evaluation
B) Cost-of-effectiveness analysis
C) Cost-minimization analysis
D) Cost-utility analysis
E) Cost-benefit analysis
Cost-effectiveness analysis involves the comparison of programs or alternatives with different safety
and efficacy profiles. Cost is measured in monetary units, and outcomes are measured in obtaining a
specific therapeutic outcome. Example of outcomes are lives saved, cases cured, or life expectancy.
Dipiro 3

_____127. It refers to the value assigned to the duration of life as modified by impairments,
functional states, perceptions, and social opportunities that are influenced by disease, injury,
treatment or policy.

A) Optimum health
B) Quality of life
C) Health outcomes
D) Responsiveness
E) Wellness of being
The commonly measured dimensions of quality of life include: physical health and functioning:
psychological health and functioning; social and role functioning; and, perceptions of general
well-being. Health outcomes are the end results associated with medical care delivery systems and
specific medical interventions. Responsiveness is the ability or power of the measure to detect
clinically important change when it occurs. Dipiro 11, 15

_____128. A study design in which neither the study subject nor the study staff is aware of which
group or intervention the subjects has been assigned.

A) Randomized control trial


B) Crossover study
C) Cohort study
D) Blinded study
E) Cross-sectional study
When both the subject and study staff are not aware of the assignments, it is a double-blinded study.
When only the subject is not aware of his or her assignment, it is single-blinded study. The blinding
minimizes the bias. Dipiro 30

_____129. A retrospective comparison of causal factors or exposure in a group of persons with


disease and those of persons without disease.

A) Randomized control trial


B) Crossover study
C) Cohort study
D) Meta analysis
E) Open label trial
Persons with the disease are referred to as “cases”, and those without the disease are the “controls”.
The purpose of a case-control study is to find the clinical finding that occurs more frequently in the
cases than the control. Dipiro 30
_____130. A retrospective or prospective follow-up study of exposed and non-exposed defined
group in which a variable of interest is measured.

A) Cross-sectional study
B) Case control study
C) Crossover study
D) Cohort study
E) Open-label trial
In Cohort study, the exposure is measured before the development of the disease. The incidence,
risk and the relative risk are measured. Dipiro 30

_____131. A trial comparing treatments in which participants, on completion of one treatment, are
switched to the other.

A) Cross sectional study


B) Cross over study
C) Meta analysis
D) Open label trial
E) Cohort study
In a crossover study, the drug therapies are administered in either a specified or random order to
each participant. Dipiro 30

_____132. A study that determines the presence or absence of a disease and other variable in a
defined population and the potential risk factor at a particular point in time or time-interval.

A) Cross sectional study


B) Cohort study
C) Case control study
D) Cross over study
E) Case series
In cross-sectional studies, exposure and outcome are determined simultaneously. The temporal
sequence of cause and effect cannot be necessarily determined. Dipiro 30

_____133. Which of the following describes the neonates?

A) Those who are 1 month-1 year of age


B) Those who are 1 year-12years of age
C) Those who are 12-16 years of age
D) Those between 1 day and 1 month of age
E) Those born before 37 weeks of gestation
Those between one month to one year old are referred to as infants; one to twelve years of age,
children; and 12 to 16 years, are adolescents. Dipiro 69

_____134. Which of the following best describes the infants?

A) Those who are 1 month-1 year of age


B) Those who are 1 year-12years of age
C) Those who are 12-16 years of age
D) Those between 1 day and 1 month of age
E) Those born before 37 weeks of gestation
Newborn before 37 weeks of gestational age are termed premature. Dipiro 69

_____135. Clinical manifestations of Gray Baby Syndrome include:

I. Characteristic gray color


II. Abdominal distention
III. Hypertension
IV. Progressive shock

A) I only
B) I and IV only
C) I, II, II only
D) I, II. IV only
E) I and III only
Gray Baby Syndrome was the first reported in two neonates who died after excessive
chloramphenicol doses. Gray Baby Syndrome patients usually have abdominal distention, vomiting,
diarrhea, a characteristic gray color, respiratory distress, hypotension, and progressive shock. Dipiro
69

_____136. Which of the following is associated with the use of thalidomide?

I. Polyneuritis
II. Mental retardation
III. Limb deformities

A) I only
B) I, II and II only
C) II and III only
D) II only
E) I and II only
Thalidomide gained much attention due to its teratogenic effects. It has been implicated as the cause
of multiple congenital fetal abnormalities, polyneuriris, nerve damage and mental retardation.

_____137. Factors that increase the risk of drug-related problems in the elderly include:

I. Polypharmacy
II. Inappropriate prescribing
III. Medication adherence
IV. Multiple diseases

A) I and II only
B) I, II, and III only
C) II and IV only
D) I, II, and IV only
E) I, II, III, and IV
In the elderly, the factors that increase the risk of drug-related problems include suboptinal
prescribing (e.g. overuse of medications or polypharmacy, inappropriate use, and underuse),
medication errors (both dispensing and administration problems), and patient medication
nonadherence (both intentional and unintentional). Dipiro 84

_____138. Potential difficulties that may occur while taking medication histories form the elderly
include:

I. Impaired hearing
II. Mental acuity
III. Multiple diseases and medication
IV. Reliance on a caregiver for the history
A) I, II, and III only
B) I. III, IV only
C) II, III, IV only
D) I, II, IV only
E) I, II, III, IV
In history taking procedures, the potential difficulties that may occur with the elderly patients include:
communication problems due to impaired hearing and vision; underreporting due to health beliefs
and cognitive impairment; vague or nonspecific symptoms in the form of altered presentations;
multiple diseases and medications; reliance on a caregiver for history information; and lack of
medical records to confirm findings. Dipiro 89

_____139. Refers to the continual monitoring for unwanted effects and other safety-related aspects
of marketed drugs.

A) Pharmacoepidemiology
B) Pharmacovigilance
C) Pharmacoinformatics
D) Pharmacogenetics
E) Pharmacogenomics
Pharmacovigilance, definition. Dipiro 91

_____140. Study the use of and effects of drugs in a large number of people.

A) Pharmacoepidemiology
B) Pharmacovigilance
C) Pharmacoinformatics
D) Pharmacogenetics
E) Pharmacogenomics
Pharmacoepidemiology, definition. Dipiro 91

_____141. Any noxious, unintended, and undesired effect of a drug that occurs at doses used in
humans for prophylaxis, diagnosis or therapy.

A) Adverse drug event


B) Allergy
C) Hypersensitivity
D) Adverse drug reaction
E) Idiosyncracy
Adverse drug reaction, definition. The term adverse drug event is used to describe an injury resulting
from administration of a drug. Dipiro 94

_____142. Which of the following is/are true regarding chest X-ray?

I. Provides supplemental information to the physical examination and is usually


the first diagnostic test in a cardiac workup
II. Provides details of internal cardiac structures
III. Gives information about position and size of the heart and chambers and
surrounding anatomy.

A) I only
B) I, II, III only
C) I and II only
D) I and III only
E) II and III only
Chest X-ray does not provide details of internal cardiac structure but give global information about
position and size of the heart and chambers of the surrounding anatomy. Initial assessment of the
chest x-ray evaluates the quality of the film for patient rotation, inspiratory effect, and penetration.
Dipiro 94

_____143.The manifestations of chromium deficiency include:

I. Alopecia
II. Depigmentation of the hair and skin
III. Red blood cell fragility
IV. Glucose intolerance

A) I and II only
B) II and IV only
C) I, II, III, and IV
D) III only
E) IV only
Deficiency in chromium results to glucose intolerance, peripheral neuropathy, and metabolic
encephalopathy. Alopecia is a zinc deficiency state, and depigmentation of skin and hair, a copper
deficiency state. Red blood cell fragility results from selenium inadequacy. Walker 69

_____144. The manifestations of zinc deficiency include:

I. Poor wound healing


II. Poor growth
III. Poor resistance to infection

A) I only
B) II and III only
C) I and IV only
D) III only
E) I and IV only
Zinc deficiency may also cause alopecia and infertility. The daily intravenous requirement for zinc is
50-100 umol. Walker 69

_____145. Incompatibilities with fat emulsions cause majority of formulation problems in TPN. Which
of the following affects lipid stability in TPN preparations?

I. Nature of amino acid solution


II. pH
III. Amount of dissolved oxygen in the solution
IV. Electrolyte content

A) I, II, and III only


B) II, III and IV only
C) I, II and IV only
D) I and II only
E) I and IV only
An increased PH results in increased stability of fat emulsions. At the same time, lipid emulsions are
destabilized by the presence of divalent and trivalent cations. Walker 71

_____146. Ascorbic acid is the most rapidly oxidized vitamin. The rate of oxidation depends on:

I. Electrolyte content
II. Presence of trace minerals
III. Amount of dissolved oxygen in the solution
IV. pH

A) IV only
B) II and III only
C) I, II, III only
D) II, III, IV only
E) II and IV only
Ascorbic acid oxidizes to dehyroascorbic acid, which is metabolically active but rapidly degrades to
2, 3- diketoglonic acid and then to oxalic acid. The oxalic acid may cause the formation of oxalate
stones. Walker 72

_____147. Which of the following is the most rapidly reduced vitamin?

A) Riboflavin
B) Niacin
C) Thiamine
D) Cevitamic acid
E) Ascorbic acid
Thiamine is Vitamin B1. Its degradation rate can be increased by sodium metabisulfite and amino
acids. Walker 72

_____148. Which of the following are associated with dilutional hyponatremia?


I. Administration of albumin
II. Congestive heart failure
III. Cirrhosis

A) II only
B) II and III only
C) I and II only
D) I only
E) I, II, and III only
Dilutional hyponatremia occurs when the extracellular fluid compartment expands without an
equivalent increase in sodium. Circhosis, congestive heart failure, nephrosis and administration of
osmotically active solutes, like albumin and mannitol, are most commonly associated with its
occurrence. Koda-Kimble 2-3

_____149. Which of the following is/are true regarding blood urea nitrogen (BUN)?

I. End product of protein metabolism


II. Produced by liver and kidney
III. Filtered completely at glomerulus

A) I and II only
B) I and III only
C) II and III only
D) I, II and III
E) I only
Urea nitrogen is solely produced by the liver, is transported in the blood, and is excreted by the
kidneys. The concentration of blood urea nitrogen in blood is filtered completely at the glomerulus of
the kidney, then reabsorbed and tubularly secreted with nephrons. Koda-Kimble 2-10

_____150. Which of the following accurately describes creatinine?

I. Not absorbed and secreted by the kidney


II. Filtered freely at the glomerulus
III. Metabolized in the kidney

A) I only
B) II only
C) III only
D) I and II only
E) II and III only
Creatinine is derived from the creatine and phosphocreatine, a major muscle constituent. Creatine is
filtered freely at the glomerulus and is not absorbed, secreted, aynthesized, or metabolized by the
kidney, making its clearance a good reflection of the glomerular filtration rate. Koda-Kimble 2-10

_____151. Considered drug of choice in the treatment of angina at rest.

A) Nitroglycerin
B) Calcium-channel blockers
C) ACE inhibitors
D) Beta blockers
E) Clopidogrel
Calcium channel blockers prevent and reverse coronary spasm by inhibition of calcium influx into the
vascular smooth muscle and myocardial muscle. Such results in the increase in blood flow, which
enhances myocardial oxygen supply. Shargel 722

_____152. Used for the same arrhythmiasfro which quinidine is give, but possess added concern
when used intravenously because of increased cardiovascular effects such as hypotension, syncope
and myocardial infarction.

A) Procainamide
B) Disopyramide
C) Tocainamide
D) Mexiletine
E) Flecainide
Procainamide is used more frequently than quinidine because it can be administered intravenously
and in sustained-release oral preparations. Disopyramide may be used as an alternative to quinidine
and procainamide for treating ventricular arrhythmias. Procainamide, quinidine and disopyramide are
all Class IA anti-arrhythmics. Shargel 746

_____153. Most commonly used to treat digitalis-induced ventricular and supraventricular


arrhythmias.

A) Propafenone
B) Flecainide
C) Phenytoin
D) Quinidine
E) Propranolol
Phenytoin can also be given to suppress ventricular arrhythmias associated with acute myocardial
infarction, open-heart surgery or ventricular arrhythmias that are refractory to lidocaine or
procainamide. Shargel 747

_____154. Class I arrhythmias work by blocking the rapid inward sodium content and thereby slowly
down the rate of rise of the cardiac tissue’s action potential. Which sub classification moderately
reduces the depolarization rate and prolong repolarization?

A) Class IA
B) Class IB
C) Class IC
D) Class ID
E) Class IA and Class IB
Class IB drugs are capable of shortenig repolarization or the refractory period, as they weakly affect
the repolarization rate. Class IC drugs are strongly depress depolarization but have a negligible
effect on the duration of repolarization. Shargel 748
_____155. Which of the following is/are not associated with cinchonism caused by Quinidine?

I. Hearing loss
II. Photophobia
III. Tinnitus
IV. Blurred vision

A) I and II only
B) III and IV only
C) I, II, III only
D) I, III, IV only
E) II, III, IV only
Cinchonism may also be manifested by gastrointestinal disturbance. In severe cases, nausea,
vomiting, diarrhea, headache, confusion, delirium, photopobia, diplopia and psychosismay also
occur. Shargel 749

_____156. Among the type III arrhythmics listed below, which has been reported as causing
Torsades de Pointes type of ventricular tachycardia?

A) Sotalol
B) Flecainide
C) Amiodarone
D) Quinidine
E) Lidocaine
Torsades de Pointes is defined as polymorphic ventricular tachycardia with a twisting QRS complex
morphology, which sometimes occurs with drugs that prolong ventricular repolarization. Initial reports
of its occurrence centered around the use of quinidine, a Class IA anti-arrhythmic drug. Today, more
than 50 drugs have been shown to affect the duration of QT interval. Shargel 739

_____157. A Class I anti-arrhythmic drug that can cause fatigue, arthralgia, myalgia, and low-grade
fever, suggestive of systemic lupus erythematosus-like syndrome.

A) Flecainide
B) Disopyramide
C) Mexiletine
D) Procainamide
E) Quinolone
The most severe side effects of procainamide are the hypersensitivity reactions. These reactions
include drug fever, agranulocytosis and SLE-like syndrome. Shargel 750

_____158. A Class IV anti-arrhythmic that is primarily indicated for the treatment of supraventricular
tachyarrhythmias.

A) Verapamil
B) Disopyramide
C) Propanolol
D) Sotalol
E) Ibutilide
Class IV anti-arrhythmics are calcium channel blockers. They inhibit AV node conduction by
depressing SV and AV nodes, where calcium channel predominate. However, these drugs must be
used cautiously in patients with CHF, sick-sinus syndrome, myocardial infarction and hepatic or renal
impairment . Shargel 756

_____159. The New York Heart Association functional limitation class indicates that the degree of
effort necessary to elicit Heart Failure symptoms is equal to those that would limit normal individuals.

A) Class I
B) Class II
C) Class III
D) Class IV
E) Class V
Class Degree I of effort to elicit HF symptoms equals those that would limit normal individuals. Class
II Degree of effort necessary to elicit HF symptoms occurs with less than ordinary exertion. Class II
Degree of effort necessary to elicit HF symptoms occurs with less than ordinary exertion. Class IV
Degree of effort necessary to elicit HF symptoms occurs while at rest. Shargel 785-786

_____160.The New York Heart Association functional limitation class indicates that the degree of
effort necessary to elicit Heart Failure symptoms occurs with less-than ordinary exertion.

A) Class I
B) Class II
C) Class III
D) Class IV
E) Class V
Refer to table in #159

_____161. The New York Heart Association functional limitation class indicates that the degree of
effort necessary to elicit Heart Failure symptoms occurs while at rest.

A) Class I
B) Class II
C) Class III
D) Class IV
E) Class V
Refer to table in #159

_____162. Term referred for that exerted on the ventricular muscle at he end of the diastole that
determines the degree of muscle fiber stretch.

A) Frank-Starling mechanism
B) Afterload
C) Cardiac output
D) Preload
E) Decompensation
Preload is known as ventricular and diastolic pressure. It is a key factor in contractility because the
more these muscles are stretched diastole, the more powerfully they contract in systole.
Frank-Sterling mechanism is a concept that works with the premise that an increased fiber dilation
heightens the contractile force, which then increases the energy released.

_____163. Determined by the amount of force needed to overcome pressure in the aorta.

A) Frank-Starling mechanism
B) Afterload
C) Cardiac output
D) Preload
E) Decompensation
Afterload, also known as the intraventricular systolic pressure, refers to the tension inventricular
muscles during contraction. It is sometimes use to describe the amount of force needed in the right
ventricle to overcome the pressure in the pulmonary artery. Shargel 790

_____164. Which of the following is/are true regarding precautions with the use of digoxin

I. Increased potassium levels favor digoxin binding to cardiac cells and


increase its effect
II. Increased calcium levels increase the force of myocardial contraction
III. Magnesium levels are inversely related to digoxin activity

A) I only
B) II only
C) I and II only
D) I and III only
E) II and III only
Potassium preparations seem to antagonize digitalis preparation. This antagonism is particularly
significant in patients taking diuretics, many of which decrease potassium levels. Shargel 793

_____165. Which of the following dosages of dopamine is selected for its positive inotropic effects in
treating the patient with Heart Failure?

A) 40 mg/kg/min
B) 40 ug/kg/min
C) 10-20 ug/kg/min
D) 5-10 ug/kg/min
E) 2 ug/kg/min
Dopamine exhibits a wide range of effects. At doses of 2-5 ug/kg/min, it increases renal blood flow
through its dopaminergic effects. At doses of 5-10 ug/kg/min, it increases cardiac output through its
beta-adrenergic stimulating effect. At doses of 10-20 ug/kg/min, it increases peripheral vascular
resistance through its alpha-adrenergic stimulating effect. Shargel 804

_____166. A loop diuretic that is useful as a rapid-acting intravenous agent in reversing acute
pulmonary edema.

A) Ethacrynic caid
B) Furosemide
C) Spironolactone
D) Bumetanide
E) Mannitol
The ability of furosemide to reverse acute pulmonary edema is due to its direct dilating effects on
pulmonary vasculature. Shargel 795

_____167. Diuretics which hav become preferred due to their ability to increase sodium excretion by
20-25% of the filtered load and maintain their efficacy until renal function is severely impaired:

I. Spironolactone
II. Ethacrynic acid
III. Furosemide
IV. Bumetanide

A) I and II only
B) II and III only
C) I, II, III only
D) II, III, IV only
E) I, III, IV only
Loop diuretics, which include furosemide, ethacrynic acid and bumetanide, are capable of exerting
such actions. They also share the advantage of reducing venous return independent of diuresis.
Shargel 795

_____168. Which of the following may be used in the acute management of digitalis toxicity?

I. Potassium
II. Cholestyramine resin
III. Fab fragment antibody

A) I only
B) II only
C) I and II only
D) I and III only
E) I, III and IV only
In digitalis toxicity, if the patient is hypokalemic, potassium supplements are administered and serum
levels are monitored. Cholestyramine may also be given because of its capability to bind to digitalis
glycosides. It may help prevent absorption and reabsorption of digitalis in the bile. In patients with
very high serum digoxin levels, purified digoxin-specific Fab fragment is given. Shargel 794

_____169. Which of the following best describes isoniazid?

A) May be nephrotoxic and ototoxic


B) Has limited use due to its hepatotoxic potential
C) Requires pyridoxine supplementation
D) May discolor the tears and urine orange red
E) A pyrazine analogue of nicotinamide
Isoniazid is a hydroxide of isonicotinic acid. It is the most widely used antitubercular agent and
should be used in combination with another antitubercular drug to prevent drug resistance in
tuberculosis. Shargel 860
_____170. Which of the following may be treated with streptomycin?

I. Bacterial endocarditis
II. Tularemia
III. Urinary tract infection
IV. Tuberculosis

A) I, II and III only


B) II, III and IV only
C) I and II only
D) II and IV only
E) III and IV only
Streptomycin may be used to treat plague, tularemia, acute brucellosis (in combination of
tetracycline), bacterial endocarditis caused by Streptococcus viridans (in combination with penicillin),
tuberculosis (in combination with other abtitubercular agents). Shargel 830

_____171. Which of the following best illustrates the relative ototoxicity of aminoglycosides?

A) Streptomycin=kanamycin>amikacin+gentamicin
B) Kanamycin>streptomycin=tobramycin>amikacin
C) Streptomycin=gentamicin>kanamycin=amikacin
D) Kanamycin>streptomycin>kanamycin
E) Streptomycin>kanamycin>amikacin
In order to minimize or prevent serious adverse effets of aminoglycosides, blood drug
concentrations. Blood urea nitrogen and serum creatinine levels should be monitored during therapy.
Shargel 830

_____172. Which of the following amonoglycosides can cause primarily vestibular damage?

A) Neomycin and amikacin


B) Gentamicin and streptomycin
C) Kanamycin and streptomycin
D) Amikacin and kanamycin
E) Streptomycin and amikacin
Vestibular damage caused by gentamicin and streptomycin may be manifested by tinnitus, vertigo
and ataxia. Such damage may be bilateral and irreversible. Shargel 830

_____173. Which of the following aminoglycosides can cause mainly auditory damage?

A) Gentamicin and streptomycin


B) Streptomycin and neomycin
C) Amikacin and kanamycin
D) Kanamycin and streptomycin
E) Amikacin and gentamicin
Auditory damage is manifested by hearing loss. Only tobramycin can result in both vestibular and
auditory damage. Shargel 830

_____174. Which is the least nephrotoxic aminoglycoside?


A) Streptomycin
B) Tobramycin
C) Neomycin
D) Amikacin
E) Netilmicin
Mild renal dysfunction develop in up to 25% of patients receiving aminoglycosides for several days
or more. This is due to the accumulation of these drugs in the proximal tubule. Streptomycin remains
to be the least nephrotoxic aminoglycosides. Shargel 830

_____175. Which is the most nephrotoxic aminoglycoside?

A) Streptomycin
B) Tobramycin
C) Neomycin
D) Amikacin
E) Netilmicin
Neomycin is the most nephrotoxic aminoglycosides. Risk factors for increased nephrotoxic effects
include: preexisting renal disease, previous of prolonged aminoglycosides therapy, concurrent
administration of other nephrotoxic and impaired renal flow unrelated to renal disease. Shargel 830

_____176. Which carbapenem need to be combined with cilastatin sodium to inhibit the action of
renal dipeptidases to this drug?

A) Aztreonam
B) Meropenem
C) Carbamazepine
D) Ertapenem
E) Imipenem
Carbapenems are beta-lactams that contain a fused beta-lactam ring and a 5-membered ring
system that differs from penicillin in being unsaturated and containing a carbon instead of a sulfur
atom. Cilastatin is not required with meropenem and ertapenem since these are not sensitive to
renal dipeptidase. Shargel 831

_______ 177. Which of the following is/are the routes of administration of Penecillin G sodium and
potassium?

I. Intradermal
II. Intravenous
III. Intramuscular
IV. Oral

A) II, III, and IV only


B) I, III and IV only
C) II and III only
D) IV only
E) II and IV only
Penicillin G sodium and potassium salts can be administered orally, intravenously or intramuscularly.
Penicillin V is administered orally. Penicillin G procaine and Penicillin G benzathine are repository
drug forms that can administered intramuscularly. Shargel 834
______178. Anaphylaxis is a life threatening reaction that most commonly occurs with parenteral
administration of penicillin. The following are its signs and symptoms:

I. Nausea
II. Bronchoconstriction
III. Abdominal pain
IV. Severe hypertension

A) I only
B) I and II only
C) I, II, III only
D) I, II, IV only
E) II only
Signs and symptoms of anaphylaxis due to penicillin include severe hypotension,
bronchoconstriction, nausea, vomiting, abdominal pain and extreme weakness. Shargel 835

______179. With which antibiotics does antibiotic antagonism occur when given 1 hour of penicillin
administration?

A) Tetracycline and cephalosphorins


B) Aminoglycosides and macrolides
C) Cabapenems and vancomycin
D) Erhythromycin and chloramphenicol
E) Carbenicillin and isoniazid
Antibiotic antagonism with penicillins occurs when erythromycin, tetracyclines or chloramphenicol is
given within one hour of the penicillin administration. The clinical significance of such is not yet
established. Shargel 835

_______180. An extended spectrum penicillin that is 10 times as active as carbenicillin against


Pseudomonas organisms.

A) Carbenicillin
B) Piperacillin
C) Ticarcillin
D) Meszlocillin
E) Tazobactam
Piperacillin is an extended-spectrum penicillin. Together with mezlocillin, they are referred to as the
ureidopenicillins. Piperacillin is also more active than carbenicillin against streptococcal organisms.
Shargel 836

______181. Include tumors of connective tissues such as bone or muscle.

A) Carcinomas
B) Sarcomas
C) Lymphomas
D) Leukemia
E) Hematoma
Sarcomas are solid tumors. They include tumors of bone like osteosarcoma, and tumors of muscles
like leiomyosarcomas. Shargel 1097

_____182. Include tumors of blood-forming elements and are classified as acute or chronic, myeloid
or lymphoid.

A) Carcinomas
B) Sarcomas
C) Lymphomas
D) Leukemia
E) Hematoma
Leukemias and lymphomas are hematological malignancies. Lymphomas are tumors of the
lymphatic system and include Hodgkin’s an non-Hodgkin’s lymphomas. Shargel 1097

_____183. The stage in cardiogenesis wherein the environment becomes altered to allow
preferential growth of mutated cells over normal cells.

A) Promotion
B) Initiation
C) Progression
D) Regression
E) Metastasis
Carcinogenesis or the mechanism of hoe cancer occur, is thought to be a multi-stage multifunctional
process that involves both genetic and environmental factors. It has three stages: initiation,
promotion and progression. Shargel 1096

_____184. The first stage of carcinogenesis wherein normal cells are exposed to a carcinogen.

A) Promotion
B) Initiation
C) Progression
D) Regression
E) Metastasis
Initiation, definition. Shargel 1096

_____ 185. A metabolite of cyclophosphamide and ifosmamide that is thought to cause a chemical
reaction of bladder mucosa leading to bleeding.

A) Mesna
B) Amifostine
C) Acrolein
D) Dexrazoxane
E) Histamine
Acrolein is thought to cause hemorrhagic cystitis and is usually seen after administration of
cyclophosphamide and ifosfamide. Preventive measures include aggressive hydration and frequent
urination and the administration of uroprotectant mesna. Shargel 1110

_____ 186. Which of the following is/are true regarding pain?


I. An unpleasant sensory and emotional experience that is usually associated
with structural or tissue damage
II. A subjective, individual experience
III. Has physical, psychological, and social determinants

A) I and II only
B) II and III only
C) I, II, III only
D) III only
E) I only
Pain, definition and description. Shargel 1116

____ 187. Stimulation of mu receptors produces the characteristic necrotic effects. These
morphine-like effects include:

I. Analgesia
II. Euphoria
III. Sedation
IV. Physical dependence

A) I, II, III, IV
B) I, II III only
C) I, II, IV only
D) I III only
Aside from those listed, the other characteristics narcotic effects that are elicited upon the stimulation
of the Mu receptors include miosis, respiratory depression and bradycardia. Shargel 1120

_____ 188. A component of red paper that causes the release of substance P from sensory nerve
fibers, resulting in the prolonged cutaneous pain transmission, histamine release, and erythema
because of reflex vasodilation.

A) Glucosamine sulfade
B) Chondroitin
C) Capsaicin
D) Capsicum oleoresin
E) Cyclooxygenase

_____189. All of the following provide acid suppression similar to omeprazole 20mg every day
except:

A) Lansoprazole 30 mg every day


B) Pantoprazole 40 mg every day
C) Rabeprazole 20 mg every day
D) Famotidine 20 mg every day
E) All provide equivalent acid suppression
Doses of omeprazole 20mg, lansoprazole 30mg, pantoprazole 40mg and rabeprazole 20mg
administered once daily provide similar levels of acid suppression. All provide significantly better
acid inhibition than famotidine, even at doses of 20mg twice a day or more. Shargel et.al,
Comprehensive Pharmacy Review 6th Ed. p1124
_____ 190. Natural or synthetic polysaccharide derivatives that adsorb water to soften the stool and
increase the bulk.

A) Saline laxatives
B) Lubricant laxatives
C) Emollient laxatives
D) Bulk-forming laxatives
E) Stimulant laxatives
Bulk-forming laxatives work in both the small and large intestines. Their onset of action is slow.
Thus, they are used to prevent constipation rather than to treat severe acute constipation. Shargel
607

______ 191. Laxatives that act as surfactants by allowing absorption of water into the stool, which
makes the softened stool easier to pass.

A) Saline laxatives
B) Lubricant laxatives
C) Emollient laxatives
D) Bulk-forming laxatives
E) Stimulant laxatives
The increased volume results in distention of the intestinal lumen, causing increased peristalsis and
bowel motility. Shargel 608

____ 192. Clusters of dilated blood vessels n the lower rectum or anus.

A) Anal abcess
B) Cryptitis
C) Anal fissure
D) Polyps
E) Hemorrhoids
Hemorrhoids, definition. Shargel 616

_____ 193. Which of the following statements adequately describe bulk-forming laxatives?

A) Can cause diarrhea if not taken with water


B) Onset of action is 4-8 hours
C) Provide much more complete evacuation of constipation than stimulant products
D) Are derived from polysaccharides and resemble fiber in mechanism of action
E) All of its kind can cause abdominal cramping
Bulk-forming laxatives are natural or synthetic polysaccharide derivatives that adsorb water to soften
stool and increase bulk. Shargel 607-608

____ 194. Whjch of the following is/are risk factor/s for hyperphoshatemia and death from sodium
phosphate enema?

I. Renal insufficiency
II. Hirschsprung’s disease
III. Anorectal malformations

A) I and II only
B) II and III only
C) I, II, III only
D) III only
E) I only
Chronic renal disease, anorectal malformations and/or Hirschsprung’s disease allow phosphate
blood concentrations to become abnormally high and potassium and calcium to become low. These
conditions predispose patients to cardiac arrhythmias and potentially death. Shargel 627

_____ 195. Which of the following is true regarding lactulose?

I. An osmotic laxative
II. May cause flatulence and cramping
III. Should be taken with milk or juice to increase its palatability
A) I and II only
B) II and III only
C) I, II, III only
D) III only
E) II only
Lactulose is available only by prescription. It is used to decrease blood ammonia levels in hepatic
encephalopathy. Shargel 609

____ 196. A condition characterized in a poorly functioning colon and resembles the symptoms of
ulcerative colitis, caused by chronic use of stimulant laxatives.

A) Melanosis coli
B) Cathartic colon
C) Lipid pneumonitis
D) Laxative abuse
E) Hyperphosphatemia
Cathartic colon, definition and description. Shargel 609

____ 197. A dark pigmentation of the colonic mucosa that results from long-term used of
anthraquinone laxatives.

A) Melanosis coli
B) Cathartic colon
C) Lipid pneumonitis
D) Laxative abuse
E) Hyperphosphatemia
Melanosis coli, definition and description. Shargel 609

______ 198. Adverse effects of Metoclopramide limit its usefulness. These adverse effects include:

I. Insomnia
II. Depression
III. Extra pyramidal effects

A) I and II only
B) II and III only
C) II only
D) I, II III only
E) III only
Adverse effects of metoclopramide include sedation, depression and extrapyramidal effects. Shargel
622

_____ 199. Which of the following statements is/are true regarding emollient laxatives?

I. Not good for acute constipation


II. More effective than placebo for long term use
III. Good for patients who should not strain by passing hard stool

A) I, II, III only


B) I and III only
C) I and II only
D) II and III only
E) I only
Clinical trials evaluating emollient laxatives show that these products, when compared to placebo, do
not affect the weight or water content of the stool or frequency of stool passing. Shargel 609

______ 200. Potential adverse effects of Aspirin include:

I. Gastrointestinal ulceration
II. Hypersensitivity asthma
III. Cardiac arrhythmias

A) I and II only
B) II only
C) III only
D) I and III only
E) I, II, III only

____ 201. If the hospital pharmacy is decentralized, each satellite pharmacy must be:

A) Supervised by a single pharmacist who is knowledgeable and experienced in


hospital operations
B) Supervised by a pharmacy technician or aide, in order to maximize human resources
in the hospital
C) Supervised by a licensed pharmacist who reports to the director of pharmacy
D) Supervised by pharmacy interns in order to facilitate full training experience
E) Supervised by a registered nurse if there is no pharmacist available
It is not always required that a pharmacist be a full-time employee. However, it is recommended that
a pharmacist be available at all times. The service extended by the pharmacist must meet the needs
of the patients in the hospital as determined by the medical staff. Brown 32
_____ 202. Routine inspections must be done in all areas where medications are dispensed,
administered and stored. Documented checks are to be made to make sure that:

I. Outdated and other unusable drugs are removed from locations where they
could be administered in error
II. The use of investigational drugs conforms poorly with pharmacy policies and
procedures
III. External and internal drug preparations must be stored in the same
pharmacy area

A) I, II III only
B) I and II only
C) II only
D) I only
E) I and III only
Documented checks must also ensure that drugs are stored under proper conditions, and that
emergency drug supplies are in order and adequate. The person assigned to do the routine
inspections must be qualified by previous training. Brown 33

_____ 203.Responsible for the development and surveillance of pharmacy and therapeutics policies
and practices, particularly drug utilization within the hospital.

A) Medical director
B) Board of directors
C) Pharmacy and therapeutics committee
D) Chief pharmacist
E) Pharmaceutical services director
The pharmacy and Therapeutics Committee is responsible for advising on the use of all the drugs in
the hospital. It should also consider problems such as drug interactions, medication errors, and the
misuse of drugs Brown 35

____204. Which of the following are the recommended functions of the Pharmacy and Therapeutics
Committee?

I. Advises the medical staff and pharmacy service on matters pertaining to the
choice of drugs available for patient and diagnostic testing
II. Adds and deletes drugs in the hospital formulary or drug list and continually
reviews it
III. Ensures constant supply of drugs by facilitating regular inventory and stocks
review

A) I, II, III only


B) I, II only
C) I, III only
D) II and III only
E) I only
The pharmacy and Therapeutics Committee is also mandated to review all drug reactions occurring
in the hospital, and to evaluate and approve the protocols for the use of investigational and
experimental drugs Brown 35

_____ 205. By custom, acts as the Secretary of the Pharmacy and Therapeutics Committee.

A) Registered nurse
B) Head of the legal division of the hospital
C) Medical director
D) Pharmacist
E) Administrative clerk
By custom, it is the pharmacist who acts as the secretary of the Pharmacy and
Therapeutics Committee. However, this is not always a good idea since it is difficult to
take the minutes and at the same time participate actively in the meeting. Brown 35

____ 206. When medications are dispensed to patients at the time of their discharge, the labels
must not include:

I. Full name of the patient


II. Name of the drug, strength, and amount dispersed
III. Unit cost of the drug
IV. Name of prescribing practitioner

A) I, II, III only


B) I, II, III, IV
C) II only
D) I and II only
E) I, II and IV only
Other pertinent information that must be written on the label of medications include: name, address,
and phone number of the hospital pharmacy; date and pharmacy’s prescription number; directions
for use; identification of individual dispensing; and required or any other important cautionary labels.
Brown 39

_____ 207. Refers to a defined course or method to guide and determine present and future
decisions.

A) Framework
B) Policy
C) Operations
D) Procedure
E) Strategic planning
A policy is a broad, general plan that provides a framework for action. A policy statement addresses
specifically what must be done. Brown 45

_____ 208. Refers to a series of steps followed in a regular definite sequence in order to
accomplish something.
A) Framework
B) Policy
C) Operations
D) Procedure
E) Vision-mission statement
A procedure addresses the question of how the thing that must be done. It provides an explanation
of the means or method, by which a policy is carried out and, in so doing, starts at the beginning of
the task and, in step-by-step process, outlines that task through a complete cycle. Brown 45

______209. Which of the following are included in the administrative information division of most
policy and procedure manuals?

I. Personnel policies and procedure


II. Philosophy
III. Organizational relationships
IV. Job descriptions
A) I only
B) II only
C) I, II, III only
D) II and IV only
E) II, III, IV only
Administrative information section would include, but not limited to, information regarding personnel
policies and procedures, organizational relationships, job descriptions, and control procedures for
use of departmental resources. The institution’s and the department’s philosophies, together with
their description, development, objectives and goals, comprise the general information section.
Brown 46

_____ 210. Which of the following is/are characteristics of the information contained in policy and
procedure manuals?

I. Current
II. Reliable
III. Static

A) I only
B) II only
C) I, II III only
D) I, II only
E) II, III only
The information contained in the policy and the procedure manual must be current and reliable.
Because of the dynamic character of the pharmacy department, the manual must also be flexible
and readily revisable.Brown 56

_____ 211. Which of the following factors mainly determine the type of services offered by a
hospital?

I. Government health policies


II. The health care services demand of the patients in the target market
III. The resources of the organization
A) I only
B) II only
C) I, II III only
D) I, II only
E) II, III only
In health care terms, the product or good is presented in terms of service offered. The type of service
(i.e. inpatient care, outpatient care, rehabilitation service, etc.) is determined by the demands of the
patients in the target market and the resources of the organization available. Brown 58

____ 212. Which of the following are components of hospital marketing?

I. Product or type of service


II. Price
III. Place
IV. Promotion

A) I and II only
B) II and IV only
C) I, II, III only
D) II, III, IV only
E) I, II, III, IV
The four P’s of marketing include the product, price, place and promotion. Place refers to where the
services are offered. Promotion refers to how the organization lets know about its services. Brown
58-59

____ 213. Refers to an amount that is above the break-even cost of providing a product or service
and is necessary to provide for replacement and upgrading of hospital equipment and facilities.

A) Overhead costs
B) Revenue deductions
C) Direct personnel costs
D) Supply cost
Profit margin, definition. A revenue deduction refers to a percentage of third party disallowances,
contractual allowances, bad debts, etc., which reduces the actual revenue received from charges.
Direct personnel cost refers to salaries for the man hours required; a percentage for vacation and
absences ; a percentage to account for an average productivity of less than 100%; and, a
percentage for fringe benefits. Direct supply cost accounts for labels, vials, syringes, needles, etc.
used in preparing or distributing the product. Fixed overhead cost refers to a percentage of fixed
departmental management, supervision and clerical coat, office supplies, subscriptions, equipment,
depreciation etc. Brown 67

____ 214. Refers to the pricing method wherein the price is calculated by adding a fixed fee to the
acquisition cost of the unit of product dispensed.

A) Percentage markup method


B) Dispensing fee method
C) Per diem method
D) Standard method
E) Special cost method
Dispensing fee method is the most commonly used pricing method. This method allocates the
pharmacy service charge without any relationship to the cost of the product dispensed. Brown 68

____215. Refers to the pricing method where the patient receiving a more costly drug pays for a
proportionally larger part of overall pharmacy service costs regardless of amount of service received.

A) Percentage markup method


B) Dispensing fee method
C) Per diem method
D) Standard method
E) Special cost method
The percentage mark up method is based on the assumption that the inventory holding cost of a
higher cost product is proportionally higher. This method ignores the professional service
component, and that the cost of drug product.Brown 68S

_____ 216. Refers to the pricing method wherein the average drug cost per patient day, the average
pharmacy service cost per patient day, and the desired profit margin are computed to arrive at a
single pharmacy charge for each day the patient stayed in the hospital.

A) Percentage markup method


B) Dispensing fee method
C) Per diem method
D) Standard method
E) Special cost method
The per diem charge method has been promoted on the premise that the pharmacy costs do not
vary sufficiently among patients to warrant the clerical and logistical costs of processing charges for
individual units or doses of drugs. Brown 68

____ 217. A visual display technique that can be used as a scheduling tool for the entire planning
process.

A) Curve fitting
B) Gant chart
C) Cyclical change
D) Regression
E) Distribution
The Ghantt chart can be modified to show activities, persons responsible, and the time schedule in
graphic form. Brown 161

____ 218.A physical quantity of a drug product ordered by a prescriber to be administered to a


specific patient at one time, in ready to administer form with no further physical or chemical
alterations required.

A) Unit dose package


B) Single unit dose
C) Unit dose
D) Multiple dose
E) Unit dose dispensing system
Unit dose, definition. Brown 325

_____ 219. A package that contains one discrete pharmaceutical dosage form

A) Unit dose package


B) Single unit dose
C) Unit dose
D) Multiple dose
E) Unit dose dispensing system
Unit dose package, definition. Brown 325

____ 220. Refers to the act of a pharmacist in supplying one or more drug products to or for patient,
usually in response to an order from an authorized prescriber.

A) Patient counseling
B) Drug utilization review
C) Distribution
D) Compounding
E) Dispensing
Dispensing, definition. Brown 325

_____ 221. Refers to a system that has as its purpose the selection, acquisition, control, storage,
dispensing, delivery, preparation and administration of a drug product in health care institutions in
response of an authorized prescriber.

A) Drug distribution control system


B) Drug use information system
C) Drug distribution system
D) Drug packaging system
E) Drug availability system
Drug distribution system, definition. It may also be referred as medication distribution system, drug
management system, or drug management system, or drug delivery system. Brown 325

____ 222. Which of the following are considered as drug distribution control information necessary to
ensure that a unit dose package is delivered as ordered to the right patient, at the right date, at the
right time?

I. A description of the unit dose ordered


II. Socio-economic status of the patient
III. Identification and location of the patient
IV. Time and date of administration

A) I, II, III, IV
B) I, II, III only
C) I, III, IV only
D) II, III, IV only
E) I, II IV only
Other drug distribution control information include: an order validated by a pharmacist; transportation
instructions; labeling, both in route and in temporary storage locations; and feedback signaling
deviations from the performance standards. Brown 325-326

____223. A standard “bid” dosing schedule means that the drug must be given at:

A) 7am and 7pm


B) 8am and 8pm
C) 6am and 6pm
D) 9am and 9pm
E) 10am and 10pm
There exists a standard dosing schedule in each hospital that dictates for a given patient care unit
the hours at which doses will be given. Generally, the dosing schedule are as follows: tid
9AM-1PM-5PM; bid is 10AM-10PM; and 16h is 9 AM-3PM-9PM-3AM. Brown 325-326

_____ 224. A standard “tid” dosing schedule means that the drug must be given at:

A) 8am-12nn-4pm
B) 9am-1pm-5pm
C) 10am-2pm-6pm
D) 7am-11am-3pm
E) 6am-10am-2pm
Refer to #223.

_____ 225.a standard “q6h” dosing schedule means that the drug must be given at:

A) 7am-1pm-7pm-1am
B) 8am-2pm-8pm-2am
C) 9am-3pm-9pm-3am
D) 6amn-12nn-6pm-12am
E) 10am-4pm-10pm-4am
Refer to #223.

_____226. Refers to a drug distribution system wherein a bulk supply of each drug product is
maintained on the nursing unit in advance of need and the nurse prepares the dose s for
administration.

A) Patient prescription system


B) Floor stock
C) Emergency carts
D) Unit dose dispensing system
E) Decentralized system
Floor stock system cuts the lines of communication for drug orders short. Decisions about orders
and dose preparation are decentralized and can be made at the patient’s bedside. The distribution
from the pharmacy to patient care areas can be done as a batch. At present, this system is utilized
primarily on military and government hospitals, where the demand for unit dose is specialized and
thereby limited in variety. Brown 333

____ 227. Refers to a drug distribution system wherein all drugs are retained in the pharmacy until
the order is received, and then a multiple dose supply is dispensed to the nursing unit labeled for
use by a specific patient.

A) Patient prescription system


B) Floor stock
C) Emergency carts
D) Unit dose dispensing system
E) Decentralized system
The placing of each patient’s doses in drawer of a medicine cart or cassette with his or her name on
it performs essentially the same control function as labeling a prescription with the patient’s name.
Brown 333

____228. Factors aside from the system design that affect the total system cost of maintaining a
hospital drug distribution system.

I. Mission of the hospital


II. Bed size
III. Physical layout of the hospital
IV. Enforcement of performance standards

A) I, II, III,IV
B) II and III only
C) I, II, IV only
D) I, IV only
E) I, III, IV only
Other external factors that affect the total system cost of a hospital drug distribution system
maintenance include: objectives of the system; policies on personal utilization; and automation.
Brown 342

_____229. Which of the following is/are advantages of unit dose dispensing systems?

I. Less drug losses due to waste and pilferage


II. Greater accuracy in inventory control, cost accounting, and charging
III. Improved ability of the hospital to attract and retain highly qualified
and motivated pharmacy personnel

A) I and II only
B) II and III only
C) I and III only
D) II only
E) I, II, III only
The cost per dose to the hospital operating under a unit dose system is less. Decentralized
distribution systems are more expensive than centralized systems due to loss of economies of scale,
poor utilization of personnel due to inflexibility in staffing, duplication of inventories, extra space and
equipment costs, and poor intrapharmacy communication control. Brown 345

____ 230. A “stat” order means:

I. The drug is to be administered immediately


II. The order takes precedence over the regularly scheduled medications
III. The drug is to be administered at a specified time

A) I only
B) I and II only
C) I and III only
D) I, II, III only
E) II and III only
A stat order takes precedence over the regularly scheduled medications since it needs to be
administered immediately. The ordering of such is sometimes abused with the result that they soon
become treated as any other routine order. Brown 352

_____231. Which of the following is/are true regarding subcutaneous injection administration?

I. The main areas of administration are the thigh, abdomen and the
upper arm
II. The needle is inserted through the skin at a 90 degree angle
III. Rotation of the injection site will reduce the chance of local irritation

A) I, II, III only


B) I and II only
C) I and III only
D) II and III only
E) II only
In subcutaneous injection administration, a 23-gauge needle of ¾-inch length is usually selected. It
is injected at a 45-60 degree angle with a quick dart-throwing action of the wrist. Brown 353

_____232. Which of the following is/are true regarding intramuscular injection administration?

I. The usual sites are the deltoid and gluteal muscles


II. 5-6mLis usually the recommended maximal volume
III. The needle is inserted through the skin at a 45-60 degree angle

A) I, II, III only


B) I, II only
C) I, III only
D) I only
E) III only
In intramuscular injection administration, the needle is inserted by the dart technique through the
skin at a 90-degree angle. The recommended maximal volume is usually set as 3-4 ml. Brown 353
______ 233. Which of the following is/are true regarding intradermal injection?

I. Used to check circulating antibodies to the injected antigen


II. Intradermal skin test is usually read at 24, 36, 48 hours
III. A reddened area of induration of 5mm or grater usually indicates an intact
immune system

A) I, II, III only


B) I, II only
C) I, III only
D) I only
E) III only
The intradermal injection is primarily used to inject antigens just below the dermal layer of the skin.
The technique for this type of injection is to cleanse the area with 70% alcohol or iodophor, hold the
skin taut, and insert the needle bevel up, approximately 1/8 inch into the skin, at an angle almost
parallel to the skin. Brown 354

_____ 234. Which of the following are indications of intravenous therapy?

I. Replacement of fluids and electrolytes


II. Parenteral nutrition
III. Administration of drugs requiring a rapid high blood level
IV. Emergency situations like cardiac arrest

A) I, III only
B) I, II, III only
C) I, III, IV only
D) II, III, IV only
E) I, II, III, IV
In addition, intravenous therapy can also be indicated for administration of drugs that are too
irritating to be given intramuscularly or subcutaneously such as vincristine, and when no other route
is available for the administration of the medication. Brown 354

_____ 235. Which of the following are possible adverse effects of intravenous administration?

I. Thrombophlebitis
II. Air emboli
III. Speed shock
IV. Injection of pathogens

A) I, III only
B) I, II, III only
C) I, III, IV only
D) II, III, IV only
E) I, II, III, IV
Although IV injection provides the mist rapid and complete absorption of a medication, there are
adverse effects and complications that could possibly result from it. These include thrombophlebitis,
speed shock, air emboli and injection of pathogens. Brown 354
_____ 236. The optimal pH for aminophylline:

A) Below 7.0
B) Above 7.0
C) Above 8.0
D) Below 8.0
E) Above 6.0
The optimal pH range for aminophylline is above pH 8.0 theophylline crystals will deposit below pH
8.0, but probably not unless the concentration is over 40mg/mg. the high pH of aminophylline may
also create stability problems with a number of antibiotics and other alkaline-labile compounds.
Brown 373

____ 237.Which of the following is/are nutrition support activities of a pharmacist?

I. Catheter care
II. Nutritional assessment
III. Parenteral formulation design
IV. Metabolic monitoring of therapy

A) I and III only


B) II and III only
C) I, III, IV only
D) III and IV only
E) I, II, III, IV
Nutrition support activities of a pharmacist also include ensuring the availability of IV nutrients,
proper preparation and delivery of TPN preparations, supply management and education of home
patients and pharmacologic interventions with nutrition support. Catheter care is a function of a
nurse and nutritional assessment is a function of a dietician. Brown 376

____ 238. Which of the following is/are the essential human nutrients for intravenous inlets?

I. Carbohydrates
II. Proteins
III. Electrolytes
IV. Fats

A) I and III only


B) II and III only
C) I, III, IV only
D) III and IV only
E) I, II, III, IV
The seven basic nutrients required by the human body that are essential for intravenous diets
include carbohydrate, protein, electrolytes, fats, vitamins, trace minerals and water. Brown 379

___ 239. The most important human nutrient required for tissue synthesis, repair, transport of body
nutrients and waste, and maintenance of immune function.

A) Carbohydrates
B) Proteins
C) Fats
D) Electrolytes
E) Water
For the hospitalized patient, protein of high biologic value as provided by commercially available
amino acid products is recommended. Brown 379

____ 240.The most widely utilized carbohydrate in parenteral nutrition because of its low cost,
availability, and proven utility.

A) Glycerol
B) Fructose
C) Sucrose
D) Dextrose
E) Lactose
Dextrose is available in a wide variety of concentration to allow formulation individualization. A
relatively new carbohydrate source is glycerol. Aside from its ability to yield same kcal/g as dextrose
, it has the advantage of not stimulating insulin release. Brown 381

____ 241. Which of the following is/ are mechanical tube feeding complications?

I. Aspiration pneumonia
II. Mucosal erosions
III. Hypertonic
IV. Nasopharyngeal irritation

A) I. II only
B) III, IV only
C) I, II, III, only
D) I, II, IV only
E) I, II, III, IV
Mechanical tube feeding complications include aspiration pneumonia, mucosal erosions, tube lumen
obstruction, tube displacement. And nasopharyngeal irritation. Brown 387

____ 242. Which of the following is/ are metabolic tube feeding complication?

I. Electrolyte imbalance
II. Glucose intolerance
III. Diarrhea
IV. Distention

A) I, II only
B) III, IV only
C) I, II, III only
D) I, II, IV only
E) I, II, III, IV
Metabolic tube feeding complications include electrolyte imbalance, fluid overload, glucose
intolerance, and hypertonic dehydration. Brown 387
______ 243. The extent to which a technique consistently measures whatever it measures
regardless of the investigator or the situation.

A) Validity
B) Dependability
C) Vulnerability
D) Sensitivity
E) Reliability
Reliability, definition. Brown 437

_____ 244. Which of the following is/are gastrointestinal tube feeding complications?

I. Electrolyte imbalance
II. Glucose intolerance
III. Diarrhea
IV. Distention

A) I and II only
B) III, IV only
C) I, II, III only
D) I, II, IV only
E) I, II ,III, IV
Gastrointestinal complications include cramping, diarrhea, distention and vomiting. Brown 387

_____ 245. The extent to which a technique measures what is intended to measure?

A) Validity
B) Dependability
C) Vulnerability
D) Sensitivity
E) Reliability
Validity, definition. Brown 437

____ 246. An authorized, structured and continuing program that reviews, analyzes, and interprets
patterns of the drug usage in a given health care delivery system against pre-determined standards.

A) Pharmaceutical care plan


B) Therapeutic drug monitoring
C) Drug utilization
D) Quality assurance
E) Drug auditing and inventory control
Drug utilization review, definition. Brown 449

____247. Pre-clinical studies that determine the new drug’s absorptive, distributive, metabolic and
excretory pathways.

A) Pharmacologic studies
B) Pharmacodynamic studies
C) Toxicologic studies
D) Pharmacokinetic studies
E) Biogenetic studies
The general purpose of pharmacokinetic studies is to estimate the pharmacokinetic parameters,
such as plasma drug concentration, biologic half-life, drug distribution, metabolism and excretion.
Brown 471

____ 248. Pre-clinical studies that determine the action of the new drug in animals to estimate the
magnitude of its intended therapeutic effect.

A) Pharmacologic studies
B) Pharmacodynamic studies
C) Toxicologic studies
D) Pharmacokinetic studies
E) Biogenetic studies
Pharmacologic studies are initiated to obtain data to show that there will not be unreasonable risks in
conducting human research. This is done by extrapolating, when possible, the animal data to
humans. Brown 471

_____ 249. Pre-clinical studies that determine the relative safety in human and monitor parameters
that will be used in clinical trials.

A) Pharmacologic studies
B) Pharmacodynamic studies
C) Toxicologic studies
D) Pharmacokinetic studies
E) Biogenetic studies
Preclinical toxicity studies are important because the initial benefit or risk assessment for use of the
drug in humans is made from these studies. Brown 471

______250. Any drug that has not yet been released for general use, and has not yet been cleared
for sale in commerce.

I. Investigational drug
II. Experimental drug
III. Regulated drug
IV. New drug

A) I only
B) I and II only
C) I and III only
D) I and IV only
E) I, II IV only
Investigational, or new drug, definition. This drug is not necessarily a new chemical substance, but
maybe an old or approved drug proposed for a new use, a new combination of old drugs in new
proportion, or a new dosage form or method of administration, or it maybe a new drug because it
contains a new component such as an excipient, a coating, or a menstrum. Brown 471
______ 251. Various strategies and techniques are in place for use in counseling and educating
patients. The affective domain of learning process involves:

I. Perceptions
II. Emotions
III. Memory
IV. Beliefs
A) I only
B) I, II only
C) I, II, IV only
D) I, II, III only
E) IV only
Tine affective domain of learning process involves the formation of attitudes such as feelings, beliefs,
perceptions, emotions, and appreciations. Remington 1679

_____ 252. The behavioral domain of learning process involves:

I. Decision making
II. Actions
III. Physical abilities
IV. Perceptions

A) I only
B) I, II only
C) I, II, IV only
D) I, II, III only
E) IV only
This behavioral domain of learning process is developed from what the person knows and feels, in
conjunction with the nature and requirements of their social environments. This domain covers
actions, decision making and physical abilities. Remington 1679

_______253. An order for medication issued by physician, dentist, or other properly licensed medical
practitioner.

A) Medication profile
B) Patient chart
C) Voucher
D) Prescription
E) Insurance policy
The prescription order is a part of the professional relationship among the prescriber, the pharmacist,
and the patient. Prescriptions designate a specific medication and dosage to be administered to a
particular patient at a particular time. Remington 1687

_______ 254. Medications that may be dispensed legally only on prescription.

A) Over-the-counter drugs
B) Sample drugs
C) Imported drugs
D) Investigational drugs
E) Legend drugs
Medications that may dispensed legally only on prescriptions are referred to as prescriptions or
legend drugs. Nonprescription drugs or over-the-counter drugs may be obtained without prescription.
Remington 1687

________ 255. Prescription part generally understood to be a contraction of the Latin verb “recipe”
meaning “take thou” or “you take”.

A) Superscription
B) Inscription
C) Subscription
D) Signature
E) Transcription
Superscription is the Rx symbol. This symbol is believed to have originated from the sign of Jupiter
which was employed by the ancients in requesting aid in healing. Remington 1688

_______ 256. The body or principal part of the prescription order that contains the names and the
quantities of the prescribed ingredients.

A) Superscription
B) Inscription
C) Subscription
D) Signature
E) Transcription
Today, since most drugs are already prepared or prefabricated into dosage forms, the medications
may be prescribed under their trademarked or manufacturer’s propriety name or by their generic
names. Remington 1688

______ 257. Part of prescription that consists of directions to the pharmacist for preparing the
prescription.

A) Superscription
B) Inscription
C) Subscription
D) Signature
E) Transcription
In a majority of prescriptions, the subscription serves merely to designate the dosage form and the
number of dosage units to be supplied. Remington 1688

______258. Part of the prescription where the prescriber indicates the directions for the patient’s use
of medication.

A) Superscription
B) Inscription
C) Subscription
D) Signature
E) Transcription
The word Signature is usually abbreviated to Signa or sig. which means mark thou. The directions
are commonly written using abbreviated forms of English and Latin terms or a combination of the
two. Remington 1690

_____ 259. They are commonly used abbreviations in prescription and medication orders.

A) Before meals
B) Up to
C) Left ear
D) Each ear
E) Of each
The abbreviation means as means of each. Remington 1689

______260. The abbreviation “po” means:

A) After surgery
B) Rectally
C) By mouth
D) When necessary
E) After eating
F) The abbreviation po means by mouth.
G) Remington 1689

_____261. The abbreviation “ss” means:

A) Without
B) Symptom
C) Suppository
D) One half
E) Immediately
The abbreviation as means one-half. Remington 1689

______262. Defined as the preparation, mixing, assembly, packaging, or labeling of a drug/ device
as a result of prescription-drug order.

A) Pharmacy dispensing
B) Pharmacy compounding
C) Pharmacy manufacturing
D) Pharmacy quality assurance
E) Pharmacy practice
Pharmacy compounding, definition. Remington 1689

______ 263. Used for bulk powders, large quantities of tablets, capsules, viscous liquids that cannot
be poured readily.
A) Prescription bottles
B) Applicator bottles
C) Wide-mouth bottles
D) Round vials
E) Sifter-top containers
Applicator bottles are used for applying liquid medication to a wound or skin surface. Round vials are
used primarily for solid dosage forms as capsules and tablets. Remington 1697

________ 264. Used for powders to be applied by sprinkling:

A) Prescription bottles
B) Slide boxes
C) Wide-mouth bottles
D) Dropper bottles
E) Aerosol containers
Slide boxes are used for dispensing suppositories and powders appeared in packets. Aerosol
containers are used for pharmaceutical aerosol products. They are pressurized systems dispensed
by the pharmacist in the original container. Remington 1697

_______ 265. Used for dispensing solid dosage forms:

A) Prescription bottles
B) Applicator bottles
C) Wide-mouth bottles
D) Round vials
E) Sifter-top containers
Ointment jars and collapsible tubes are used to dispense semi-solid dosage forms, such as creams
and ointments. Dropper bottles are used for dispensing ophthalmic, nasal, otic, or oral liquids to be
administered by drop. Remington 1697

______266. Any substance that may be considered a food or part of a food that provides medical or
health benefits, including prevention and treatment of disease.

A) Designer food
B) Nutraceutical
C) Pharmafood
D) Phytochemical
E) Chemopreventive agent
Nutraceutical, definition. Remington 1734

_____267. Substances found in edible fruits and vegetable that may be ingested by humans daily in
gram quantities and that exhibit an potential for modulating human metabolism in a manner
favorable for cancer prevention.

A) Designer food
B) Nutraceutical
C) Pharmafood
D) Phytochemical
E) Chemopreventive agent
Phytochemical, definition. Remington 1734

_____268. Nutritive or non nutritive food component being scientifically investigated as a potential
inhibitor of carcinogenesis for primary and secondary cancer.

A) Designer food
B) Nutraceutical
C) Pharmafood
D) Phytochemical
E) Chemopreventive agent
Chemopreventive agent, definition. Remington 1734

_____ 269. Processed food that are supplemented with food ingredients naturally rich in
disease-preventing substances.

A) Designer food
B) Nutraceutical
C) Pharmafood
D) Phytochemical
E) Chemopreventive agent
Designer food, definition. Remington 1734

_____270. Which of the following are considered as categories of Complementary and Alternative
Medicine?

I. Mind-body interventions
II. Biolectromagnetic interventions
III. Herbal medicine
IV. Diet and nutrition

A) I, II only
B) II, IV only
C) I, II ,III only
D) I, II, IV only
E) I, II, III, IV
According to the National Center for Complementary and Alternative Medicine, the seven categories
of complementary and alternative medicine are: mind-body interventions; bioelectromagnetic
therapies; alternative systems of medical practice; manual healing methods; pharmacological and
biological treatments; herbal medicine; and, diet and nutrition. Remington 1766

_____ 271. An ancient Chinese healing art that employs fine needles inserted at various locations in
the body to restore the smooth flow of energy.
A) Hypnosis
B) Allopathy
C) Acupuncture
D) Acupressure
E) Homeopathy
Acupuncture has been a primary practice of the health care system of China for at least 2500 years.
The National Institute of Health concluded that there is a clear evidence of efficacy for treating
postoperative and chemotherapy nausea and vomiting, the nausea of pregnancy and postoperative
dental pain Remington 1766

_______272. A therapeutic method that clinically applies the Law of Similar (like cures like) and the
uses medically active, potentized substances at weak or infinitesimal doses.

A) Ayurvedic medicine
B) Chiropractic
C) Hypnosis
D) Homeopathy
E) Allopathy
Homeopathy, definition. Remington 1779

______ 273. A state of altered consciousness, sleep or trance induced artificially in a subject by
means of verbal suggestion or by the subject concentrating upon some subject.

A) Hypnosis
B) Chiropractic
C) Allopathy
D) Homeopathy
E) Iridology
Hypnosis, definition. Remington 1779

______ 274. A diagnostic tool that purports to correlate changes in the color and texture of the iris
with mental and physical disorders.

A) Hypnosis
B) Chiropractic
C) Allopathy
D) Homeopathy
E) Iridology
Iridology, definition. Remington 1779

_____ 275. Referred to a specialty practice of pharmacy that focuses upon the safe and efficacious
use of radioactive drugs.

I. Nuclear pharmacy
II. Radiopharmacy
III. Institutional Pharmacy
A) I only
B) II only
C) I and II only
D) I, II ,III only
E) I, III only
Nuclear pharmacy deals with the safe and efficacious use of radioactive drugs. Nuclear pharmacy is
also known as radiopharmacy. Remington 1781

_____ 276. A drug that exhibits spontaneous disintegration of unstable nuclei with the emission of
nuclear particles or photons.

A) Designer drug
B) Prohibited drug
C) Dangerous drug
D) Over-the-counter drug
E) Radioactive drug
Radioactive drug, definition. It includes any non-radioactive kit or muscle generator that is intended
to be used in the preparation of any such substance. Remington 1781

____277. Which of the following is/are true regarding radiopharmaceutics?

I. Lack pharmacological effects


II. Employed as tracers of physiological functions
III. Small amounts produce negligible effects on biological processes

A) II only
B) I and II only
C) II, III only
D) I, II, III only
E) III only
Most radiopharmaceuticals consist of radioactive atoms attached to, or incorporated into other
chemical compounds that serve to carry the radioactive atoms to the intended tissues or organs.
Their radioactivity allows noninvasive external monitoring or targeted therapeutic irradiation.
Remington 1781

_____ 278. A diagnostic radiopharmaceutical used for bone scans:

A) Tc-99m diphosphonates
B) Tc-99m macroaggregated albumin
C) TI- thalous chloride
D) Sr-89 strontium chloride
E) I-131 sodium iodide
Radiopharmaceuticals can either be diagnostic or therapeutic. Examples of diagnostic include:
Tc-99m diphoshonates for bone scans; Tc-99m macroaggregated albumin for lung scans; and TI-201
thallous chloride for myocardial perfusion scans. Remington 1781

_____ 279. A diagnostic radiopharmaceutical for myocardial perfusion scans:


A) Tc-99m diphosphonates
B) Tc-99m macroaggregated albumin
C) TI- thalous chloride
D) Sr-89 strontium chloride
E) I-131 sodium iodide
Refers to #278

____ 280. A diagnostic radiopharmaceutical used for lung scans:

A) Tc-99m diphosphonates
B) Tc-99m macroaggregated albumin
C) TI- thalous chloride
D) Sr-89 strontium chloride
E) I-131 sodium iodide
Refers to #278

____ 281. Which of the following is/are true regarding the functions of Vit. A?

I. Acts as anti-rachitic vitamin


II. Maintains the integrity of epithelial membranes
III. Essential in the formation of rhodopsin and the normal functioning of
the retina

A) I only
B) II, III only
C) I, II, III only
D) I, III only
E) II only
Vitamin A was the first fat-soluble vitamin discovered. Of its known function, it role in the visual
process is established best. It also participates in the maintenance of the integrity of the epithelial
membranes. Vitamin D is the antirachitic vitamin. Remington 1799-1800

_____282. Which of the following is/are true regarding the functions of Vit.C?

I. Formation of intercellular collagen


II. Healing of bone fractions
III. Metabolism of tyrosine

A) I only
B) II, III only
C) I, II, III only
D) I, III only
E) II only
Vitamin c, also known as the antiscorbutic vitamin, is needed to prevent and cure scurvy. It serves as
a function in the formation of intercellular collagen, healing of bone fractures and tyrosine
metabolism. Remington 1805
____283. Hospitals may be classified in different ways according to:

I. Type of service
II. Length of stay
III. Ownership
IV. Bed capacity

A) I, II, III, IV
B) I, II, II only
C) I, III only
D) I, IV only
E) I, II, IV only
Hospitals can be classified in different ways. The classification may be according to type of service,
length of stay, ownership and bed capacity. Remington 1915

_____ 284. Which of the following are fundamental functions of a hospital?

I. Patient care
II. Wellness
III. Research
IV. Education

A) I, II only
B) I, II, IV only
C) I, II ,III only
D) I, II only
E) I, II, III, IV
Traditionally, hospitals have a limited function of giving treatment and care of the sick and the
injured. However, through the years, its functions expanded to cover patient care, education,
research, and wellness. Remington 1916

_____285. Which of the following is/are true regarding open questions?

I. Allow the patient sufficient latitude in interpreting and responding to


requests for information
II. Cannot be answered in one phrase or with “yes” or “no”
III. Its unlimited use will result in an overly long unfocused and inefficient
interview

A) I only
B) II, III only
C) I, II, III
D) I, II only
E) II only
Open questions are broad and asks for information and specifics about the topic in a general way.
The excessive use of open questions will result in an overly long, rambling, unfocused, and
inefficient interview. Remington 1961
_____ 286. The provision of integrated, accessible health-care services by clinicians who are
accountable for addressing a large majority of personal health-care needs developing a sustained
partnership with patient, and practicing in the context of family and community.

A) Disease management
B) Acute care
C) Primary care
D) Institutional care
E) Long-term care
Primary care, definition. Remington 1991

_____ 287. An evaluative approach to health-care delivery system that attempts to improve
outcomes for patients with a specific disease while optimizing the overall use of health-care
outcomes.

A) Patient-focused care
B) Benefit management
C) Disease management
D) Case management
E) Primary care
Disease management, definition. Remington 1993

_____ 288. A process by which an experienced professional works with patients, providers, and
insurers to coordinate all services deemed necessary to provide the patient with medically
appropriate health care.

A) Patient-focused care
B) Benefit management
C) Disease management
D) Case management
E) Institutional care
Case management, definition. Remington 1994

_____ 289. Categories listed by application of the US Food and Drug Administration definitions to
available clinical data in order to define a drug’s potential to cause birth defects or fetal death.

A) Phenol coefficient
B) Relative risk ratio
C) Bioequivalence requirement
D) Pregnancy risk
E) Therapeutic index
The pregnancy risk category identifies the potential risk to the fetus. The categories are labeled A, B,
C, D, X, and NR. Drugs in category A usually are considered safe to use in pregnancy, category X
drugs are usually contraindicated. Physician’s Drug Handbook xi

_____290. A pregnancy risk category that indicates an adequate studies in pregnant women have
failed to shop a risk to the fetus in the first trimester of pregnancy and no evidence of risks in later
trimesters.
A) Category B
B) Category D
C) Category A
D) Category C
E) Category S
Animal studies have shown an adverse effect on the fetus, but there are no adequate studies in
human.

_____ 291. A pregnancy risk category that indicates that studies in animals or human show fetal
abnormalities, or adverse reaction reports indicate evidence of fetal risk.

A) Category B
B) Category A
C) Category O
D) Category X
E) Category Y
There is evidence of risk to the human fetus.

_____292. A pregnancy risk category that indicates that animal studies have shown an adverse
effect on the fetus, but there are no adequate studies in humans.

A) Category C
B) Category B
C) Category D
D) Category E
E) Category O
Adequate studies in pregnant women have failed to show a risk to the fetus in the first trimester of
pregnancy, and there is no evidence of risk in later trimesters.

_____ 293. A pregnancy risk category that indicates that there is evidence of risk to the human
fetus, but the potential benefits of use in pregnant women may be acceptable despite potential risks.

A) Category E
B) Category B
C) Category D
D) Category A
E) Category X
Animal studies have shown an adverse effect on the fetus, but there are no adequate studies in
humans.

____ 294. A pregnancy category that indicates that animal studies have not shown an adverse effect
on the fetus, but there are no adequate clinical studies in pregnant women.

A) Category D
B) Category B
C) Category X
D) Category A
E) Category C
Animal studies have not shown an adverse effect on the fetus, but there are no adequate clinical
studies in pregnant women.

____ 295. Which of the following is/ are forms of non-verbal communication?

I. Clothing and style


II. Posture
III. Eye contact
IV. Proximity to other person

A) II, III only


B) III only
C) I, II, III only
D) I. II, IV only
E) I, II, III, IV
Forms of non verbal communication may be speech related (e.g. timing and pauses), non-verbal
behavior (e.g. posture and position, eye contact, proximity to other person, etc.), use of body,
aspects of personal appearance (e.g. clothing and hairstyle, smell, etc.,), symbols, sign languages,
written word, or use of media. Katz 119-120

_____ 296. Which of the following is/are examples of mechanical barriers to effective listening and
communication?

I. Noise
II. Harsh lighting
III. Impaired eyesight
IV. Uncomfortable chairs

A) I, II, III, IV
B) I, II, III only
C) II, III only
D) I, II, IV only
E) I. II only
Mechanical barriers to effective listening and communication may include lack of a conducive
environment in which to communicate, fatigue, medication side effects and impaired sight. Katz 131

_____ 297. A skilled and principled use of relationships to facilitate self-knowledge, emotional
acceptance and growth, and the optimal development of personal resources.

A) Communication
B) Counseling
C) Listening
D) Health promotion
E) Problem-solving
Counselling, definition by British Association of Counselling, 1989. Katz 133

_____ 298. The ability to see the world from the point of view of another person, through their frame
of reference, through their conceptual and emotional spectacle.

A) Sympathy
B) Genuineness
C) Listening
D) Empathy
E) Involvement
Empathy, definition. Katz 143

____ 299. According to the preventive model, which of the following is/ are function/s of tertiaty level
of prevention?

I. Prevention of deterioration, relapse, and complications


II. Promotion of rehabilitation
III. Prevention of onset disease

A) I, II, III only


B) I, II only
C) I, III only
D) II. III only
E) I only
According to the preventive model, tertiary level of prevention is concerned to prevent deterioration,
relapse and complications, promote rehabilitation, and help adjustment to terminal conditions. Katz
164

____ 300. According to the preventive model, which of the following is/ are function/s of secondary
level of prevention?

I. Help adjustment to terminal conditions


II. Minimization of the severity of an existing disease
III. Reversal of progress of an existing disease

A) I, II III only
B) I, II only
C) I, III only
D) II, III only
E) I only
According to preventive model, secondary level of prevention is concerned to prevent development
of existing disease, minimize its severity, reverse its progress, and reduce prevalence. Primary level
of prevention is concerned to prevent onset of disease, and reduce incidence Katz 164
Clinical and Hospital Pharmacy (PACOP BLUE)

1. It is refers to the responsible of drug therapy to achieve definite outcomes that are intended to improve a
patients quality of life
A. Professional – patient relationship
B. Therapeutic drug monitoring
C. Drug therapy assessment
D. Pharmaceutical care
E. Formal documentation

2. It involves measuring directly and indirect cost attributable to a specific disease.


A. Cost-of –illness evaluation
B. Cost-effectiveness analysis
C. Cost-minimization analysis
D. Cost-utility analysis
E. Cost- benefit analysis

3. The method that allows for identification, measurement, and comparison of the costs of a program or
treatment alternative
A. Cost-of –illness evaluation
B. Cost-effectiveness analysis
C. Cost-minimization analysis
D. Cost-utility analysis
E. Cost- benefit analysis

4. The method that involves the determination of the least costly alternative when comparing two or more
treatment alternatives
A. Cost-of –illness evaluation
B. Cost-effectiveness analysis
C. Cost-minimization analysis
D. Cost-utility analysis
E. Cost- benefit analysis

5. The method that integrates the patient preferences and health-related quality of life
A. Cost-of –illness evaluation
B. Cost-effectiveness analysis
C. Cost-minimization analysis
D. Cost-utility analysis
E. Cost- benefit analysis

6. A way of summarizing the health benefits and resources used by competing health care programs so that
policy makers can choose among them
A. Cost-of –illness evaluation
B. Cost-effectiveness analysis
C. Cost-minimization analysis
D. Cost-utility analysis
E. Cost- benefit analysis

7. It refers to the value assigned to duration of life as modified by impairments, functional states, perceptions,
and social opportunities that are influenced by disease, injury, treatment or policy
A. Optimum health
B. Quality of life
C. Health outcomes
D. Responsiveness
E. Wellness of being

8. A study design in which neither the study subject nor the study staff is aware of which group or intervention
the subjects has been assigned
A. Randomized control trial
B. Crossover study
C. Cohort study
D. Blinded study
E. Cross-sectional study

9. A retrospective comparison of casual factors or exposures in a group of persons with disease and those of
persons without disease
A. Randomized control trial
B. Case control study
C. Cohort study
D. Meta analysis
E. Open-label trial
10. A retrospective or prospective follow-up study of exposed and non-exposed defined groups in which a
variable of interest is measured
A. Cross sectional study
B. Case control study
C. Cross over study
D. Cohort study
E. Pen label study

11. A trial comparing treatments in which participants, on completion of one treatment, are switched to the other
A. Cross sectional study
B. Cross over study
C. Meta analysis
D. Open label trial
E. Cohort study

12. A study that examines the presence or absence of a disease and other variable in a defined population
A. Cross sectional study
B. Cohort study
C. Case control study
D. Cross over study
E. Case series

13. Which of the following best describes the neonates?


A. Those who are 1 month to 1 year of age
B. Those who are 1 year to 12 year of age
C. Those who are 12 to 16 years of age
D. Those between 1 day and 1 month of age
E. Those born before 37 weeks of gestational age

14. Which of the following best describes the infants?


A. Those who are 1 month to 1 year of age
B. Those who are 1 year to 12 year of age
C. Those who are 12 to 16 years of age
D. Those between 1 day and 1 month of age
E. Those born before 32 weeks of gestational age

15. Clinical manifestations of gray baby syndrome include:


I. Characteristic gray color
II. Abdominal distention
III. Hypertension
IV. Progressive shock
A. I only
B. I and IV only
C. I,II, and III only
D. I, II, and IV only
E. I and III only

16. Which of the following is associated with the use of thalidomide?


I. Polyneuritis
II. Mental retardation
III. Limb deformities
A. I only
B. I, II and III only
C. II and III only
D. III only
E. I and III only

17. Factors that increase the risk of drug related problems in the elderly include
I. Polypharmacy
II. Inappropriate prescribing
III. Medication adherence
IV. Multiple diseases
A. I only
B. I, II and III only
C. II and IV only
D. I, II and IV only
E. I, II, III, and IV

18. Potential difficulties that may occue while taking medication histories from the elderly include:
I. Impaired hearing
II. Mental acuity
III. Multiple diseases and medication
IV. Reliance on a caregiver for the history
A. I, II and III only
B. I, III and IV only
C. II, III and IV only
D. I, II and IV only
E. I, II, III and IV

19. Refers to the continual monitoring for unwanted effects and other safety-related aspects of marketed drugs
A. Pharmacoepidemiology
B. Pharmacovigilance
C. Pharmacoinformatics
D. Pharmacogenetics
E. Pharmacogenomics

20. Study the use of and effectsof drugs in a large number of people
A. Pharmacoepidemiology
B. Pharmacovigilance
C. Pharmacoinformatics
D. Pharmacogenetics
E. Pharmacogenomics

21. Any noxious, unintended, and undesired effect of a drug that occurs at doses used in humans for
prophylaxis, diagnosis or therapy
A. Adverse drug event
B. Allergy
C. Hypersensitivity
D. Adverse drug reaction
E. Idiosyncrasy

22. Which of the following is/are true regarding chest X-ray?


I. Provides supplemental information to the physical examination and usually the first
diagnostic test in a cardiac workup.
II. Provides details of internal cardiac structures.
III. Gives information about position and size of the heart and chambers and surrounding
anatomy.
A. I only
B. I, II and III
C. I and II only
D. I and III only
E. II and III only

23. The manifestations of chromium deficiency include:


I. Alopecia
II. Depigmentation of hair and skin
III. Red blood cell fragility
IV. Glucose intolerance
A. I and II only
B. II and IV only
C. I, II, III and IV
D. III only
E. IV only

24. The manifestation of zinc deficiency include:


I. Poor wound healing
II. Poor growth
III. Poor resistance to infection
A. I only
B. II and III only
C. I and IV only
D. III only
E. I, II and III

25. Incompatibilities with fat emulsion cause majority of formulation problems in TPN. Which of the following
affects lipid stability in TPN preparations?
I. Nature of amino acid solution
II. pH
III. the amount of dissolved oxygen in the solution
IV. electrolyte content
A. I, II and III only
B. II, III and IV only
C. I, II and IV only
D. I and II only
E. I and IV only

26. Ascorbic acid is the most rapidly oxidized vitamin. The rate of oxidation depends on:
I. Electrolyte content
II. Presence of trace animals
III. Amount of dissolved oxygen in the solution
IV. pH
A. IV only
B. II and III only
C. I, II and III
D. II, III and IV only
E. II and IV only

27. Which of the following is the most rapidly reduced vitamins?


A. Riboflavin
B. Niacin
C. Thiamine
D. Cevitamic acid
E. Ascorbic acid

28. Which of the following are associated with dilutional hyponatremia?


I. Administration of albumin
II. Congestive heart failure
III. Cirrhosis
A. II only
B. II and III only
C. I and II only
D. I only
E. I, II and III

29. Which of the following is/are true regarding blood urea nitrogen (BUN)?
I. End product of protein metabolism
II. Product by liver and kidney
III. Filtered completely at glomerulus
A. I and II only
B. I and III only
C. II and III only
D. I, II and III
E. I only

30. Which of the following accurately describes creatinine?


I. Not absorbed and secreted by the kidney
II. Filtered freely at the glomerulus
III. Metabolized by the kidney
A. I only
B. II only
C. III only
D. I and II only
E. II and III only

31. Which of the following is/are true regarding Aspartate aminotransferase (AST)?
I. Also called serum glutamic pyruvic transaminase
II. Abundant in heart and liver tissue
III. Used to evaluate myocardial injury and case prognosis of liver disease resulting from
heparocellular injury
A. I only
B. I and II only
C. I and III only
D. II and III only
E. III only

32. Which of the following is/are true regarding the comparison of ALT and AST enzyme?
I. ALT elevations persist longer than those of AST
II. ALT is more liver specific
III. The liver contains 3.5 times more AST than ALT
A. I, II and III
B. I and II only
C. II and III only
D. I and III only
E. II only

33. Which of the following may result to a decrease in hematocrit?


I. Hemolysis
II. Polycythemia vera
III. Sickle cell anemia
A. I only
B. II only
C. I and II only
D. I and III only
E. I, II and III

34. According to the seventh report of the Joint National Committee (JNC 7), which of the following best
describes the normal blood pressure?
A. An SBP less than 130, and DBP less than 85
B. An SBP between 120 and 139, or a DBP between 80 to 89
C. An SBP less than 120, and DBP less than 80
D. An SBP between 140 and 159, or a DBP between 90 to 99
E. An SBP equal to or greater than 160, or a DBP equal to or greater than 100

35. According to JNC 7, which of the following best describes Stage 1 hypertension?
A. An SBP less than 130, and DBP less than 85
B. An SBP between 120 and 139, or a DBP between 80 to 89
C. An SBP less than 120, and DBP less than 80
D. An SBP between 140 and 159, or a DBP between 90 to 99
E. An SBP equal to or greater than 160, or a DBP equal to or greater than 100

36. Which of the following are the objectives of evaluating patients with documented hypertension?
I. To assess lifestyle and identify other cardiovascular risk factors or concomitant that may
affect prognosis and guide treatment
II. To reveal identifiable causes of high blood pressure and allow patient self-medication
III. To assess the presence or absence of target organ damage and cardiovascular disease
A. I only
B. II only
C. I and II only
D. I, II and III only
E. I and III only

37. Routine laboratory tests for hypertension recommended before initiating therapy include:
I. Electrocardiogram
II. Urinary albumin excretion
III. Blood glucose and hematocrit
IV. Lipid profile
A. I and IV only
B. I and II only
C. I, II and III only
D. I, III and IV only
E. I, II and IV only

38. Lifestyle modifications for hypertension include:


I. Diet rich in potassium and calcium
II. Physical activity
III. Moderation of alcohol consumption
IV. Weight reduction for overweight and obese patients
A. II, III and IV only
B. I, III and IV only
C. I, II and IV only
D. I, II and III only
E. I, II, III and IV

39. Which of the following diuretics should be used as initial therapy for most patients with hypertension, either
alone or in combination?
A. Thiazide diuretics
B. Loop diuretics
C. Potassium-sparing diuretics
D. Osmotic diuretics
E. A and B

40. Which of the following statements is/are true regarding the potential unfavorable effectrs of anti hypertensive
drug choices?
I. Thiazide diuretics should be used cautiously in patients with gout
II. Beta blockers should generally be avoided in individuals with asthma
III. Aldosterone antagonists and potassium-sparing diuretics can cause hypokalemia
A. I, II and III
B. I and II only
C. I and III only
D. II and III only
E. I only

41. Factors that increase the chances of patients’ non adherence to antihypersensitive therapy include:
I. Misunderstanding of condition or treatment
II. Lack of patient involvement in the care plan
III. Unexpected adverse effects of medications
IV. Complexity of care
A. I and II only
B. III and Iv only
C. I, II and III only
D. I, II and IV only
E. I, II, III, and IV

42. Which of the following is/are true regarding cholesterol?


I. It is naturally occurring alcohol
II. It is the precursor molecule for the synthesis of the bile acids and steroid hormones
III. It is used by the body to form cell membranes
A. II only
B. II and III only
C. I and II only
D. III only
E. I, II and III

43. General principles of the diet therapy intended to lower cholesterol include:
I. Choose foods high in complex carbohydrates like starch and fiber
II. Replace monounsaturated fats with saturated fats and fish oils
III. Eat less high-fat, high cholesterol foods
A. I only
B. III only
C. I, II and III
D. II and III
E. I and III

44. Which of the following i/are true regarding bile acid resins as part of management of hypercholesterolemia?
I. Resins are not absorbed from the gastrointestinal tract and thus lack systemic toxicity
II. They reduce total LDL cholesterol in a dose-independent manner
III. They have a strong record established from years of use
A. I only
B. III only
C. I, II and III
D. II and III
E. I and III

45. Which of the following is/are the side effects of bile acid resins?
I. Constipation
II. Bloating and epigastric fullness
III. Nausea
IV. Flatulence
A. I and II only
B. II and IV only
C. I and IV only
D. II, III and IV only
E. I, II ,III and IV

46. Which of the following is/are true regarding niacin in the management of hypercholesterolemia?
I. It inhibits the mobilization of free acids from peripheral adipose tissue to the liver
II. It reduces the synthesis and secretion of VLDL particles by the liver
III. One of its side effects is vasoconstriction which leads to flushing, itching and headache
A. I and II only
B. II and IV only
C. I and IV only
D. II, III and IV only
E. I, II, III and IV
47. Which of the following drugs has the most potent cholesterol lowering potential?
A. Bile acid resins
B. HMG-CoA Reductase Inhibitors
C. Beta Blockers
D. ACE inhibitors
E. Calcium Channel Blockers

48. Drug interactions with statins that result in higher blood levels of the statin or active metabolite can increase
risk of myositis. Which of the following statins are most vulnerable to this interaction?
A. Atorvastatin and Pravastatin
B. Fluvastatin and Simvastatin
C. Cerivastatin and Lovastatin
D. Lovastatin and Simvastastin
E. Atorvastatin and Fluvastatin

49. A small part of clot that braeks off and travels to another part of the vascular system
A. Embolus
B. Thrombosis
C. Infarction
D. Ischemia
E. Stroke

50. Which of the following are the primary factors that influence the information of pathologic clots?
I. Abnormalities of blood flow that cause venous stasis
II. Abnormalities of blood vessel walls
III. Hypocoagubility resulting from alterations in the availability or the integrity of blood
clotting components
A. I and II only
B. II and III onyl
C. I and III only
D. I, II and III
E. I only

51. Which of the following is/are true regarding heparin?


I. A rapid-acting anticoagulant that is effective only when administered parenterally
II. Obtained from bovine lung or porcine intestinal mucosa
III. Acts by binding to the naturally circulating anticoagulant antithrombin II
A. I only
B. II and III only
C. I and III only
D. I, II and III
E. I only
F.

52. Which of the following is/are true regarding warfarin?


I. Acts as vitamin K agonist
II. A rapid-acting anticoagulant that is effective only when administered parenterally
III. Rapidly and completely absorbed in the upper gastrointestinal tract
A. I only
B. II and III only
C. I and III only
D. I, II and III
E. I only

53. Which of the following is/are true regarding prothrombin Time (PT)?
I. Prolonged by deficiencies of clotting factors II, V, VII and X
II. Reflects alterations in the intrinsic and common pathways of the clotting cascade
III. The internationally recognized standard for monitoring warfarin therapy when expressed
as INR
A. I only
B. II and III only
C. I and III only
D. I, II and III
E. I only

54. Which of the following is/are true regarding Activated Partial Thrombopalstin time (aPTT)?
I. Reflects alterations in the extrinsic and common pathways of clotting cascade
II. Used to monitor heparin therapy
III. Performed by adding a surface-activating agent, a partial thromboplastin reagent, and
calcium to the plasma sample
A. I only
B. II and III only
C. I and III only
D. I, II and III
E. I only

55. A clinical syndrome characterized by discomfort in the chest, jaw, shoulder, back or arm
A. Atrial fibrillation
B. Angina pectoris
C. Myocardial infarction
D. Deep vein thrombosis
E. Ischemic heart disease

56. Which of the following statements describe/s accurately events in the process of atherosclerosis?
I. Extracellular lipids accumulate and undergo oxidation
II. As lesions progress, smooth muscle cells migrate proliferate and secrete large amounts
of extracellular matrix
III. The end result is the presence of an elevated plaque, which occludes the vessel lumen
A. I only
B. II and III only
C. I, II and III
D. III only
E. II only

57. A type angina which can be induced by exertion, emotional stress, or a heavy meal, and can be relieved by
rest, nitroglycerin, or both
A. Prinzmetal’s angina
B. Angina decubitus
C. Unstable angina
D. Stable or classic angina
E. Preinfarction angina

58. A type of angina precipitated by coronary artery spasm that reduces blood flow, and usually occurs at rest
rather than with exertion or emotional stress
A. Prinzmetal’s angina
B. Angina decubitus
C. Unstable angina
D. Stable or classic angina
E. Preinfarction angina

59. Which of the following is/are true regarding the mechanism of action of nitrates?
I. Causes venous dilation, which reduces left ventricular volume and myocardial wall
tension, decreasing oxygen requirements
II. Reduces arteriolar resistance, helping to reduce afterload, which decreases myocardial
oxygen demand
III. Facilitates collateral circulation by reducing pressure in cardiac tissues
A. I only
B. II and III only
C. I and III only
D. I, II and III
E. III only

60. Nitrate tolerance is one major problem with the long-term use of nitroglycerin and long-acting nitrates. Which
of the following drugs have shown to be capable of reversing this phenomenon?
I. Acetylcysteine
II. Beta blockers
III. Ace inhibitors
IV. Diuretics
A. I and II only
B. II and III only
C. II, III and IV only
D. I, III and IV only
E. I, II, III and IV

61. Considered drug of choice in treatment of angina at rest


A. Nitroglycerin
B. Calcium-channel blockers
C. ACE inhibitors
D. Beta blockers
E. Clopidogrel
62. Used for the same arrhythmias for which quinidine is given, but possess added concern when used
intravenously because of increased cardiovascular effects such as hypotrension syncope and myocardial
depression.
A. Procainamide
B. Disopyramide
C. Tocainamide
D. Mexiletine
E. Flecainide

63. Most commonly used to treat digitalis-induced ventricular and supraventricular arrhythmias
A. Propafenone
B. Flecainide
C. Phenytoin
D. Quinidine
E. Propanolol

64. Class I anti-arrhythmias work by blocking the rapid inward sodium current and thereby slow sown the rate of
rise of the cardiac tissue’s action potential. Which sub classification moderately reduces the depolarization
rate and prolong repolarization?
A. Class IA
B. Class IB
C. classic
D. class ID
E. Class IA and IB

65. Which of the following is/are not associated with cinchonism caused by quinidine?
I. Hearing loss
II. Photophobia
III. Tinnitus
IV. Blurred vision
A. I and II only
B. III and IV only
C. I, II and III only
D. I, III and IV only
E. II, III and IV only

66. Among the type III antiarrhythmics listed below, which has been reported as causing Torsades de pointes
type of ventricular tachycardia?
A. Sotalol
B. Flecainaide
C. Amiodarone
D. Quinidine
E. Lidocaine

67. A class I antiarrhythmic drug that can causes fatigue, arthralgia, myalgia, and low-grade fever, suggestive of
systemic lupus erythematosus like syndrome
A. Flecainide
B. Disopyramide
C. Mexilitine
D. Procainamide
E. Quinolone

68. A class IV antiarrhythmic that is primarily indicated for the treatment of supraventricular tachyarrhythmias
A. Verapamil
B. Disopyramide
C. Mexilitine
D. Procainamide
E. Ibutilide

69. The New York heart association functional limitation class indicates that the degree of effort necessary to
elicit Heart failure symptoms is equal to those that would limit normal individuals
A. Class I
B. Class II
C. Class III
D. Class IV
E. Class V

70. The New York heart association functional limitation class that indicates that the degree of effort necessary
to elicit Heart failure symptoms occurs with less than ordinary exertion
A. Class I
B. Class II
C. Class III
D. Class IV
E. Class V

71. The New York heart association functional limitation class that indicates that the degree of effort necessary
to elicit Heart failure symptoms occurs while at rest
A. Class I
B. Class II
C. Class III
D. Class IV
E. Class V

72. Refers to the for that exerted on the ventricular muscle at the end of diastole that determines the degree of
muscle fiber stretch
A. Frank-string mechanism
B. Afterload
C. Cardiac output
D. Preload
E. Decompensation

73. Determined by the amount of force needed to overcome pressure in the aorta
A. Frank-string mechanism
B. Afterload
C. Cardiac output
D. Preload
E. Decompensation

74. Which of the following is/are true regarding precautions with the use of digoxin
I. Increased potassium levels favor digoxin binding to cardiac cells and increase its effect
II. Increased calcium levels increase the force of myocardial contraction
III. Magnesium levels are inversely related to digoxin activity
A. I only
B. II only
C. I and II only
D. I and III only
E. II and III only

75. Which of the following dosages of dopamine is selected for its positive inotropic effects intreating the patient
with Heart failure?
A. 40 mg/kg/min
B. 40 mcg/kg/min
C. 10-20 mcg/kg/min
D. 5-10 mcg/kg/min
E. 2 mcg/kg/min

76. A loop diuretic that is used as a rapid-acting intravenous agent in reversing acute pulmonary edema
A. Ethacrynic acid
B. Furosemide
C. Spironolactone
D. Bumetinide
E. Mannitol

77. Diuretics which have become preferred due to their ability to increase sodium excretion by 20-25% of the
filtered load and to maintain their efficacy until renal function is severly impaired
I. Spironolactone
II. Ethacrynic acid
III. Furosemide
IV. Bumetanide
A. I and II only
B. II and III only
C. I, II and III only
D. II, III and IV only
E. I, III and IV only

78. Which of the following may be used in the acute management of digitalis toxicity?
I. Potassium
II. Cholestyramine resin
III. Fab fragment antibody
A. I only
B. II only
C. I and II only
D. I and III only
E. I, II and III

79. Which of the following best describes isoniazid?


A. May be nephrotoxic and ototoxic
B. Has limited use due to its hepatotoxic potential
C. Requires pyridoxine supplementation
D. May discolor the tears and urine orange re
E. A pyrazine analogue of nicotinamide

80. Which of the following may be treated with streptomycin


I. Bacterial endocarditis
II. Tularemia
III. Urinary tract infection
IV. Tuberculosis
A. I, II and III only
B. II, III and IV only
C. I and II only
D. II and IV only
E. III and IV only

81. Which of the following best illustrates the relative ototoxicity of aminoglycosides?
A. Streptomycin=kanamycin>amikacin=gentamicin
B. Kanamycin>streptomycin=tobramycin>amikacin
C. Streptomycin=gentamicin>kanamycin=amikacin
D. Kanamycin>streptomycin>kanamycin>amikacin
E. Streptomycin>kanamycin>amikacin>gentamicin

82. Which of the following aminoglycosides can cause primarily vestibular damage?
A. Neomycin and amikacin
B. Gentamicin and neomycin
C. Kanamycin and streptomycin
D. Amikacin and streptomycin
E. Amikacin and gentamicin

83. Which of the following aminoglycosides can cause mainly auditory damage?
A. Gentamicin and streptomycin
B. Streptomycin and neomycin
C. Amikacin and kanamycin
D. Kanamycin and streptomycin
E. Amikacin and gentamicin

84. Which is the least nephrotoxic aminoglycoside?


A. Streptomycin
B. Tobramycin
C. Neomycin
D. Amikacin
E. Netilmicin

85. Which is the most nephrotoxic aminoglycoside?


A. Streptomycin
B. Tobramycin
C. Neomycin
D. Amikacin
E. Netilmicin

86. Which carbapenem needs to be combined with cilastatin sodium to inhibit the action of renal dipeptidases to
this drug?
A. Aztreonam
B. Meropenem
C. Carbamazepine
D. Ertapenem
E. Imipenem

87. Which of the following is/are the routes of administration of Penicillin G sodium and potassium?
I. Intradermal
II. Intravenous
III. Intramuscular
IV. Oral
A. II, III and IV only
B. I, III and IV only
C. II and III only
D. IV only
E. II and IV only

88. Anaphylaxis is a life threatening reaction that most commonly occurs with parenteral administration of
penicillin. The following are its sign and symptoms:
I. Nausea
II. Bronchoconstriction
III. Abdominal pain
IV. Severe hypertension
A. I only
B. I and II only
C. I, II and III only
D. I, II and IV only
E. II only

89. With which antibiotics does antibiotic antagonism occur when given 1 hour of penicillin administration?
A. Tetracycline and cephalosporins
B. Aminoglycosides and macrolides
C. Carbapenems and vancomycin
D. Erythromycin and chloramphenicol
E. Carbenicillin and isoniazid

90. An extended spectrum penicillin that is 10 times as active as carbenicillin against Pseudomonas organisms?
A. Carbenicillin
B. Piperacillin
C. Ticarcillin
D. Mezlocillin
E. Tazobactam

91. Which of the following are manifestations of complex partial seizures?


I. Glassy stare
II. Visual, auditory, or olfactory hallucinations
III. Automation
A. I only
B. I and II only
C. I and III only
D. I, II and III only
E. III only

92. Seizures that present as alteration of consciousness lasting 10-30 seconds


A. Generalized seizures
B. Petit mal seizures
C. Myocolnic seizures
D. Tonic-clonic seizures
E. Complex partial seizures

93. In a grand mal seizure, which of the following may mark the clonic phase?
A. Contraction of the diaphragm
B. Aching of the back
C. Nausea
D. Muscle jerking and flaccidity of the muscle
E. Soreness

94. Which of the following is/are true regarding the tonic phase of a grand mal seizure?
I. The individual becomes rigid and falls
II. Respiration are interrupted
III. The tonic phase lasts for about a minute
IV. The individual may experience confusion and disorientation
A. I only
B. II only
C. I, II, III and IV
D. I, II, III only
E. I, II, IV only

95. Which of the following may comprise the postical phase of generalized tonic-clonic seizure?
I. Heavy salivation
II. Incontinence
III. Tachycardia
IV. Drowsiness
A. I, II, III and IV
B. I, II and III only
C. I and II only
D. I, II and IV only
E. IV only

96. Which of the following is/are true regarding the major phases of seizure activity?
I. The ictal phase may precede the seizure by hours or days depending on the changes in
the behavior or mood of the patient
II. The proximal phase may include a subjective sensation, lethargy and confusion
III. The ictal phase is the seizure itself, and its onset is heralded by a scream or cry
A. I only
B. II only
C. II and III only
D. I and II only
E. I, II and III

97. Measures the electrical activity of the brain, and helps to identify cerebral changes underlying structural
abnormalities
A. CBC
B. EEG
C. MRI
D. PET
E. CSF

98. Which of the following is/are true regarding neurological imaging studies for seizures?
I. The MRI can detect brain lesions related to epilepsy
II. The PET measures the electrical activity of the brain
III. EEG offer functional views of the brain to detect hypermetabolism or relative hypoperfusion
A. I only
B. I and II only
C. I and III only
D. I, II and III
E. II only

99. Which of the following anticonvulsants is contraindicated in patients with a history of tricyclic antidepressants
hypersensitivity?
A. Ethosuximide
B. Carbamazepine
C. Phenobarbital
D. Phenytoin
E. Gabapentin

100. What are the most common adverse effect of anticonvulsive drugs?
I. Alternation in cognition and mentation
II. Headaches and dizziness
III. Gastointestinal symptoms
A. II only
B. Carbamazepine
C. Phenobarbital
D. Phenytoin
E. Gabapentin

101. Which of the following drugs may increase the level of carbamazepine due to its ability to decrease
carbamazepine metabolism?
A. Valproic acid
B. Primidone
C. Phenytoin
D. Phenobarbital
E. Topiramate

102. Which of the following is/are known side effects of phenytoin?


I. Stevens-Johnson syndrome
II. Gingival hyperplasia
III. Ataxia
IV. Lupus erythematosus
A. II and III only
B. I, II and III
C. II and IV only
D. II only
E. I, II, III and IV
103. Which of the following anticonvulsive drugs has higher incidence of kidney stones?
A. Phenytion
B. Ethosuximide
C. Topiramate
D. Carbamazepine
E. Phenobarbital
104. A slowly progressive degenerative neurological disease characterized by tremors, rigidity, bradykinesia
and postural instability
A. Cerebral palsy
B. Multiple sclerosis
C. Schizophrenia
D. Parkinson disease
E. Disease epilepsy

105. Which of the following is/are true regarding Parkinson’s disease?


I. It is neurodegenerative disease associated with depigmentation of substantia nigra
II. It involves surplus of dopaminergic input to the basal ganglia
III. There is an imbalance between dopamine and acetylcholine
A. I only
B. I and II only
C. I and III only
D. II and III only
E. I, II and III

106. Which of the following drugs may cause secondary parkinsonism?


I. Chlorpromazine
II. Carbamazepine
III. Perphenazine
IV. Haloperidol
A. I, II and III only
B. II and IV only
C. I, III and IV only
D. IV only
E. I and III only

107. Refers to slowness in performing common voluntary movements, including standing, walking, writing
and talking
A. Bradykinesia
B. Dystonia
C. Akinesia
D. Ataxia
E. Dyskinesia

108. Which of the following is/are true regarding selegiline?


I. Selective inhibitor of MAO type A, which prevents the breakdown of dopamine selectivity
in the brain
II. Most commonly used as an adjunct with levodopa/carbidopa when patients experience a
“wearing off” phenomenon
III. Most patients experience side effects at doses higher than 30-40 mg/day
A. I only
B. I, II and III only
C. I and III only
D. II only
E. II and III only

109. Failure to adequately minimize severe and long term airway inflammation in asthma may result in this
kind of change
A. Airway remodeling
B. Lung remodeling
C. Respiratory remodeling
D. Bronchial reconstruction
E. Respiratory arrest

110. Hyperactivity in asthmatic patients ca be measured in a physician’s office by having them perform one
of the following activity/ies
I. Inhale nebulized histamine
II. Inhale nebulized nitric oxide
III. Treadmill
IV. Inhale nebulized methacoline
A. I only
B. I, II and III only
C. II, III and IV only
D. II and III only
E. I, III and IV only

111. According to NIH Guidelines, the classification of asthma is arranged according to increasing severity is:
I. Intermittent
II. Mild persistent
III. Moderate persistent
IV. Severe persistent
A. I only
B. II only
C. I, II and III only
D. I, II, III and IV
E. I and II only

112. What are the most common adverse effect of anticonvulsive drugs?
I. Decreased vital Capacity
II. Increased residue volume
III. Decreased total lung capacity
IV. Normal total lung capacity
A. I and IV only
B. I, II and IV only
C. II, III and IV only
D. I, II, III and IV
E. I, II and III

113. Agent/s that inhibit smooth muscle contractions in asthmatic patients is/are
I. Beta-adrenergic
II. Methylxanthines
III. Glucocorticoids
IV. Anticholinergics
A. I and IV
B. I, II and IV
C. II, III and IV
D. I, II, III and IV
E. I, II and III

114. These agents are called “quick relief medications” for asthmatic patients
I. Beta-adrenergic
II. Methylxanthines
III. Glucocorticoids
IV. Anticholinergics
A. I and IV
B. I, II and IV
C. II, III and IV
D. I, II, III and IV
E. I, II and III

115. These agents are called “long-term controlled medications” for asthmatic patients
I. Beta-adrenergic
II. Leukotriene inhibitors
III. Glucocorticoids
IV. Receptor antagonists
A. I and IV
B. I, II and IV
C. II, III and IV
D. I, II, III and IV
E. I, II and III

116. These agents are analogues and produce airway dilation through stimulation of beta-adrenergic
receptors activation of G proteins with the resultant formation of cyclic adenosine monophosphate (AMP)
I. Catecholamines
II. Resorcinols
III. Saligenins
IV. Methylxanthines
A. I and IV
B. I, II and IV
C. II, III and IV
D. I, II, III and IV
E. I, II and III
117. This agent inhibit the degranulation of mast cells in asthmatic patients, thereby preventing the release of
the chemical mediators of anaphylaxis
A. Theophylline
B. Albuterol
C. Cromolyn sodium
D. Catecholamines
E. Saligenins

118. Opiates, sedatives and tranquilizers should be absolutely avoided in the acutely ill patient with asthma
because of
A. Depressing alveolar ventilation
B. Bronchospasm
C. Depressing tracheal ventilation
D. Bronchoconstriction
E. Seizures

119. What is the difference of chronic bronchitis, a primary contributor to COPD, in airway inflammation to
that asthma?
I. Slower to develop and is nonspecific
II. Predominance of IL-8
III. Relative absence of IL-5 and eosinophils
IV. Predominance of type 1 helper (CD8) T cells
A. I and IV only
B. I, II and IV
C. II, III and IV
D. I, II, III and IV
E. I, II and III

120. The primary contributor/s to COPD is/are


A. Asthma
B. Chronic bronchitis
C. Emphysema
D. Asthma and chronic bronchitis
E. Chronic bronchitis and emphysema

121. It is primary initial intervention in the management of COPD


A. Proper diet
B. Exercise
C. Smoking cessation
D. Breathing regimen
E. Pulmonary transplant

122. The primary pharmacologic therapy used in the management of COPD


I. Beta agonist
II. Anticholinergics
III. Methylxanthines
IV. Theophylline
A. I and IV only
B. I, II and IV
C. II, III and IV
D. I, II, III and IV
E. I, II and III

123. Acute renal failure is characterized clinically by an abrupt decrease in renal function over a period of
hours to days, one of the effects is azotemia. What is azotemia?
A. Inability to maintain and regulate fluid, electrolyte and acid-base balance
B. Inability to control the flow of urine output
C. Accumulation of nitrogenous waste products
D. Accumulation of oxalates in the nephrons
E. Decrease in renal threshold

124. Acute renal failure incidence is principally observed in


A. Home-cared patients
B. Hospitalized patients
C. Community dwellers
D. City dwellers
E. Provincial residents

125. The oliguric phase in acute renal failure is characterized by a/an


A. Progressive decrease in urine output and may last from days to several weeks
B. Increased urine production over several days
C. Occurrence of several weeks to months and the normalization of kidney function
D. Progressive decrease in urine reabsorptive capacity of the kidney for several days
E. Increased urine blood flow for a week

126. The diuretic phase in acute renal failure is characterized by a/an


A. Progressive decrease in urine output and may last from days to several weeks
B. Increased urine production over several days
C. Occurrence of several weeks to months and the normalization of kidney function
D. Progressive decrease in urine reabsorptive capacity of the kidney for several days
E. Increased urine blood flow for a week

127. Which of the following is recommended bronchodilator therapy for stage II COPD?
A. Beta 2 agonist
B. Anticholinergic and beta 2 agonist
C. Long acting beta 2 antagonist
D. Sustained release theophylline
E. Long acting beta 2 agonist and sustained release theophylline

128. Which of the following is recommended bronchodilator therapy for stage I COPD?
A. Beta 2 agonist
B. Anticholinergic and beta 2 agonist
C. Long acting beta 2 antagonist
D. Sustained release theophylline
E. Long acting beta 2 agonist and sustained release theophylline

129. Which of the following is recommended bronchodilator therapy for stage III COPD?
A. Beta 2 agonist
B. Anticholinergic and beta 2 agonist
C. Long acting beta 2 antagonist
D. Sustained release theophylline
E. Long acting beta 2 agonist and sustained release theophylline

130. Which of the following is/are the mechanism/s by which NSAIDs induce mucosal injury seen during
peptic ulcer disease?
I. Interruption of prostaglandin synthesis
II. Intracellular of NSAIDs in an ionized form
III. Alteration of the surface mucosal layer, permitting back diffusion of H and pepsin
IV. Extracellular entrapment of NSAIDs in an ionized form
A. I and IV
B. I, II and IV
C. II, III and IV
D. I, II, III and IV
E. I, II and III

131. Prevpac, an anti-Helicobacter pylori regimen available in prepackaged formulation include:


A. Lansoprazole, tetracycline and metronidazole
B. Bismuth subsalicylate, tetracycline and metronidazole
C. Lansoprazole, tetracycline and amoxicillin
D. Bismuth subsalicylate, clarithromycin and amoxicillin
E. Lansoprazole, clarithromycin and amoxicillin

132. Helidac, an anti-Helicobacter pylori regimen available in prepackaged formulation include:


A. Lansoprazole, tetracycline and metronidazole
B. Bismuth subsalicylate, tetracycline and metronidazole
C. Lansoprazole, tetracycline and amoxicillin
D. Bismuth subsalicylate, clarithromycin and amoxicillin
E. Lansoprazole, clarithromycin and amoxicillin

133. Mechanism/s by which prostaglandins maintain gastric epithelial integrity


I. Regulate the release of mucosal bicarbonate and mucus
II. Inhibit parietal cell secretion
III. Maintain mucosal blood flow and epithelial cell restitution
A. I and IV
B. I, II and IV
C. II, III and IV
D. I, II, III and IV
E. I, II and III

134. Heart burn, commonly described as a pain in the center of the chest is a classic symptom of
A. Cardiac angina
B. Asthma
C. GERD
D. Zollinger-ellison syndrome
E. Chronic obstructive pulmonary disease

135. What is the reasonable way of administering the Proton pump inhibitor, omeprazole, with NG tubes?
A. Placed directly into the NG tube
B. Crushed and mixed with water to make a slurry/ suspension
C. Dissolution of the gelatin capsule with sodium bicarbonate injection
D. Suspension of the enteric coated beads in sodium bicarbonate injection
E. Crushed and mixed with sodium bicarbonate to make a slurry suspension koda-kimbe 25-19

136. What is/are the joint changes observed in rheumatoid arthritis?


I. Loosening of tendon sheath and other periarticular structures
II. Synovial thickening, leading to pannus formation
III. Joint space widening, leading to bone erosion and osteoporosis
IV. Erosion of articular surfaces
A. I and IV
B. I, II and IV
C. II, III and IV
D. I, II, III and IV
E. I, II and III

137. These agents preventor or slow joint or slow joint destruction in rheumatic arthritis
I. NSAIDs
II. Azathioprine
III. D-Penicillamine
IV. Hydroxychloroquine
A. I and IV
B. I, II and IV
C. II, III and IV
D. I, II, III and IV
E. I, II and III

138. In the synovium of the rheumatoid arthritis patients, the balance of cytokinesis is heavily weighted
toward the proinflammatory cytokines, leading to sustained inflammation and tissue destruction.
Proinflammatory cytokines include/s
I. IL-1
II. IL-Ra
III. IL-10
IV. IL-11
A. I only
B. I and IV
C. II and IV
D. I, III and IV
E. I, II and III

139. This anti-inflammatory drug for rheumatoid arthritis should only be administered for a brief period due to
serious adverse effects such as osteoporosis, cataract formation and impaired wound healing on long-term
use
A. Aspirin
B. Prednisone
C. Sulfasalazine
D. Infliximab
E. Gold

140. Synovial joint consist of two bone ends covered by articular cartilages. This cartilage can be
characterized to be
I. Avascular and aneural
II. Metabolically active and undergone continual internal remodeling
III. Incharge of the production of proteoglycans and collagen in the extracellular matrix
IV. A membrane that produces a viscous fluid that lubricates the joint
A. I only
B. I, II and IV
C. II only
D. I, II, III and IV
E. II and I

141. This is a hallmark of the pathophysiology of rheumatoid arthritis


A. Generic marker such as HLA-DR4
B. Bacterial and viral infection
C. Inflammed synovium
D. Prostaglandin of TNF-a, IL-1, IL-6
E. Chonrocytes degradation

142. Classified as a Disease-Modifying Antirheumatic Drug, it indicates as amonotherapy for rheumatoid


arthritis and necessities liver function monitoring
A. Etanercept
B. Infliximab
C. Methotrexate
D. Leflunomide
E. Sulfasalazine

143. Neds, a 51 yo postal office clerk was diagnosed with osteoarthritis, the nonpharmacological treatment
most likely advised to her is/are
I. Aerobic exercise
II. Use supportive mechanism e.g. cane
III. Weight loss
IV. Physical therapy
A. I only
B. I, II and III
C. II only
D. I, II, III and IV
E. I, II and III

144. In the patients diagnosed with gout, it is expected that this serum acid level rises
A. Picric
B. Lactic
C. Uric
D. Dipicolinic
E. Glycolic

145. The following drug/s is/are contraindicated to patients with gout and hyperuricemia since it/they
increase/s uric acid production
I. Ethambutol
II. Salicylates
III. Diuretic
IV. Acetaminophen
A. I only
B. I, II and IV
C. II only
D. I, II and III
E. II and IV

146. If gout is left untreated eventual joint deformity and disability may be observed. This occurrence is due to
tophi or tophaceous deposit. Tophi is defined as
A. Remains of degraded urate crystals
B. Deposits of monosodium urate crystals
C. Deposits of disodium urate crystals
D. Precipitate of dipotassium urate crystals

147. Assymptomatic hyperuricemia is characterized by increased serum uric acid level but has no signs or
symptoms of deposit disease. The logical primary intervention would be
A. Community intervention
B. Pharmacological intervention
C. Supportive intervention
D. Pharmaceutical intervention
E. Nutritional (increased purine rich food) intervention

148. Urate-lowering drugs should be administered for acute gouty arthritis when
A. Acute attack is controlled
B. Acute attack has not yet occurred as prophylaxis
C. An acute attack is transpiring
D. An acute attack just had occurred
E. In concomitant use with proinflammatory agents

149. The greatest potential risks of therapy with uricosuric drugs are the formation of uric acid crystals in
urine and the deposition of uric acid in the renal tubules, pelvis, or ureter. To reduce this risk, patients are
advised to do the following activities:
I. Maintain high fluid intake (2L/day)
II. Maintain high fluid intake (1.5L.day)
III. Take 1 g of NaHCO3, 3-4 times a day
IV. Take 1 g of NahCO3, 2-3 times a day
A. I and IV
B. II and III
C. II and IV
D. I only
E. I and III
150. Sarcomas are malignant tumors of the
A. Epithelial cells
B. Lymphoid
C. Nephron
D. Schwann cells
E. Lung cancer

151. The tumor marker Carcinoembryonic antigen (CEA) is commonly seen in


A. Hepatocellular carcinoma
B. Colorectal cancer
C. Prostate cancer
D. Breast cancer
E. Lung cancer

152. Cyclophosphamide is an alkylating chemotherapeutic agent where they generally form covalent DNA
adducts. The following should be consider for cyclophosphomide regimen
I. Absence of liver disease
II. Coadministartion with mercaptoethanesulfonate
III. Maintenance of excellent hydration during therpy
IV. Control of purine-rich food
A. I only
B. I, II and IV
C. II only
D. I, II and III
E. II and IV

153. This antidopaminergic phenothiazine drug can be administered rectally and acts directly at the
chemoreceptor trigger zone (CTZ)
A. Haloperidol
B. Lorazepam
C. Prochlorperazine
D. Thiethylperazine
E. ondasentron

154. IV catheters are commonly used in cancer therapy and are prone to infections. When is it advised to
remove the catheter?
I. If the patient has a “tunneled” catheter with a red streak over the subcutaneous part of
the tunnel
II. If it is an exit-site infection
III. If it is coagulase-positive staphylococcal infection
IV. If infection is due to P. aeruginosa and Candida species
A. I only
B. I, II and IV
C. II only
D. I, II and III
E. I, III and IV

155. Metformin is used in the management of diabetes mellitus. Patients receiving it should be monitoredfor
development of
A. Lactic acidosis
B. Tinnitus
C. Agranulocytosis
D. Hirsutism
E. Parakeratosis

156. Which of the following is an effect associated with the use of pilocarpine ophthalmic products?
I. Mydriasis
II. Cholinergic agonism
III. Papillary constriction
A. I and II only
B. II and III only
C. III only
D. I only
E. I, II and III

157. Psoriasis is one of the most common dermatologic disease. It best characterized by
A. Siler micaceous scale
B. Polygonal, flat-topped papules
C. Small red vesicles
D. Granulomas lesions
E. Water-filled blister

158. Which of the following statement is/are true regarding candidiasis?


I. It is a fungal infection caused by a related group of yeast
II. It may be systemic and life-threatening
III. It is very common among HIV-infected individuals
A. I only
B. I and II only
C. I, II and III
D. II and III only
E. III only

159. The clinical hallmark of acne vulgaris


A. Papuls
B. Pustules
C. Nodules
D. Comedones
E. Lesions

160. Transient lesions that are composed of a central wheal surrounded by an erythematous halo
A. Bullae
B. Acne
C. Psoriasis
D. Eczema
E. Urticaria

161. Include tumors of connective tissue such as bone or muscle


A. Carcinomas
B. Sarcomas
C. Lymphomas
D. Leukemia
E. Hematoma

162. Include tumors of blood-forming elements and are classified as acute or chronic, myeloid or lymphoid
A. Carcinomas
B. Sarcomas
C. Lymphomas
D. Leukemia
E. Hematoma

163. The stage in carcinogenesis wherein the environment becomes altered to allow preferential growth of
mutated cells over normal cells
A. Promotion
B. Initiation
C. Progression
D. Regression
E. Metastasis

164. The first stage of carcinogenesis wherein normal cells are exposed to a carcinogen
A. Promotion
B. Initiation
C. Progression
D. Regression
E. Metastasis

165. A metabolite of cyclophosphamide and ifosfamide that is thought to cause a chemical irritation of bladder
mucosa resulting to bleeding
A. Mesna
B. Amifostine
C. Acrolein
D. Dextrazoxane
E. Histamine

166. Which of the following is/are true regarding pain?


I. An unpleasant sensory and emotional experience that is usually associated with
structural or tissue damage
II. A subjective, individual experience
III. Has physical, psychological and social determinants
A. I and II only
B. II and III only
C. I, II and III
D. III only
E. I only

167. Stimulation of mu receptors produces the characteristics narcotic effects. These morphine-like effects
incude
I. Analgesia
II. Euphoria
III. Sedation
IV. Physical dependence
A. I, II, III and IV
B. I, II and III only
C. I, II and IV only
D. I and II only
E. I and III only

168. A component of red pepper that causes release of subsatance P from sensory nerve fibers, resulting in
the prolonged cutaneous pain transmission, histamine release, and erythema because of reflex
vasodilatation
A. Glucosamine sulfate
B. Chondroitin
C. Capsaicin
D. Capsicum oleoresin
E. Cycooxygenase

169. Natural or synthetic polysaccharide derivatives that adsorb water to soften the stool and increase the
bulk
A. Saline laxatives
B. Osmotic laxatives
C. Emollient laxatives
D. Bulk-forming laxatives
E. Stimulant laxatives

170. Natural or synthetic polysaccharide derivatives that adsorb water to soften the stool and increase the
bulk
A. Saline laxatives
B. Osmotic laxatives
C. Emollient laxatives
D. Bulk-forming laxatives
E. Stimulant laxatives

171. Laxatives that act as surfactants by allowing absorption of water into the stool, which makes the
softened stool easier pass
A. Saline laxatives
B. Osmotic laxatives
C. Emollient laxatives
D. Bulk-forming laxatives
E. Stimulant laxatives

172. Clusters of dilated blood vessels in the lower rectum or anus


A. Anal abscess
B. Cryptitis
C. Anal fissure
D. Polyps
E. Hemorrhoids

173. Which of the following statements adequately describes bulk-forming laxatives


A. Can cause diarrhea if not taken with water
B. Onset of action is in 4-8 hours
C. Produce much more complete evacuation of constipation than stimulant products
D. Are derived from polysaccharides and resemble fiber in mechanism of action
E. All of its kind can cause abdominal cramping

174. Which of the following is/are factor/s for hyperphosphatemia and death from sodium phosphate enema?
I. Renal insufficiency
II. Hirschprung’s disease
III. Anorectal malformations
A. I and II only
B. II and III only
C. I, II and III
D. III only
E. I only

175. Which of the following is/are true regarding lactulose?


I. An osmotic laxative
II. May cause flatulence and cramping
III. Should be taken with fruit juice or milk to increase its palatability
A. I and II only
B. II and III only
C. I, II and III
D. III only
E. II only

176. A condition characterized in a poorly functioning colon and resembles the symptoms of ulcerative colitis,
caused by chronic use of stimulant laxatives
A. Melanosis coli
B. Cathartic colon
C. Lipid pneumonitis
D. Laxative abuse
E. Hyperphosphatemia

177. A dark pigmentation of the colonic mucosa that results from long-term use of anthraquinone laxatives
A. Melanosis coli
B. Cathartic colon
C. Lipid pneumonitis
D. Laxative abuse
E. Hyperphosphatemia

178. Adverse effects of metoclopramide limit its usefulness. These adverse effects include
I. Insomnia
II. Depression
III. Extrapyramidal effects
A. I and II only
B. II and III only
C. II only
D. I, II and III
E. III only

179. Which of the following statements is/are true regarding emollient laxatives?
I. Not good for acute constipation
II. More effective than placebo for long term use
III. Good for patients who should not strain by passing a hard stool
A. I, II and III
B. I and III only
C. I and II only
D. II and III only
E. I only

180. The biological active thyroid hormone produced by the thyroid gland that is four times more potent than
the other is:
A. Triiodothyronine
B. Thyroxine
C. Thyroid stimulating hormone
D. TRH
E. A and B

181. It is the hyper metabolic syndrome that occurs form excessive production of thyroid hormone with classic
symptoms of weight loss despite increased appetite, nervousness, palpitations, heat tolerance and insomnia
A. Hypothyroidism
B. Hashimoto’s disease
C. Thyrotoxicosis
D. Graves disease
E. Cushing syndrome

182. Thyroid preparations that are used to treat hypothyroidism


I. L-thyroxine
II. PTU
III. Methimazole
IV. Liothrix
A. I and II only
B. I, II and III
C. II and III only
D. I and IV only
E. I, II, III, and IV

183. In this condition, there is a loss of insulin secretion that results from autoimmune destruction of the
insulin-producing B-cells in the pancreas
A. Type 2 DM
B. Type 1 DM
C. Diabetes insipidus
D. Gestational diabetes
E. Type 3 DM

184. It is a condition defined as any carbohydrate intolerance with onset of the first recognition during
pregnancy and its duration affect the prognosis for a good obstetric and perinatal outcome
A. Juvenile-onset DM
B. Adult-onset DM
C. Gestational DM
D. Obese type 2 DM
E. Pediatric DM

185. The normal fasting plasma glucose levels (FPG)


A. <140 mg/dL (7.8 mmol/L)
B. <100 mg/ dL (6.1 mmol/L)
C. <126 mg/dL (7.0 mmol/L)
D. 110-125 mg/dL (6.1-7.0 mmol/dL)
E. >200 mg/dL (11mmol/L)

186. The classical signs of diabetes Mellitus include the following: polydipsia, polyuria, ketonuria, rapid weight
loss, combined with random plasma glucose levels of:
A. ≥200 mg/dL
B. 105 mg/dL
C. 190 mg/dL
D. 140 mg/dL
E. ≥100 mg/dL

187. It is a rapid-acting insulin preparation that has a 4-5 hours duration of action
A. Insulin glargine
B. Insulin lispro
C. NPH
D. Lente (Insulin Zinc Suspension)
E. RAI

188. It is a condition where blood glucose concentration is < 50mg/dL and patient may be asymptomatic or a
blood glucose of < 40mg/dL where patients suffers from blurred vision, sweaty palms, generalized sweating,
tremors, hunger, confusion, anxiety and circumoral tingling and numbness and that severe condition can
lead to seizure and coma.
A. Hyperglycemia
B. Hyperuricemia
C. Ketoacidosis
D. Hypoglycemia
E. Diabetes insipidus

189. It reversibly inhibits a variety of enzymes present in the brush-border of the mucosa of the small intestine
that are responsible for the breakdown of complex polysaccharide and sucrose into absorbable
monosaccharides
A. Metformin
B. Acarbose
C. Miglitol
D. B and C
E. A and B

190. First generation sulfonylureas


I. Tolbutamide
II. Chlorpropamide
III. Acetohexamide
IV. Glipizide
A. I, II and III
B. I and II only
C. I only
D. I and III only
E. II only

191. A condition characterized by an increase in intraocular pressure (IOP) and is influenced by the
production of aqueous humor by the ciliary processes and the outflow of aqueous humor through the
trabelar meshwork
A. Glaucoma
B. Stye
C. Conjunctivitis
D. Corneal ulcers
E. Diplopia

192. An IOP greater than 21 mm Hg, normal visual fields, normal optic discs, open angles, and the absence
of any ocular disease contributing to the elevation of IOP
A. Angle-closure glaucoma
B. Open-angle glaucoma
C. Ocular hypertension
D. Primary open angle glaucoma
E. Diplopia

193. In this condition, aqueous humor outflow from the anterior chamber is constantly subnormal primarily
because of a degenerative process in the trabecular meshwork
A. Angle-closure glaucoma
B. Ocular hypertension
C. Primary open angle glaucoma
D. Diplopia
E. Corneal ulcers

194. It is a parasympathetic agent that causes contraction of ciliary muscle fibers attached to the trabecular
meshwork and sclera spur. This opens the trabecular meshwork to enhance aqueous humor outflow
A. Epinephrine
B. Pilocarpine
C. Apraclonidine
D. Levobunolol
E. Atropine

195. Are infections of the hair follicle or sebaceous glands of the eyelids commonly caused by
Staphylococcus aureus
A. Conjunctivitis
B. Hordeolum
C. Corneal ulcers
D. Cataracts
E. Comedones

196. Cholestatic jaundice is an adverse effect reported for all the salts of this antimicrobial especially for its
estolate salt
A. Tetracycline
B. Erythromycin
C. Clindamycin
D. Netilmicin
E. Polymixin

197. Associated with aplastic anemia and gray baby syndrome


A. Flucytosine
B. Ketoconazole
C. Quinolones
D. Chloramphenicol
E. Salicylates

198. Type of pain that usually last less than 30 days and occurs following muscle strains and tissue injury
A. Breakthrough pain
B. Chronic pain
C. Acute pain
D. Chronic non-malignant pain
E. Chronic cancer pain

199. Type of pain which is the intermittent, transitory increase in pain that occurs at a greater intensity over
baseline chronic pain
A. Acute pain
B. Chronic malignant pain
C. Chronic non-malignant pain
D. Breakthrough pain
E. Chronic cancer pain

200. Potential adverse effects of aspirin include


I. Gastrointestinal ulceration
II. Hypersensitivity asthma
III. Cardiac arrhythmias
A. I and II only
B. II only
C. III only
D. I and III only
E. I, II and III

201. If the hospital pharmacy is decentralized, each satellite pharmacy must be


A. Supervised by a single pharmacist who is knowledgeable and experienced in hospital operations
B. Supervised by a pharmacy technician or aide, in order to maximize human resources in the hospital
C. Supervised by a licensed pharmacist who reports to the director of the pharmacy
D. Supervised by pharmacy interns in order to facilitate full training experience
E. Supervised by a registered nurse if there is no pharmacist available

202. Routine inspections must be done in all areas where medications are dispensed, administered and
stored. Documented checks are to be made to make sure that
I. Outdated and other unusable drugs are removed from locations where they could be
admitted in error
II. The use of investigational drugs conforms poorly with pharmacy policies and procedures
III. External and internal drug preparations must be stored in the same pharmacy area
A. I, II and III
B. I and II only
C. II only
D. I only
E. I and III only

203. Responsible for the development and surveillance of pharmacy and therapeutics policies and practices,
particularly drug utilization within the hospital
A. Medical director
B. Board of directors
C. Pharmacy and therapeutics committee
D. Chief pharmacist
E. Pharmaceutical services director

204. Which of the following are the recommended functions of Pharmacy and Therapeutics Committee
I. Advises the medical staff and pharmacy service on matters pertaining to the choice of
drugs available for patient care and diagnostic testing
II. Adds and deletes drugs in the hospital formulary or drug list and continually reviews it
III. Ensures constant supply of drugs by facilitating regular inventory and stocks review
A. I, II and III
B. I and II only
C. I and III only
D. II and III only
E. I only

205. By customs, acts as the secretary of the Pharmacy and Therapeutics Committee
A. Registered nurse
B. Head of the legal division of the hospital
C. Medical doctor
D. Pharmacist
E. Administrative clerk

206. When medications are dispersed to inpatients at the time of their discharge, the labels must include
I. Full name of the patient
II. Name of the drug, strength, and amount dispensed
III. Unit cost of the drug
IV. Name of the prescribing practiotioner
A. I, II and III only
B. I, II, III and IV
C. II only
D. I and II only
E. I, II and IV only

207. Refers to a define course or method of action to guide and determine present and future decisions
A. Framework
B. Policy
C. Operations
D. Procedure
E. Strategic planning

208. Refers to a series of steps followed in a regular definite sequence in order to accomplish something
A. Framework
B. Policy
C. Operations
D. Procedure
E. Vision-mission statement

209. Which of the following are included in the administrative information division of most policy and
procedure manual?
I. Personnel policies and procedures
II. Philosophy
III. Organizational relationships
IV. Job descriptions
A. I only
B. II only
C. I, II and III only
D. III and IV only
E. II, III and IV only

210. Which of the following is/are characteristics of the information contained in policy and procedure
manuals?
I. Current
II. Reliable
III. Static
A. I only
B. II only
C. I, II and III only
D. I and II only
E. II and III only

211. Which of the following factors mainly determine/s the type of services offered by a hospital
I. Government health policies
II. The health care services demands of the patients in the target market
III. The resources of the organization
A. I only
B. II only
C. I, II and III only
D. I- and II only
E. II and III only

212. Which of the following are components of hospital marketing?


I. Product or types of service
II. Price
III. Place promotion
A. I and II only
B. III and IV only
C. I, II and III only
D. II, III and IV only
E. I, II, III and IV

213. Refers to an amount that is above the break-even cost of providing a product or service and is
necessary to provide for replacement and upgrading of hospital equipment and facilities
A. Overhead cost
B. Revenue deductions
C. Profit margin
D. Direct personnel costs
E. Supply cost

214. Refers to the pricing method wherein the price is calculated by adding a fixed fee to the acquisition cost
of the unit of product dispensed
A. Percentage markup method
B. Dispensing fee method
C. Per diem method
D. Standard method
E. Special cost method
215. Refers to the pricing method where the patient receiving a more costly drug pays for a proportionately
larger part of overall pharmacy service costs regardless of the amount of service received
A. Percentage markup method
B. Dispensing fee method
C. Per diem method
D. Standard method
E. Special cost method

216. Refers to the pricing method wherein the average drug cost per patient day, the average pharmacy
service cost per patient day, and the desired profit margin are computed to arrive at a single pharmacy
charge for each day the patient stayed in the hospital
A. Percentage markup method
B. Dispensing fee method
C. Per diem method
D. Standard method
E. Special cost method

217. A visual display technique that can be used as a scheduling tool for the entire planning process
A. Curve fitting
B. Gant chart
C. Cyclical change
D. Regression
E. Distribution

218. A physical quantity of a drug product ordered by a prescriber to be administered to a specified patient at
one time, in ready to administer from with no further physical or chemical alterations required
A. Unit dose package
B. Single unit package
C. Unit dose
D. Multiple dose
E. Unit dose dispensing system

219. A package that contains one discrete pharmaceutical dosage form


A. Unit dose package
B. Single unit package
C. Unit dose
D. Multiple dose
E. Unit dose dispensing system

220. Refers to the act of a pharmacist in supplying one or more drug products to or for a patient, usually in
response to an order from an authorized prescriber
A. Patient counseling
B. Drug utilization review
C. Distribution
D. Compounding
E. Dispensing

221. Refers to a system that has its purpose the selection, acquisition, control, storage, dispensing, delivery,
preparation, and administration of drug product in health care institution in response of an authorized
prescriber
A. Drug distribution control system
B. Drug use information system
C. Drug distribution system
D. Drug packaging system
E. Drug availability system

222. Which of the following are considered as drug distribution control information necessary to ensure that a
unit dose package is delivered as ordered to the right patient, at the right date, and the right time?
I. A description of the unit dose ordered
II. Socio-economic statues of the patient
III. Identification and location of the patient
IV. Time and date of administration
A. I, II, III and IV
B. I, II and II only
C. I, III and IV only
D. II, III and IV only
E. I, II and IV only

223. A standard “bid” dosing schedule means that a drug must be given at
A. 7am & 7pm
B. 8am & 8pm
C. 6am & 6pm
D. 9am & 9pm
E. 10am & 10pm

224. A standard “tid” dosing schedule means that the drug must be given at
A. 7am – 12nn – 4pm
B. 8am – 1pm – 5pm
C. 10am – 2pm – 6pm
D. 7am – 11am – 3pm
E. 6am – 10am – 2 pm

225. A standard “q6h” dosing schedule means that the drug must be given at
A. 7am – 1pm – 7pm – 1am
B. 8am – 2pm – 8pm – 2am
C. 9am – 3pm – 9pm – 3am
D. 6 am – 12nn – 6pm- 12am
E. 10am – 4 pm – 10pm – 4am

226. Refers to a drug distribution system wherein a bulk supply of each drug product is maintained on the
nursing unit in advance of need and the nurse prepare the doses of administration
A. Patient prescription system
B. Floor stock
C. Emergency carts
D. Unit dose dispensing system
E. Decentralized system

227. Refers to a drug distribution system wherein all drugs are retained in the pharmacy until the order is
received, and then a multiple dose supply is dispensed to the nursing unit labeled for use by a specific
patient
A. Patient prescription system
B. Floor stock
C. Emergency carts
D. Unit dose dispensing system
E. Decentralized system

228. Factors aside from the system design that affect the total systems cost of maintaining a hospital drug
distribution system
I. Mission of the hospital
II. Bed size
III. Physical layout of hospital
IV. Enforcement of performance standards
A. I, II, III and IV
B. II and III only
C. I, II and IV only
D. I and IV only
E. I, III and IV only

229. Which of the following is/are advantage/s of unit dose dispensing systems?
I. Less drug losses due to waste and pilferage
II. Greater accuracy in inventory control, cost accounting, and charging
III. Improved ability of the hospital to attract and retain highly qualified and motivated
pharmacy personnel
A. I and II only
B. I and II only
C. I and III only
D. II only
E. I, II and III only

230. A “stat” order means


I. The drug is to be administered immediately
II. The order takes precedence over the regularly scheduled medications
III. The drug is to be administered at a specified time
A. I only
B. I and II only
C. I and III only
D. I, II and III
E. II and III only

231. Which of the following is/are true regarding subcutaneous injection administration?
I. The main areas of administration are the thigh, abdomen and the upper arm
II. The needle is inserted through the skin at a 90-degree angle
III. Rotation of the injection sites will reduce the chance of local irritation or damage
A. I, II and III only
B. I and II only
C. I and III only
D. II and III only
E. II only

232. Which of the following is/are true regarding intramuscular injection administration?
I. The usual sites of administration are the deltoid and gluteal muscles
II. 5-6 ml is usually the recommended maximal volume
III. The needle is inserted through the skin at a 45-60 degree angle
A. I, II and III only
B. I and II only
C. I and III only
D. I only
E. III only

233. Which of the following is/are true regarding intradermal injection?


I. Used to check for circulating antibodies to the injected antigen
II. Inradermal skin test is usually read at 24, 36 and 48 hours
III. A reddened area of in duration of 5 mm or greater usually indicates an intact immune
system
A. I, II and III only
B. I and II only
C. I and III only
D. I only
E. III only

234. Which of the following are indications of intravenous therapy?


I. Replacement of fluid and electrolytes
II. Parenteral nutrition
III. Administration of drugs requiring a rapid high blood level
IV. Emergency situations like cardiac arrest
A. I and II only
B. I, II and III only
C. I, II and IV only
D. II, III and V only
E. I, II, III and IV only

235. Which of the following are possible adverse effects of intravenous administration?
I. Thrombophlebitis
II. Air emboli
III. Speed shock
IV. Injection of pathogens
A. I and III only
B. I, II and III only
C. I, III and IV only
D. II, III and IV only
E. I, II, III and IV

236. The optimal pH range for aminophylline


A. Below 7.0
B. Above 7.0
C. Above 8.0
D. Below 8.0
E. Above 6.0

237. Which of the following is/are nutrition support activities of a pharmacist?


I. Catheter care
II. Nutritional assessment
III. Parenteral formulation design
IV. Metabolic monitoring of therapy
A. I and III only
B. II and III only
C. I, III and IV only
D. III and IV only
E. I, II, III and IV

238. Which of the following is/are essential human nutrients for intravenous diets?
I. Carbohydrates
II. Protein
III. Electrolytes
IV. Fats
A. I and III only
B. I, II, and III only
C. I, III, and IV only
D. II, III, and IV only
E. I, II, III and IV

239. The most important human nutrient required for tissue synthesis, repair, transport of body nutrients and
waste, and maintenance of immune function
A. Carbohydrates
B. Protein
C. Fats
D. Dextrose
E. Lactose

240. Most widely utilized carbohydrate in parenteral nutrition because of its low cost, availability, and proven
utility
A. Glycerol
B. Fructose
C. Sucrose
D. Dextrose
E. Lactose

241. Which of the following is/are mechanical tube feeding complication/s?


I. Aspiration pneumonia
II. Mucosal erosions
III. Hypertonic dehydration
IV. Nasopharyngeal irritation
A. I and II only
B. III and IV only
C. I, II and III only
D. I, II and IV only
E. I, II, III and IV

242. Which of the following is/are metabolic tube feeding complication/s?


I. Electrolyte imbalance
II. Glucose intolerance
III. Diarrhea
IV. Distention
A. I and II only
B. III and IV only
C. I, II and III
D. I, II, and IV only
E. I, II, III and IV

243. The extent to which a technique consistently measures whatever it measures regardless of the
investigator or the situation
A. Validity
B. Dependability
C. Vulnerability
D. Sensitivity
E. Reliability

244. Which of the following is/are gastrointestinal tube feeding complication/s?


I. Electrolyte imbalance
II. Glucose intolerance
III. Diarrhea
IV. Distention

A. I and II only
B. III and IV only
C. I, II and III
D. I, II, and IV only
E. I, II, III and IV

245. The extent to which a technique measures what it is intended to measure?


A. Validity
B. Dependability
C. Vulnerability
D. Sensitivity
E. Reliability

246. An authorized, structured and continuing program that reviews, analyzes and interpret patterns of the
drug usage in a given health care delivery system against pre determined standards
A. Pharmaceutical care plan
B. Therapeutic drug monitoring
C. Drug utilization review
D. Quality assurance
E. Drug auditing and inventory control

247. Pre-clinical studies that determine the new drug’s absorptive, distributive, metabolic and excretory
pathways
A. Pharmacologic studies
B. Pharmacodynamics studies
C. Toxicologic studies
D. Pharmacokinetic studies
E. Biogenetic studies

248. Pre-clinical studies that determine the action of the new drug in animals to estimate the magnitude of its
intended therapeutic effect
A. Pharmacologic studies
B. Pharmacodynamics studies
C. Toxicologic studies
D. Pharmacokinetic studies
E. Biogenetic studies

249. Pre-clinical studies in animals that determines the relative safety in humans and monitor parameters that
will be used in clinical trials
A. Pharmacologic studies
B. Pharmacodynamic studies
C. Toxicologic studies
D. Pharmacokinetic studies
E. Biogenetic studies

250. Any drug that has not yet been released for general use, and has not yet been cleared for sale in
commerce
I. Investigational drug
II. Experimental drug
III. Regulated drug
IV. New drug
A. I only
B. I and II only
C. I and III only
D. I and IV only
E. I, II and IV only

251. Various strategies and techniques are in place for use in counseling and educating patients. The
effective domain of learning process involves
I. Perceptions
II. Emotions
III. Memory
IV. Beliefs
A. I only
B. I and II only
C. I, II, and IV only
D. I, II, III only
E. IV only

252. The behavioral domain of learning process involves


I. Decision making
II. Actions
III. Physical abilities
IV. Perceptions
A. I only
B. I and II only
C. I, II and IV only
D. I, II, and III only
E. IV only

253. An order for medication issued by physician, dentist, or other properly licensed medical practitioner
A. Medication profile
B. Patient chart
C. Voucher
D. Prescription
E. Insurance policy

254. Medications that may be dispensed legally only on prescription


A. Over-the-counter drugs
B. Sample drugs
C. Imported drugs
D. Investigational drugs
E. Legend drugs

255. Prescription part generally understood to be a contraction of the Latin verb recipe, meaning take thou or
you take
A. Superscription
B. Inscription
C. Subscription
D. Signature
E. Transcription

256. The body or principal part of the prescription order t hat contains the names and the quantities of t he
prescribed ingredients
A. Superscription
B. Inscription
C. Subscription
D. Signature
E. Transcription

257. Part of the prescription that consists of directions to the pharmacist for preparing the prescription
A. Superscription
B. Inscription
C. Subscription
D. Signature
E. Transcription

258. Part of the prescription where the prescriber indicates the directions for the patient’s use of medication
A. Superscription
B. Inscription
C. Subscription
D. Signature
E. Transcription

259. They are commonly used abbreviations in prescription and medication orders. What does “aa.” mean?
A. Before meals
B. Up to
C. Left ear
D. Each ear
E. Of each

260. The abbreviation “po” means


A. After surgery
B. Rectally
C. By mouth
D. When necessary
E. After eating

261. The abbreviation “ss” means


A. After surgery
B. Rectally
C. By mouth
D. One half
E. Immediately

262. Defined as the preparation, mixing, assembling, packaging, or labeling of a drug or device as a result of
prescription-drug order
A. Pharmacy dispensing
B. Pharmacy compounding
C. Pharmacy manufacturing
D. Pharmacy quality assurance
E. Pharmacy practice
263. Use for bulk powders, large quantities of tablets or capsules, and viscous liquids that can not be poured
readily
A. Prescription bottles
B. Applicator bottles
C. Wide mouth bottles
D. Round vials
E. Sifter-top containers

264. Use for powders to be applied sprinkling


A. Prescription bottles
B. Slide boxes
C. Wide mouth bottles
D. Dropper bottles
E. Aerosol containers

265. Use for powders to be applied sprinkling


A. Prescription bottles
B. Collapsible tubes
C. Wide mouth bottles
D. Dropper bottles
E. Ointment jars

266. Any substance that may be considered a food or part of a food and provides medical or health benefits,
including prevention and treatment of disease
A. Designer food
B. Nutraceutical
C. Pharmafood
D. Phytochemical
E. Chemopreventive agent

267. Substances found in edible fruits and vegetable that may be ingested by humans daily in gram quantities
and that exhibit a potential for modulating human metabolism in a manner favorable for cancer prevention
A. Designer food
B. Nutraceutical
C. Pharmafood
D. Phytochemical
E. Chemopreventive agent

268. Nutritive or nonnutritive food component being scientifically investigated as a potential inhibitor of
carcinogenesis for primary and secondary cancer prevention
A. Designer food
B. Nutraceutical
C. Pharmafood
D. Phytochemical
E. Chemopreventive agent

269. Processed food that are supplemented with food ingredients naturally rich in disease-preventing
substances
A. Designer food
B. Nutraceutical
C. Pharmafood
D. Phytochemical
E. Chemopreventive agent

270. Which of the following are considered as categories of complementary and alternative medicine?
I. Mind-body interventions
II. Bioelectromagnetic intravenous
III. Herbal medicine
IV. Diet and nutrition
A. I and II only
B. III and IV only
C. I, II and III only
D. I, II and IV only
E. I, II, III and IV

271. An ancient Chinese healing art that employs art that employs fine needles inserted at various locations
in the body to restore the smooth flow of energy
A. Hypnosis
B. Allopathy
C. Acupuncture
D. Acupressure
E. Homeopathy

272. A therapeutic method that clinically applies the law of similar (like cures like) and uses medically active,
potentized substances at weak or infinitesimal doses
A. Aryuvedic medicine
B. Chiropractic
C. Hypnosis
D. Homeopathy
E. Allopathy

273. A state of altered consciousness, sleep or trance induced artificially in a subject by means of verbal
suggestion or by the subject concentrating upon some subject
A. Hypnosis
B. Chiropractic
C. Allopathy
D. Homeopathy
E. Iridology

274. A diagnostic tool that purports to correlate changes in the color and texture of the iris with mental and
physical disorders
A. Hypnosis
B. Chiropractic
C. Allopathy
D. Homoepathy
E. Iridology

275. Referred to a specialty practice of pharmacy that focuses upon the safe and efficacious use of
radioactive drugs
I. Nuclear pharmacy
II. Radiopharmacy
III. Institutional pharmacy
A. I only
B. II only
C. I and II only
D. I, II and III
E. I and III only

276. A drug that exhibits spontaneous disintegration of unstable nuclei with the emission of nuclear particles
or photons
A. Designer drug
B. Prohibited drug
C. Dangerous
D. Over-the-counter drug
E. Radioactive drug

277. Which of the following is/are true regarding radiopharmaceuticals?


I. Lack of pharmaceutical effects
II. Employed as tracers of physiological functions
III. Small amounts produce negligible effects on biological processes
A. II only
B. I and II only
C. II and III only
D. I, II and III
E. III only

278. A diagnostic radiopharmaceutical used for bone scans


A. Tc-99m diphosphonates
B. Tc-99m macroaggregated albumin
C. Tl-201 thalous chloride
D. Sr-89 strontium chloride
E. I-131 sodium iodide

279. A diagnostic radiopharmaceutical used for myocardial perfusion scans


A. Tc-99m diphosphonates
B. Tc-99m macroaggregated albumin
C. Tl-201 thalous chloride
D. Sr-89 strontium chloride
E. I-131 sodium iodide

280. A diagnostic radiopharmaceutical used for lung scans


A. Tc-99m diphosphonates
B. Tc-99m macroaggregated albumin
C. Tl-201 thalous chloride
D. Sr-89 strontium chloride
E. I-131 sodium iodide

281. Which of the following is/are true regarding the functions of Vitamin A?
I. Act as antirachitic vitamin
II. Maintains the integrity of epithelial membranes
III. Essential in the formation of rhodopsin and the normal functioning of the retina
A. I only
B. II and III only
C. I, II and III
D. I and III only
E. II only

282. Which of the following is/are true regarding the functions of Vitamin C?
I. Formation of intercellular collagen
II. Healing of bone fractures
III. Metabolism of tyrosine
A. I only
B. II and III only
C. I, II and III
D. I and III only
E. II only

283. Hospitals may be classified in different ways according to


I. Type of service
II. Length of stay
III. Ownership
IV. Bed capacity
A. I, II, III and IV
B. I, II and III only
C. I and III only
D. I and IV only
E. I, III and IV

284. Which of the following are fundamental functions of a hospital?


I. Patient care
II. Wellness
III. Research
IV. Education
A. I and II only
B. I and III only
C. I, II and III only
D. I and III only
E. I, II, III and IV

285. Which of the following is/are true regarding open questions?


I. Allow the patient sufficient latitude in interpreting and responding to requests for
information
II. Cannot be answered in one phrase or with a “yes” or a “no”
III. Its limited use will result in an overly long unfocused and inefficient interview
A. I only
B. II and III only
C. I, II and III
D. I and II only
E. II only

286. The provision of integrated, accessible health-care services by clinicians who are accountable for
addressing a large majority of personal health-care needs developing a sustained partnership with the
patient, and practicing in the context of family and community
A. Disease management
B. Acute care
C. Primary care
D. Institutional care
E. Long-term care

287. An evaluative approach to health-care delivery system that attempts to improve outcomes for patients
with a specific disease while optimizing the overall use of the health-care resources
A. Patient focused care
B. Benefit management
C. Disease management
D. Case management
E. Primary care

288. A process by which an experienced professional works with patients, providers, and insurers to
coordinate all services deemed necessary to provide the patient with medically appropriate health care
A. Patient focused care
B. Benefit management
C. Disease management
D. Case management
E. Primary care

289. Categories listed by application of the US Food and Drug Administration definitions to available clinical
data in order to define a drug’s potential to cause birth defects or fetal death
A. Phenol coefficient
B. Relative risk ratio
C. Bioequivalence requirement
D. Pregnancy risk
E. Therapeutic index

290. A pregnancy risk category that indicates an adequate studies in pregnant women have failed to show a
risk to the fetus in the first trimester of pregnancy and no evidence of risk in later trimester
A. Category B
B. Category D
C. Category A
D. Category C
E. Category S

291. A pregnancy risk category that indicates that studies in animals or humans show fetal abnormalities, or
adverse reaction reports indicate evidence of fetal risk
A. Category B
B. Category A
C. Category O
D. Category X
E. Category Y

292. A pregnancy risk category that indicates that animal studies have shown an adverse effect on the fetus, but
there are no adequate studies in humans
A. Category C
B. Category B
C. Category D
D. Category E
E. Category O

293. A pregnancy risk category that indicates that there is evidence of risk to the human fetus, but the potential
benefits of use in pregnant women may be acceptable despite potential risks
A. Category E
B. Category B
C. Category D
D. Category A
E. Category X

294. A pregnancy category that indicates that animal studies have not shown an adverse effect on fetus, but
there are no adequate clinical studies in regnant women
A. Category D
B. Category B
C. Category X
D. Category A
E. Category C

295. Which of the following is/are forms of non verbal communication?


I. Clothing and hairstyle
II. Posture
III. Eye contact
IV. Proximity to other person
A. II and III only
B. III only
C. I, II and III only
D. I, II and IV only
E. I, II, III and IV
296. Which of the following is/are examples of mechanical barriers to effective listening and communication?
I. Noise
II. Harsh light
III. Impaired eyesight
IV. Uncomfortable chairs
A. I, II, III and IV
B. I, II and IV only
C. II and III only
D. I, II and IV only
E. I and II only

297. A skilled and principled use of relationships to facilitate-self knowledge, emotional acceptance and
growth, and the optimal development of personal resources
A. Communication
B. Counseling
C. Listening
D. Health promotion
E. Problem solving

298. The ability to see the world from the point of view of another person, through their frame of reference,
through their conceptual and emotional spectacles
A. Sympathy
B. Genuineness
C. Listening
D. Empathy
E. Involvement

299. According to the preventive model, which of the following is/are function/s of tertiary level of prevention
I. Prevention of deterioration, relapse and complications
II. Promotion of rehabilitation
III. Prevention of onset disease
A. I, II and III
B. I and II only
C. II and III only
D. II and III only
E. I only

300. According to the preventive model, which of the following is/are function/s of secondary level of
prevention?
I. Help adjustment to terminal conditions
II. Minimization of the severity of an existing disease
III. Reversal of progress of an existing disease

A. I, II and III
B. I and II only
C. I and III only
D. II and III only
E. I only
Hospital Pharmacy Page 1 of 22

D 1. A book ready for inspection by the BFAD anytime during business hours and should be kept for two years:
a. poison book d. prescription book
b. sales book e. purchase book
c. additional opium book

C 2. A type of drug distribution system wherein all drugs are retained in the main pharmacy until receipt of the physician’s
initial order:
a. floor stock system c. individual prescription system
b. decentralized unit dose system d. both a and c

B 3. Which of the following is not an anti-asthma preparation?


A. terbutaline d. salbutamol
b. nifedipine e. theophylline
c. prednisolone

D 4. Oral hypoglycemics are usually most useful in:


a. Type I diabetes d. Type II diabetes
b. ascariasis e. UTI
c. asthma

B 5. It enables the pharmacist to monitor drug utilization of each patient confined in the hospital:
a. Admission history d. adverse drug reaction report
b. patient medication profile e. progress note
c. patient chart

C 6. FeSO4 is used for:


a. cough d. parasitism
b. pain e. arthritis
c. anemia

C 7. A laminar flow hood is used when intravenous solutions are:


a. labeled d. administered
b. stored e. both a and c
c. prepared

E 8. San Lazaro Hospital specializes in:


a. mental health d. bone disorder
b. child birth e. contagious diseases
c. pulmonary diseases

A 9. The purpose of the Pharmacy and Therapeutic Committee are the following:
a. advisory and educational d. dispensing and inventory
b. advisory and planning e. both a and d
c. educational and planning

B 10. Hospital department that supplies sterile linen, operating room packs and other medical surgical supplies:
a. nursing department d. laboratory service department
b. central service department e. pharmacy department
c. medical social department

D 11. Ranitidine is used for:


a. High blood pressure d. peptic ulcer
b. asthma e. diabetes
c. allergy

D 12. Total parenteral nutrition should be protected from light because it contains:
a. fats d. vitamins
b. carbohydrates e. proteins
c. electrolytes

B 13. The first recognized representative of pharmaceutical profession:


a. community pharmacist d. industrial pharmacist
b. hospital pharmacist e. clinical pharmacist
b. manufacturing pharmacist

A 14. A person who is engaged in managing, directing and performing both professional and administrative services
associated with the operation of the hospital pharmacy department:
a. chief pharmacist d. pharmacy and therapeutics committee
b. staff pharmacist e. medical director
c. pharmacy supervisor
Hospital Pharmacy Page 2 of 22

C 15. Serves as a tool with which the pharmacist assures appropriateness, safety and effectiveness of drug therapy:
a. medical staff d. physician’s original order
b. pharmacy and therapeutics committee e. patient chart
d. patient medication profile

A 16. The following person performs some of the pharmacist’s technical functions, except:
a. governing body d. secretary
b. clerk e. pharmacy aide
c. intern

E 17. These are administrative and management responsibilities of pharmacist, except:


a. preparation of reports
b. inventory control
c. plan and integrate professional services
d. budget
e. sterilizing used syringes

A 18. Men are not accepted in this department for treatment:


a. gynecology d. nephrology
b. urology e. cardiology
c. internal medicine

D 19. PSHP was established in what year?


a. 1930 d. 1962
b. 1942 e. 1970
c. 1950

A 20. Which is not a primary function of hospital?


a. preventive medicine c. providing health care
b. giving health information d. conduct scientific research

C 21. All of the following are minimum standards for hospital pharmacy, except:
a. facilities c. drug administration
b. research d. assuring rational drug therapy

C 22. The pharmacy department is not responsible for:


a. inspecting unused/expired drugs
b. preparing parenteral nutrition
c. distributing sterile linen
d. informing the hospital staff pertaining to medication

D 23. Primary source of income of private hospitals:


a. gift shop c. pharmacy department
b. canteen d. patient’s bill

C 24. Which is not a clinical pharmacy service?


a. monitor patient response d. obtain and document medication histories
b. conduct drug utilization review e. counsel patient
c. prepare patient bed

B 25. Incumbent president of PSHP:


a. Lourdes Echauz d. Gloria Arroyo
b. Normita Leyesa e. Siopin Co
c. Eladio Tinio

C 26. Responsible for creating a hospital formulary:


a. pharmacy d. medical staff
b. medical director e. both b and c
c. Pharmacy and Therapeutics Committee

A 27. Through a _____, the policies of hospitals are implemented on a day-to-day basis:
a. chief executive officer d. doctors
b. corporation e. pharmacist
c. board of trustees

E 28. The act of a pharmacist in supplying one or more drug products to, or for a patient, usually in response to an order
from a doctor:
a. administering d. giving
b. prescribing e. dispensing
c. procuring
Hospital Pharmacy Page 3 of 22

B 29. It is a list of medications which are available only in that hospital:


a. MIMS d. RPS
b. Formulary e. both a and b
c. PDR

E 30. Most hospitals provide the following modalities:


a. surgery d. drug therapy
b. hypnosis e. a and d
c. acupuncture

C 31. The pharmacy department should have floor space for the following except:
a. office d. compounding and dispensing area
b. store room e. circulation
c. rest room

D 32. Emergency box in the hospital contains the following drugs, except:
a. phenobarbital d. guaiafenesin
b. epinephrine e. M,orphine
c. heparin

B 33. Pharmacy department is best classified under what division of hospital:


a. general c. administrative
b. clinical d. medical

B 34. During chickenpox, measles or smallpox outbreak, this department would be very busy:
a. dermatology c. pediatric
b. infectious disease d. neurology

A 35. This should be performed if the patient died while in the hospital:
a. autopsy c. biopsy
b. surgery d. physicl examination

C 36. All of these should be taken into consideration in preparing dosage form not available commercially,
except:
a. stability d. pharmacokinetic
b. quality assurance e. economic factor
c. availability of the patient

B 37. The following are general sections of a pharmacy procedural manual, except:
a. facilities d. services and activities
b. library e. personnel
c. organization

B 38. Who is not qualified to be a member of the Pharmacy & Therapeutics Committee?
a. cardiologist d. hospital administrator
b. pharmacy intern e. nurse
c. gastroenterologist

E 39. Drugs approved by the Pharmacy & Therapeutics Committee may be classified into the following categories, except:
a. specialized formulary drug
b. investigational drug
c. drug approved on a conditional trial period
d. formulary drug
e. non-formulary drug

A 40. First president of PSHP:


a. Rosario Capistrano Tan d. Marcelia Itturalde
b. Lourdes Echauz e. Siopin Co
c. Violeta Alvarez

D 41. Nutritional solution prepared by hospital pharmacist for patients who has dysfunctional GIT:
a. IV mixture d. TPN
b. D5W e. NTP
c. NSS

C 42. Medical staff who are responsible for taking care of patient and who has direct involvement with the staff
organizational and administrative duties:
a. resident medical staff d. consulting medical staff
b. honorary medical staff e. honorary medical staff
c. attending medical staff
Hospital Pharmacy Page 4 of 22

E 43. responsibilities of Pharmacy & Therapeutics Committee, except:


a. develop a mechanism for reporting and reviewing ADR
b. prepare policies governing the activities of medical representatives
c. develop a book containing the list of medications available in the hospital
d. prepare list of emergency drugs
e. interview pharmacy staff

C 44. Information which is not necessary in an inpatient prescription:


a. date when the order was written
b. name and strength of medication
c. patient’s address or graphic information
d. printed name of physician

B 45. Which will not help in ensuring the safety of a pharmacy department in the hospital?
a. segregation of poisonous materials from non-poisonous materials
b. employment of a sufficient number of nonqualified personnel
c. policy with regards to the use of investigational drugs
d. availability of adequate and safe working space

C 46. Which may not lead to an adverse drug reaction?


a. error in compounding
b. use of the wrong drug
c. rashes due to sudden change in temperature
d. overdose
e. underdose

D 47. Pharmacy division responsible for improving formulations of existing products:


a. assay and quality control division
b. manufacturing and packaging division
c. departmental services division
d. pharmaceutical services division

A 48. Responsible for preparing the pharmacy newsletter:


a. drug information services division
b. education and training division
c. administrative services division
d. pharmaceutical research division
e. departmental services division

A 49. This is not a necessary information in drug evaluation for inclusion in the hospital formulary:
a. available sizes of drugs d. therapeutic indication
b. adverse reactions e. bioavailability
c. source of supply

C 50. Takes charge of interviewing medical sales representatives:


a. education and training division
b. in-patient services division
c. purchasing and inventory control division
d. administrative services division
e. departmental services division

A 51. Drug recalls may emanate from the following agencies, except:
a. nursing department d. manufacturer
b. BFAD e. pharmacy department
c. Pharm,acy & Therapeutics Committee

D 52. Nursing stations are inspected pr visited by the pharmacist due to the following reasons, except:
a. to check labels for eligibility
b. to remove expired products
c. to remove deteriorated drugs
d. to remove empty containers

A 53. Patient drug profile is maintained in the pharmacy department for the following purposes, except:
a. to lower down the patient’s expenses
b. to prevent potential drug interactions
c. to promote rational use of drugs
d. to detect drug-induced laboratory test abnormalities
e. to prevent drug allergies
Hospital Pharmacy Page 5 of 22

B 54. Which one is not useful to the pharmacist in evaluating the quality of care provided to patient with a certain disease
state?
a. proper choice of dosage form d. proper route of administration
b. proper price of medication e. proper strength of medication
c. proper choice of therapeutic agent

B 55. The process used to determine the “waiting period” of an outpatient for his prescription?
a. drug recall c. flow process chart
b. queuing theory d. activity chart

C 56. Which question is not of importance in the pharmacist’s review of the patient’s drug therapy record?
a. Were the tests for predicting drug allergies done on patients with a suspicious history?
b. Was the drug employed one with a special effect upon the diagnosed ailment?
c. Was the drug taken with water?
d. Did the patient receive his therapy via oral route when the topical route should have been employed?

B 57. An enteric-coated tablet that disintegrates upon passing through the stomach falls under what type of defective
product?
a. faulty drug delivering apparatus c. inadequate packaging
b. defective dosage form d. deteriorated product

E 58. All of the following are direct responsibilities if the chief pharmacist, except:
a. schedules work hours for pharmacy staff
b. establishes and/or implements written policies and procedures for pharmacy operation
c. evaluates job performance of pharmacy staff
d. select, hires and institutes disciplinary action and discharges pharmacy staff
e. performs packaging and labeling functions to promote product stability

C 59. Filling of prescription is lawfully the job of:


a. physician d. sales clerk
b. nurse e. patient
c. pharmacist

C 60. The following are pharmacists’ responsibilities, except:


a. review patient’s drug administration form to insure that all doses are being administered and charted
correctly
b. assist the physician in selecting dosage regimen and schedules
c. transcribe accurately new drug administration forms
d. provide drug information to physicians, nurses and other health care personnel

B 61. The pharmacy department does not supply one of the following:
a. IV admixture d. radiopharmaceuticals
b. sterile linen e. biological
c. narcotics

A 62. What is the most important storage condition for narcotic drugs?
a. security d. light
b. sanitation e. moisture
c. temperature

A 63. Which is not an acceptable method of drug information dissemination?


a. megaphone d. leaflets
b. pharmacy bulletin e. seminar
c. newsletter

A 64. This pharmacy division coordinates and control all drug delivery and distribution systems:
a. departmental services division c. purchasing and inventory control division
b. administrative services division d. central supply services division

D 65. Not a suitable location for the hospital pharmacy department:


a. first floor c. near the emergency room
b. near the information area d. basement

A 66. Control substance which is non-refillable:


a. schedule I c. schedule IV
b. schedule III d. schedule V

E 67. Which duty should not be assigned to a pharmacy technician?


a. maintain drug inventory records
b. clean prescription equipment
c. print labels for prepackaged drugs
d. weigh ingredients in bulk compounding
e. compound prescriptions
Hospital Pharmacy Page 6 of 22

E 68. A very important liaison between the hospital, patient and his community:
a. administrator d. pharmacist
b. P & TC e. medical social service
c. priest

C 69. The service generally under the supervision of a licensed physician who has a basic interest in hematology:
a. pathology d. dietetic service
b. anesthesia e. morgue
c. blood bank

A 70. Psychiatric, AIDS and leprosy-victims must be attended by:


a. custodial hospital d. short-tem hospital
b. general hospital e. long-term hospital
c. non-accredited hospital

B 71. Which of the following is not needed in the admission sheet?


a. patient’s name and address d. occupation
b. monthly income e. family status
c. sex and date of birth

A 72. Which is not included in the ;laboratory report of a patient?


a. manufacturing data d. hematology data
b. microbiology data e. chemistry data
c. pathology data

E 73. The hospital which is affiliated with a university:


a. Philippine Heart Center d. National Kidney Institute
b. Quezon Institute e. Philippine General Hospital
c. San Lazaro Hospital

C 74. This is one of the items in the medical record for the purpose of providing the physician with a chronological picture
and analysis of the clinical course of the patient:
a. special examination d. pathologic findings
b. history of present illness e. physical examination
c. progress notes

E 75. This department enables the physician to assume the care of the patient at any time:
a. pharmacy d. nursing
b. medical social service e. medical records
c. central service supply

A 76. This type of hospital provides care to patients with various types of illnesses:
a. general hospital d. not-for-profit hospital
b. special hospital e. hospital with 50-99 beds
c. short-term hospital

D 77. This is not a chronic ailment:


a. asthma d. diarrhea
b. cancer e. diabetes
c. epilepsy

B 78. This is not a function of the dietetic service:


a. participate in ward rounds and conferences
b. recording medication histories of patients
c. plan patient’s menu
d. interview patients regarding their food habits
e. counseling patients concerning normal or modified diet regimen

B 79. One of the responsibilities of the pharmacy department is to control all drugs used in the hospital. Which of the
following does not fall in this responsibility?
a. drug samples must be distributed only through the pharmacy
b. chief pharmacist should have a private office
c. maintenance of an up-to-date formulary
d. routine inspection of drug storage areas

A 80. Hospital maybe classified in different ways, except:


a. location d. type of service
b. ownership e. length of service
c. bed capacity
Hospital Pharmacy Page 7 of 22

D 81. This group consists of medical practitioners of recognized professional ability:


a. active d. consulting
b. associate e. honorary
c. courtesy

B 82. Serves as the secretary of the P & TC:


a. medical director c. nursing director
b. pharmacy director d. administrator

C 83. It serves as a guide to the hospital pharmacist in the development and execution of effective and proficient
pharmaceutical services in the hospital:
a. formulary system d. patient’s medication profile
b. P & TC e. secondary care
c. pharmacy procedural manual

B 84. The type of ambulatory care sought for by most outpatients:


a. tertiary care c. emergency care
b. primary care d. secondary care

B 85. Contains one discrete pharmaceutical dosage form:


a. unit dose d. multiple dose package
b. single unit package e. both b and c
c. unit dose package

B 86. A process pf lifelong learning availed of by pharmacists for maintaining and improving professional competence:
a. undergrad studies c. graduate studies
b. continuing education d. intensive program

A 87. Represents the type of transaction that involves purchase of goods on specific volume and receiving certain
merchandise at no additional cost:
a. deals c. discount
b. volume contract d. group purchasing

C 88. A modern usage of the term ambulatory patient:


a. in-patient d. institutionalized patient
b. ward patient e. hospitalized patient
c. non-institutionalized patient

B 89. Medication administered to the wrong patient may be classified as:


a. wrong dose error c. wrong time error
b. unauthorized drug error d. omission error

A 90. Which of the following elements are common to ambulatory care but not to in-patient pharmaceutical service
program?
a. information on the proper use of the drug will be disseminated
b. medications dispensed will be completely labeled and packaged
c. the pharmacy program must be directed by a qualified pharmacist
d. drugs will be properly controlled

A 91. Which of the following does not correctly describe formulary system?
a. detrimental to the free enterprise system
b. appraises drug therapy
c. controls drug cost
d. assures quality of drug

C 92. A prescription label must contain all of the following, except:


a. patient name c. patient age
b. patient address d. name of drug

D 93. This consists of routine systematic review of the body:


a. provisional diagnosis d. physical examination
b. special examination e. pathologic findings
c. personal family history

C 94. Medical staff is composed of:


a. pharmacists d. medical technologists
b. nurses e. both b and c
c. physicians
Hospital Pharmacy Page 8 of 22

A 95. This ensures that drug orders are not inappropriately continued:
a. automatic stop order d. professional care service
b. P & TC e. group purchasing
c. formulary system

A 96. The final decision as to which drugs should be included in the ward stock rests upon:
a. P & TC d. physician
b. pharmacist e. administrator
c. nurse

D 97. With respect to other drug distribution methods, the advantages of unit-dose include the following, except:
a. safer for the patient
b. more effective method of utilizing professional resources
c. less workload to the pharmacists
d. less manpower requirements
e. both a and b

B 98. The following are true, except:


a. two preparations each labeled with the same active chemical components are generic equivalents
b. two preparations each labeled with different active ingredients are clinically effective equivalents
c. both high and poor quality pharmaceuticals are available under both generic names and brand names
d. a relationship exists between quality pharmaceuticals and therapeutic efficacy

D 99. Hospital Pharmacy:


a. patient, physician, nurse, pharmacist relationship
b. practice of pharmacy in hospital setting
c. stepping stone to clinical pharmacy
d. all of the above

D 100. Purpose of P and TC, except:


a. develops a formulary of accepted drugs for use in the hospital
b. evaluates clinical data concerning drug for use in a hospital
c. adds or deletes from the list of drugs accepted for use in the hospital
d. advices the doctor as to what brand of drug to use

D 101. Injections are often indicated as the correct form of administering a drug when:
a. the drug is a suspension c. the drug is insoluble
b. the drug contains alcohol d. the drug is not easily absorbed from the GIT

D 102. Functions of the P and TC, except:


a. review adverse drug reactions to drugs administered
b. to recommend addition and deletion of drugs
c. to serve as advisory to medical staff and pharmacists
d. approval of duplicate preparation to be listed in the formulary

A 103. Biological and other thermolabile medications should be kept in:


a. separate compartment in a refrigerator
b. locked cabinet
c. the display cabinet together with other drugs
d. none of the above

B 104. Basic function of hospital, except:


a. education c. patient care
b. delivery of drugs to various outlets d. research

B 105. Medical staff primarily concerned with regular care of patient:


a. honorary medical staff d. resident medical staff
b. active medical staff e. consulting medical staff
c. associate medical staff

D 106. An authorized, structured and continuing program that reviews, analyzes and interprets patterns of drug usage in a
given health care delivery system against predetermined standards:
a. adverse drug reaction review c. P and TC
b. formulary system d. drug use review

C 107. Type of drug distribution system for government hospital:


a. individual prescription order system
b. unit dose
c. combination of individual prescription and floor stock
d. complete floor stock
Hospital Pharmacy Page 9 of 22

D 108. Advantages of unit dose distribution:


a. reduction in the incidents of medication error
b. decrease ion the total cost of medication related activities
c. more accurate billings
d. all of the above

A 109. An extension of the main hospital pharmacy:


a. satellite pharmacy c. both an and b
b. clinic d. none of the above

D 110. Institutional pharmacy is basically:


a. extended care pharmacy c. manufacturing pharmacy
b. community pharmacy d. hospital pharmacy

D 111. Pyrogenic contamination may cause the patients to have:


a. fever d. a and b
b. chills e. a and c
c. infection

C 112. Education and training division:


a. coordinate programs if undergraduate and graduate pharmacy students
b. participate in hospital-wide educational programs involving doctors, nurses, etc.
c. both a and b
d. none of the above

D 113. Which is/e true about the DDB prescription form?


a. original copy – pharmacist copy
b. duplicate copy – patient’s copy
c. triplicate copy – doctor’s copy
d. all of the above

B 114. Any drug that has not yet been released for general use and has not been cleared for marketing by the BFAD:
a. prototype drug c. pro-drug
b. investigational drug d. controlled drug

D 115. Responsible for specifications as to the quality, quantity and source of drugs:
a. purchasing agent c. medical staff
b. president d. pharmacist

A 116. A modern and reliable means of controlling the volume of purchases:


a. computation of inventory turnover c. all of the above
b. non-purchase d. none of the above

D 117. In inventory control, the turnover rate is calculated by which of the following formula?
a. annual purchases + annual inventory
b. annual purchases ÷ annual inventory
c. annual purchases – annual inventory
d. annual purchases / annual inventory

C 118. The least acceptable method of transmitting prescription orders of physician:


a. by inputing the order into a computer terminal
b. by sending a carbon copy of the physician’s original medication order
c. transcribing by nursing personnel to a requisition form
d. the physician write the medication order on a separate blank and send it to the pharmacy

A 119. The leading and major organization of hospital pharmacists in the country:
a. PSHP c. ASHP
b. AMA d. PPHA

D 120. Injections are randomly tested for fever-producing agents, which are called:
a. analgesics c. histamines
b. antitussives d. pyrogens

D 121. Abilities required of hospital pharmacist, except:


a. thorough knowledge of drugs and their action
b. ability to develop and conduct a pharmaceutical manufacturing program
c. ability to conduct and participate in research
d. manage a hospital

A 122. Total parenteral nutrition is administered through:


a. subclavian vein c. none of the above
b. Femoral artery d. both a and b
Hospital Pharmacy Page 10 of 22

D 123. Which is not part of patient’s medication history?


a. past and present medication
b. drug allergies
c. previous adverse effects associated with medication use
d. cost of medication

C 124. Fundamental functions of hospitals, except:


a. patient care d. research
b. education e. public health
c. housing

C 125. Medication orders ( Chart Orders) differ from prescription in that they:
a. are intended for ambulatory use
b. contain only the generic name of the drug
c. may contain non-medication instructions from the practitioner
d. contain the quantity of medication to be dispensed

C 126. A patient medication orders states that he is to receive ampicillin 500 mg p.o. q8H. How many doses of ampicillin
250 mg capsules should be placed in the patient’s unit dose drawer daily?
a. 2 c. 6
b. 4 d. 10

B 127. Providing the correct medicine at the correct time in the correct dose and correct route of administration:
a. patient care c. research
b. rational drug therapy d. drug information

C 128. The primary goal of a hospital formulary is to:


a. strictly control prescribing in a hospital
b. save hospital money
c. insure that the patients receive the best drug therapy possible
d. influence medical staff to use approved drugs only

C 129. It shows the flow of authority and services on hand:


a. pharmacy procedural manual c. organizational chart
b. hospital pharmacy d. journal

A 130. Manner of affixing the label to an IV admixture preparation container:


a. upside down c. vertically
b. sideways d. normal way

C 131. The ideal drug distribution system


a. complete floor stock system c. unit dose system
b. individual prescription system d. none of the above

A 132. Additives of TPN are:


a. electrolytes and vitamins c. amino acids and anesthetics
b. antibiotics and fats d. none of the above

A 133. Continuous IV infusion means:


a. administration of a large volume of drug at a constant rate usually 100-150 ml/hr for 8-24 hrs
b. rapid IV push in less than 30 seconds
c. administration oat a rate not to exceed 1ml/min
d. none of the above

D 134. A separate sterile room is required for:


a. reconstitution of lyophilized injections
b. ophthalmic preparations
c. preparation of IV admixtures
d. all of the above

B 135. A sign to the pharmacist that the patient is overusing a certain product:
a. not purchasing c. healthy patient
b. frequency of refills d. discharge patient

C 136. Responsible of the procurement, distribution and control of all drugs used within the hospital:
a. purchasing agent c. pharmacist
b. medical staff d. lay personnel

D 137. Feature/s of the unit dose drug distribution system


a. maintains duplicate copy of the doctor’s order c. delivers medications in ‘unit dose’ forms
b. maintains patient medication profile d. all of the above
e. a and c only
Hospital Pharmacy Page 11 of 22

D 138. Pharmacists monitoring drug therapy would typically be checking all of the following except:
a. proper utilization of drug d. appropriate diagnosis
b. drug interactions e. none
c. allergy contraindications

A 139. True statements about D5W include all of the following except:
a. pH of 8 to 10
b. isotonic
c. often used in IV admixture
d. should be used with caution among diabetic patients
e. none

B 140. A method of administering IV fluids and medications that provides a slow, primary line infusion to maintain
therapeutic drug level or provide electrolyte replacement:
a. IV push d. heparin lock
b. drip infusion e. none
c. piggyback

A 141. A health care supply described as a portable toilet:


a. commode d. rectal tube
b. truss e. none
c. bedpan

D 142. Which of the following is not a method of operating a unit dose dispensing program:
a. Centralized UDDS d. non-charge
b. decentralized UDDS e. none
c. combination of a and b

B 143. Attending staff is also known as:


a. associated medical staff d. honorary medical staff
b. active medical staff e. consulting medical staff
c. resident medical staff

B 144. A rubber, plastic or glass tube inserted into the bladder in order to withdraw urine from patients unable to void
naturally:
a. rectal tube d. incontinence pants
b. catheter e. none
c. urinals

C 145. The injection of small amount of undiluted medication directly into the vein or through a heparin lock:
a. drip infusion d. IV piggyback
b. intermittent infusion e. none
c. IV bolus

B 146. A method of administering IV fluids and medication using a volume control device designed to administer small
amount of fluid over a specified time:
a. heparin lock d. IV push
b. additive set e. none
c. IV piggyback

C 147. Purpose of a pharmacy policy and procedure manual:


a. ensure uniformity and consistency
b. the best defense in the event of litigation
c. both of the above
d. none of the above

C 148. Responsibilities often delegated top hospital pharmacy technicians include:


a. counseling patients d. pharmacokinetic monitoring
b. review physician’s order e. none
c. prepare IV admixture

C 149. All of the following are considered desirable functions of contemporary hospital pharmacy except:
a. clinical consultation d. purchasing and inventory control
b. IV admixture e. none
c. floor stock distribution

B 150. Pharmacist’s clinical function includes the following, except:


a. drug utilization review d. pharmaceutical care
b. drug dispensing e. drug informatratrix
c. pharmacokinetic monitoring
Hospital Pharmacy Page 12 of 22

B 151. Place for critically and seriously ill patients:


a. intermediate care unit d. all of the above
b. intensive care unit e. none of the above
c. long term care unit

A 152. IV administration of a large amount of fluid over a prolonged period of time:


a. infusion c. IV push
b. IV bolus d. piggyback

A 153. Under the administrative organization chart, the central sterile supply fall under the:
a. clinical services d. all of the above
b. ambulatory services e. none of the above
c. adverse drug reaction

B 154. It helps improve drug prescribing practice by promoting the safe and rational use of drug:
a. drug utilization review d. all of the above
b. patient medication profile e. none of the above
c. adverse drug reaction

B 155. Area which offer the hospital great financial savings and pharmacists’ pride and prestige:
a. dispensing d. all of the above
b. extemporaneous compounding e. none of the above
c. drug consultant

B 156. Emergency drugs are considered as:


a. OR drug order d. automatic stop order
b. STAT order e. all of the above
c. follow up order

D 157. The risks of medication misuse or errors are often due to:
a. increase frequency of changes in medication use
b. increase number of medications for patient
c. increase potency of new drugs
d. all of the above
e. none of the above

D 158. Automatic stop-order policy fro drugs:


a. when patient goes to the operating room
b. when the patient is transferred to another service
c. after 24 hours of administration
d. a and b only
e. all of the above

E 159. Best method of providing service when pharmacy is closed:


a. provide an emergency cart for all unit
b. allow access of a nurse or doctor into the pharmacy
c. have a pharmacist on call
d. a and b only
e. a and c only

A 160. The responsible provision of drug therapy for the purpose of achieving definite outcomes that improve the quality of
life of a patient is:
a. pharmaceutical care d. all of the above
b. clinical pharmacy e. none of the above
c. traditional hospital pharmacy

C 161. Legal document which contains all the treatment done to the patient:
a. patient medication profile d. doctor’s sheet
b. therapeutic sheet e. all of the above
c. patient chart

E 162. In-charge of inspecting the contents of emergency box daily:


a. P and TC d. both a and c
b. pharmacist e. both b and c
c. nurse

D 163. Parts of formulary system except:


a. information of hospital policies and procedures concerning drugs
b. drug product listing
c. special information
d. specialty formularies
Hospital Pharmacy Page 13 of 22

D 164. Three potential benefit of formulary systems, except:


a. economic c. education
b. Therapeutic d. expensive

C 165. Advantages of unit dose drug distribution, except:


a. more efficient use of personnel c. increased floor stock
b. minimization of credits d. greater workload control

C 166. A potent analgesic emergency drug:


a. epinephrine c. pethidine
b. dopamine d. ephedrine

B 167. It encompasses the collection, organization, retrieval, interpretation, evaluation and dissemination of information
pertaining to all aspects of medicine:
a. PTC c. LAFH
b. DIS d. EDL

B 168. The group established in the Philippines for hospital pharmacists:


a. ASHP c. ASHP
b. PSHP d. PACOP

C 169. Which one is used as antiviral?


a. terramycin c. zovirax
b. garamycin d. vibramycin

B 170. Which of the following is not an anti-asthma preparation?


a. terbutaline c. salbutamol
b. cimetidine d. prednisolone

E 171. Betadine is used as:


a. antifungal d. both a and b
b. antibacterial e. all of the above
c. antiinfective

D 172. These are acceptable colors of pharmacy wall, except:


a. white d. orange
b. cream e. baby pink
c. mint

B 173. The drug atropine sulfate is used as:


a. antiasthmatic c. antio-anginal
b. anticholinergic d. anthelmintic

C 174. Excessive rapidity of respiration:


a. cytopenia c. tachypnea
b. necrosis d. tachycardia

C 175. Surgical puncture of the heart:


a. pericardium c. cardiocentesis
b. colostomy d. necroscopy

A 176. Functional disorder of the nervous system:


a. neurosis c. meningitis
b. tachycardia d. phlebotomy

B 177. The drug used to paralyze the patient during surgical procedure:
a. penthotal sodium c. nubain
b. vecuronium d. phenobarbital

E 178. These are examples of objective parameters, except:


a. blood level d. blood pressure
b. laboratory tests e. degree of pain
c. x-ray tests

B 179. NPO means:


a. non-progressive operation d. nothing after six
b. nothing by mouth e. none of the above
c. with full stomach

D 180. The most accurate method in calculating children’s dose based on weight and height is:
a. Young’s rule c. Cowling’s rule
b. Fried’s rule d. BSA
Hospital Pharmacy Page 14 of 22

C 181. The active ingredient of Desenex powder is:


a. tolnaftate c. undecylenic acid
b. salicylic acid

D 182. During eye examination, the following are noted, except:


a. visual acuity c. EOM
b. pupillary reflex d. Presence of tears

A 183. The breath of an alcoholic patient may smell:


a. fruity c. musty
b. ammoniacal d. sweet

C 184. Cromolyn sodium is useful in asthma only in:


a. oral tabs c. inhalation
b. pills d. injectable

B 185. It is ideal and proper condition for aseptic handling of hyperalimentation and intravenous fluids:
a. HEPA filters c. TPN
b. LAFH d. ventilator

B 186. The normal drop rate ion TPN therapy is:


a. 1 ml/min c. 1 ml/min
b. 2 ml/min d. 5 ml/min

B 187. The starting dose of TPN per day is:


a. 6 L/day c. 5 L/day
b. 2.5 L/day d. 2 L/day

B 188. Patient who is not bedridden, who can take care of himself and administer his own medication:
a. patient care c. geriatric
b. ambulatory patient d. pediatric

D 189. Assessment of the musculoskeletal system includes:


a. assessment of posture and gait c. ROM testing
b. measurement of the extremities d. all of these

D 190. The skin is assessed for:


a. lesions c. mobility
b. hydration d. all of these

D 191. Inspection involves:


a. concentrated visualization of the patient
b. listening to sounds that may emanate from the patient
c. noting peculiar odors of the patient
d. all of these

B 192. Blood pressure is determined by:


1) palpation 2) percussion 3) auscultation 4) inspection
a. 1, 2, 3, 4 c. 2, 4
b. 1, 3 d. 1, 2 , 3

D 193. During palpation, the following are used, except:


a. fingers c. back of the hands
b. palms d. fist

C 194. Borborygmi is:


a. a discontinuous, bubbling abdominal sound of short duration
b. a coarse abdominal sound that disappears on coughing
c. loud gurgling and tinkling sounds heard over the abdomen
d. also known as crepitation

C 195. It is a technique that delivers an amount of electrolyte solution over a period of several hours and is used to replace
electrolytes needed by the patient:
a. Drop factor c. fluid blousing
b. metabolism d. loading dose

B 196. Drug which is an antineoplastic agent:


a. gentamicin c. furosemide
b. cisplatin d. plasil

A 197. Daily administration of aspirin can impair:


a. hearing c. both a and b
b. seeing
Hospital Pharmacy Page 15 of 22

A 198. This service is responsible for the procurement, planning and preparation of food for the patient and hospital staff:
a. dietary service c. record service
b. nursing service d. laboratory service

B 199. The purpose of laboratory examination and radiological services is for:


a. therapeutic c. treatment
b. diagnostic d. exercise

C 200. Powerful tool for the orderliness, cleanliness and sanitary procedures in the hospital pharmacy department:
a. chief pharmacist d. all of the above
b. assistant chief pharmacist e. none of the above
c. staff pharmacist

D 201. Objectives of drug information service:


a. answering inquiries d. a and b
b. publication e. all of the above
c. budget preparation and printing scheme

E 202. The formulary is very important so it should be:


a. complete d. a and b
b. concise e. all of the above
d. easy to use

A 203. A bulk supply of each drug product is stored on the nursing station:
a. floor stock d. all of the above
b. individual prescription order e. none of the above
e. emergency cart

B 204. Which of the following solutions would be sterile when dispensed?


a. tinctures d. sprays
b. ophthalmic solution e. none of the above
c. spirit

E 205. Function of the Chief Pharmacist in Pharmacy and Therapeutic Committee:


a. maintenance of an adequate up to date library and drug therapy references
b. interviewing and screening of professional medical representative
c. preparation and dissemination of accurate minutes of committee meeting
d. all of the above
e. a and c

C 206. One who shall initiate regulation pertaining to the professional; policies of the pharmacy department:
a. pharmacy and therapeutic committee d. clinical pharmacist
b. medical director e. doctors
c. chief pharmacist

A 207. Relevant pharmaceutical services:


a. quality control, ICV admixtures
b. drug information, education and training
c. extemporaneous compounding
d. drug dispensing
e. all of the above

A 208. Pharmacy records on all drugs dispensed to the patient are in:
a. patient medication profile d. patient bill
b. therapeutic sheet e. all of the bill
c. patient chart

B 209. A department that supplies sterile line, sterile kits, operating room packs, needles, syringes and other medical surgical
supplies:
a. nursing department c. medical social department
b. central service department d. laboratory service department

C 210. Which of the following sulfonylureas has the longest duration of action?
a. acetazolamide d. tolazamide
b. glyburide e. acetohexamide
c. chlorpropamide

D 211. Insulin preparation is usually administered by:


a. intradermal injection c. intravenous bolus
b. intramuscular injection d. subcutaneous injection
Hospital Pharmacy Page 16 of 22

E 212. Function/s of purchasing and inventory control:


a. maintain drug inventory control
b. purchase all drugs
c. receive, store and distribute drugs
d. coordinate and control all drug delivery and distribution system
e. all of the above

A 213. Levels of patient care requiring hospital setting:


a. intermediate and intensive care c. self care and ambulatory care
b. self care and home care d. b and c only

D 214. To achieve the goals of pharmacy service department, the hospital pharmacist should:
a. perform technical task
b. practice in patient care areas
c. perform task that require professional judgment
d. b and c
e. all of the above

B 215. The following are the responsibilities of pharmacy and therapeutics committee, except:
a. develop formulary
b. update the policies of the pharmacy department
c. choice of drugs to be requisitioned
d. add and delete from the list of drug
e. none of the above

B 216. The most common side effect of erythromycin therapy is:


a. hysterical laughing c. weight gain
b. abdominal cramping d. drowsiness

C 217. These are H2 blockers except:


a. cimetidine c. felodipine
b. nizatidine d. ranitidine

C 218. Prepare the hospital pharmacy newsletter:


a. administrative services c. pharmacy information service
b. education and training service d. pharmaceutical research division

C 219. The final decision as to which drugs shall be placed on the pavilion in the selection of charge floor stock drugs rests
on:
a. chief pharmacist d. medical staff
b. pharmacy and therapeutic committee e. both a and d
c. ?

A 220. Advantage/s of the floor stock system of drug distribution:


a. minimized return of medication
b. increased nursing time in medication activities
c. decreased potential for drug misadventuring
d. all of the above
e. none of the above

B 221. Chlorazepate dipotassium:


a. antiepileptic c. major tranquilizer
b. minor tranquilizer d. anesthetic

C 222. These are primary centers for clinical investigation on new drugs:
a. school d. a and b
b. manufacturing firm e. all of the above
c. hospital

A 223. The following are services that involve primary the professional care of the patient except:
a. admitting section d. medical record service
b. nutrition service e. anesthesia service
c. pathology service

C 224. First hospital pharmacist in the United States:


a. Charles Rice c. Jonathan Roberts
b. Jonathan Wilbert d. none of the above
Hospital Pharmacy Page 17 of 22

B 225. The following brought about the increasing need of hospital formularies, except:
a. increasing number of new drugs being marketed
b. unbiased advertising and unscientific drug literature
c. highly competitive marketing practices of the pharmaceutical industry
d. all of the above
e. none of the above

B 226. Drug distribution system used more frequently in small private hospitals:
a. floor stock d. all of the above
b. individual prescription number e. none of the above
c. unit-dose

B 227. Pharmacy technician performs the following responsibilities, except:


a. receive deliveries of medicine
b. compounds prescription and fill requisition
c. responsible for cleanliness of the pharmacy
d. do the lifting of heavy drugs

C 228. This provides the patient the best possible therapy and also provides guidelines to physicians and veterinarians:
a. pharmacy and therapeutic committee
b. medical dictionary
c. hospital formulary
d. a and c
e. none of the above

C 229. Duphaston is a/an:


a. cathartic c. infertility drug
b. laxative d. anti-TB drug

C 230. A generic substitute means:


a. therapeutic equivalent d. all of the above
b. bio-equivalent e. both a and c
c. chemical equivalent

C 231. The hospital department which is responsible for the safe and proper distribution of drugs to all patients:
a. purchasing c. pharmacy
b. medicine d. nursing

C 232. Disadvantage/s of the individual prescription order system of drug distribution:


a. increased potential for medication errors
b. increased drug inventory
c. delay in drug administration
d. all of the above
e. none of the above

D 233. All are policies of a hospital pharmacy, except:


a. prescription written by the physician who is not a member of the hospital staff should not be dispensed by
hospital pharmacy
b. only those orders and prescription within the hospital should be dispensed by hospital pharmacy
c. all employees of the hospital should be given a 36% discount
d. there should be no ongoing formal program of quality pharmaceutical service

A 234. Which of the following drugs is administered solely by inhalation?


a. beclomethasone dipropionate c. dexamethasone
b. epinephrine HCl d. none of the above

A 235. Commercially available amino acids include:


a. aminosyn c. liposyn
b. intralipid d. all of the above

D 236. A modern pharmaceutical library shall have the following reference material, except:
a. United States Pharmacopeia
b. National Formulary
c. Pharmacology, Toxicology and Therapeutics
d. Almanac
e. Bacteriology

C 237. These are personnel utilized whenever possible but never in activities requiring knowledge of the pharmacist:
a. nurse c. pharmacy helper
b. dietician d. all of the above
Hospital Pharmacy Page 18 of 22

A 238. One who prepares reports on the progress of the pharmacy department and submits them to the director of the hospital:
a. chief pharmacist d. all of the above
b. assistant chief pharmacist e. none of the above
c. staff pharmacist

E 239. IV medications may be administered as:


a. infusion d. saddleback
b. bolus e. a, b and c
c. piggyback

A 240. Each entry to the drug products listing of the hospital formulary must include, except:
a. cost information d. all of the above
b. size/s stocked by the pharmacy e. none of the above
c. common synonyms

A 241. This section is the heart of the formulary and consist of one or more descriptive entries for each formulary item plus
one or more indexes to facilitate use of formulary:
a. drug product listing c. inventory control
b. formulary system d. drug management and supply

C 242. Physiological antidote for narcotics:


a. atropine c. naloxone
b. ephedrine d. codeine

D 243. Allotment of space in the pharmacy should be based on:


a. establishment of formulary system to reduce the amount of storage space capacity
b. based on the ability of the director to haggle for more available space
c. basic functions to be carried on in the pharmacy
d. a and c only

D 244. Medications histories are taken by pharmacist of every patient admitted to the hospital or seen on the ambulatory care
section and which are taken:
a. to detect drug-induced laboratory test abnormalities
b. to help improve drug prescribing practices
c. to help prevent potential drug toxicities
d. all of the above
e. none of the above

D 245. A hospital pharmacist must develop physical assessment skills to:


a. be able to decipher and interpret common findings in the physical examination
b. have at least a basic understanding of such skills
c. have a background for more direct patient-care responsibilities
d. all of these

C 246. The most relaxed and comfortable position for examination of a patient with abdominal pain is:
a. Sim’s position c. dorsal position
b. lithotomy position d. supine position

B 247. This enable the pharmacist to screen for drug interactions and incompatibilities, drug allergies of drug prescribed:
a. nurses notes d. patient history
b. patient drug profile e. all of the above
c. pharmacy record book

E 248. In-patient pharmacist’s responsibilities:


a. keeps the central dispensing area neat and orderly
b. provide drug information as necessary to the pharmacy, medical and nursing staff
c. insure that good techniques are used ion compounding IV mixture
d. a and b
e. b and c

A 249. Another name of cost-plus rate method of pricing of goods in the hospital pharmacy:
a. Zugish system c. a and b
b. Storage system d. none of the above

D 250. Automated drug delivery system:


a. carboys c. pneumatic tubes
b. dumbwaiters d. all of the above

A 251. The following can legally receive drug samples, except:


a. nurses c. dentists
b. pharmacists d. veterinarians
Hospital Pharmacy Page 19 of 22

B 252. Chest percussion is done to:


a. identify the tactile fremitus
b. determine the amount of liquid in the lungs
c. assess the size of the lungs
d. all of these

A 253. TPN therapy is most frequently complicated by septicemia due to:


a. Candida c. Pseudopmonas
b. Enterobacter d. E. coli

D 254. The use of an investigational new drug within a particular institution requires that the:
a. institution be certified by BFAD
b. licensed prescriber be approved by the chief investigator like pharmacy and therapeutics committee
c. hospital administrator, licensed prescriber and pharmacy be certified by BFAD
d. institution and the physician be certified by the BFAD

B 255. The otoscope is used to:


a. measure the angle of bone and muscle movement
b. inspect the internal ear and tympanic membrane
c. assess the cardiovascular function
d. elicit reflexes

C 256. Non-licensed personnel play an important role in the hospital pharmacy because:
a. many of them aspire to be pharmacists and working in the hospital pharmacy provides training
b. they can be trained to act in the place of a pharmacist within 3 months
c. they can perform certain functions under the supervisions of a registered pharmacist and therefore free
other pharmacists to perform more professional duties

C 257. Administrative policies fir a pharmacy unit are generally developed:


a. with the approval of the pharmacy and therapeutics committee
b. with the approval of the medical and/or nursing staffs]
c. with the approval of the administrative officers of the hospital;
d. by the pharmacy unit alone

C 258. The pharmacy and therapeutics committee is not designed to:


a. serve an educational function
b. advise on professional policies involving the use of drugs in the hospital
c. dispense drugs for hospital use
d. recommend in-service programs relating to drugs

B 259. Drug samples are used for the following purposes, except:
a. to encourage clinical use by the physician
b. to be sold to indigent patients
c. for wounded civilians
d. for medical outreach purposes

A 260. Which one of the following does not have to be in the form of a written policy and procedure in the pharmacy?
a. patient admission c. orienting new pharmacy employee
b. handling flammable materials d. handling of narcotics

D 261. The following information can be deduced from the assessments of the skin, except:
a. scabs and scars – old and recent wounds
b. reddish skin – recent exposure under the sun
c. yellowish stains ion the second and third fingers – chronic smoker
d. produce sweating – dehydration

D 262. The following are abnormal heart sounds, except:


a. S3 and S4 c. murmurs and hums
b. rubs and gallops d. S1 and S2

A 263. Hair is important to assess since:


a. it is useful indicator of a patient’s health/emotional status
b. hair coloring gives the approximate age of the patient
c. length of hair implies the presence or absence of lice
d. amount of hair implies proper grooming

B 264. The clinical functions of a hospital pharmacist may include all of the following, except:
a. monitoring drug interactions
b. administration of medications
c. pharmacokinetic consultation
d. participation on a nutritional support team
e. making rounds with physicians
Hospital Pharmacy Page 20 of 22

D 265. The clinical functions of a hospital pharmacist may include all of the following, except:
a. Monitoring drug interactions
b. Participation on a nutritional support team
c. Making rounds with physicians
d. Administration of medications

D 266. These are developing trends in pharmacists functions, except:


a. use of generics
b. advice physician on dosage regimen
c. influence physician in choice of drugs
d. diagnosis

B 267. The focal point about which all activities of the hospital revolves:
a. physician c. governing board
b. patient d. administrator

C 268. All of the following are advantages of unit dose drug distribution system, except:
a. pharmacist review of medication errors
b. participation on a nutritional; support team
c. prompt delivery of new medication orders
d. reduced pharmacy costs

D 269. Information sources utilized by drug information centers may include all of the following, except:
a. medical journals c. on-line computer information sources
b. textbooks d. popular literature

B 270. All of the following are competencies required in institutional pharmacy practice, except:
a. assimilation and provision of comprehensive information on drugs and their actions
b. effective administration and management of hospital
c. development and conduct of product formulation and packaging program
d. conduct of and participation in research

A 271. Which of the following doses not correctly describes formulary system?
a. detrimental of the free enterprise system c. control dug cost
b. appraise drug therapy d. assure quality of drug

B 272. Pharmacist’s responsibility:


a. taking blood pressure c. laboratory testing
b. monitoring drug usage d. diagnosis

B 273. Total parenteral nutrition contains the following compounds:


a. carbohydrates, lipids and antibiotics
b. lipids, vitamins, electrolytes and amino acids
c. water and carbohydrates only
d. none of the above
e. all of the above

B 274. A unit dose package can be defined as a:


a. vial of 1gm of Keflin to be reconstituted by the nurse sent to the nursing unit
b. package containing the exact dose of drugs to be administered to a patient at a specific time
c. bottle containing enough medication for 24 hours
d. package containing enough medication for the nest three doses

D 275. Responsibilities often delegated to hospital pharmacy technicians include:


a. counseling of patients being discharged
b. review of physician’s medication orders
c. pharmacokinetic monitoring
d. affix pre-printed labels to containers of pre-packaged drugs

A 276. When a supervising pharmacist serves on the pharmacy and therapeutics committee, which of the following types of
detailed information should be supplied by him for consideration and evaluation?
a. information pertaining to the therapeutic, pharmaceutic and economic aspects of a drug
b. information pertaining to the economic aspects of a drug only
c. information pertaining to the therapeutic and economic aspects of a drug

D 277. The risk of medication misuse or errors are often due to:
a. increase frequency of change in mediation use
b. increase number of medications for patient
c. increased potency of modern medication
d. all of the above
e. none of the above
Hospital Pharmacy Page 21 of 22

C 278. The major reason for drug use review:


a. drug cost d. drug administration
b. patient drug history e. all of the above
d. quality of drug therapy

E 279. Life support techniques used to treat critically ill patient, except:
a. pulmonary wedge pressures d. hemodialysis
b. intra-aortic balloons e. none of the above
d. extracorporeal membrane oxygenation

B 280. A trade name for chlorpropamide is:


a. tolinase d. micronase
b. diabinase e. orinase
c. dimelor

E 281. In disease ,management, pharmacists are responsible for:


a. drug therapy d. both a and c
b. improving patient outcomes e. all of the above
c. decreasing the cost of drug therapy

D 282. The following statement is true about Drug Utilization Review (DUR), except:
a. DUR is a system for assessing drug distribution and medication system
b. DUR is a system for monitoring adverse drug reaction
c. DUR is expected to predict potential drug interaction
d. DUR recommends the use of brand names in prescribing medicines
e. All of the above are correct

C 283. The following are focus of DU R, except:


a. drugs d. health providers
b. patients e. diagnosis
c. research

E 284. It is considered as the backbone of any hospital staff:


a. medical staff d. clinical department
b. administration e. all of the above
c. support services

D 285. Treatment of hypoglycemia includes:


a. insulin d. both b and c
b. candy e. all of the choices
c. fruit juice

C 286. What extra label should be included on the Mycostatin prescription?


a. shake well d. apply early in the morning
b. avoid alcoholic beverages e. dissolve ion water
c. do not take by mouth

A 287. Characteristic/s of a TPN preparation:


a. complete source of nutrition d. contains great amount lipids
b. hypo-osmolar e. all of the above
c. can be infused together with blood products

D 288. The purpose of medical records:


a. serve as basis for planning and continuity of patient care
b. to provide date for use in research education
c. to serve as a basis for interview and evaluation of the care
d. all of the above
e. none of the above

E 289. All of these are possible indication s for the use of ibuprofen, except:
a. dental pain d. bursitis
b. dysmenorrhea e. acne
c. rheumatoid arthritis

A 290. A patient complains that his urine has become bright orange-red. Which of his current prescription medications
may be responsible?
a. rifampicin d. INH
b. acyclovir e. ethambutol
c. erythromycin

D 291. The smallest diameter of needle cannula is gauge:


a. 18 c. 25
b. 20 d. 27
Hospital Pharmacy Page 22 of 22

B 292. Which of the following is not a minimum standard for pharmacies in hospitals?
a. facilities d. research
b. drug administration e. drug distribution and control;
c. assuring rational drug therapy

C 293. Which of the following iron preparations is not available in an oral dosage form?
a. ferrous fumarate c. iron dextran
b. ferrous sulfate d. ferrous gluconate

D 294. Which of the following may increase the absorption of iron?


a. antacid d. ascorbic acid
b. folic acid e. milk
c. high protein meal

B 295. Another name for regular insulin is:


a. isophane insulin d. NPH
b. crystalline zinc insulin e. Semilente
c. protamine zinc insulin

C 296. Which vitamins and minerals should be supplemented routinely in the pregnant woman?
a. iron and pyridoxine d. calcium and pyridoxine
b. iron and thiamine e. calcium and vitamin A
c. iron and folate

E 297. Total parenteral nutrition is administered through:


a. jugular artery d. carotid artery
b. antecubital vein e. subclavian vein
c. femoral artery

A 298. Used to treat infections associated with COPD:


a. erythromycin 500 mg qid d. sulfa methoxazole 400mg/day
b. ampicillin 100mg bid e. both b and c
c. tetracycline 50 mg tid

D 299. The maximum dose of theophylline in children under 9m years of age is 24 mg/kg/day, while the maximum in
adults is:
a. 40 mg/kg/day d. 16 mg/kg/day
b. 36 mg/kg/day e. 10 mg/kg/day
c. 100 mg/kg/day

A 300. The pharmacist has been asked to educate the patient about signs of digoxin toxicity that require immediate
notification of the physician. Which of the following is not a sign of toxicity?
a. dyspnea d. drowsiness
b. bradycardia e. confusion
c. visual disturbance
HOSPITAL PHARMACY

A 1. A book ready for inspection by the BFAD anytime during business hours and should be kept for two years:
a. poison book d. prescription book
b. salesbook e. purchase book
c. additional opium book

C 2. A type of drug distribution system wherein all drugs are retained in the main pharmacy until receipt of the
physician's initial order:
a. floor stock system c. individual prescription system
b. decentralization unit dose system d. both a & c

B 3. Which of the following is not an antiasthma preparation?


a. terbutaline d. salbutamol
b. nifedipine e. theophylline
c. prednisolone

D 4. Oral hypoglycemics are usually most useful in:


a. Type I diabetes d. Type II diabetes
b. ascariasis e. UTI
c. asthma

B 5. It enables the pharmacist to monitor drug utilization of each patient confined in the hospital:
a. admission history d. adverse drug reaction report
b. patient medication profile e. progress note
c. patient chart

C 6. FeSO4 is used for


a. cough d. parasitism
b. pain e. arthritis
c. anemia

C 7. A laminar flow hood is used when intravenous solutions are:


a. labeled d. administered
b. stored e. both a & c
c. prepared

E 8. San Lazaro Hospital specializes in:


a. mental health d. bone disorder
b. childbirth e. contagious diseases
c. pulmonary diseases

A 9. The purpose of the Pharmacy and Therapeutic Committee are the ff:
a. advisory & educational d. dispensing & inventory
b. advisory & planning e. both a & d
c. educational & planning

B 10. Hospital department that supplies sterile linen, operating room packs and other medical surgical supplies:
a. nursing department d. laboratory service dept.
b. central service dept. e. pharmacy dept.
c. medical social dept.

D 11. Ranitidine is used for:


a. high blood pressure d. peptic ulcer
b. asthma e. diabetes
c. allergy

D 12. Total parenteral nutrition should be protected from light because it contains:
a. fats d. vitamins
b. carbohydrates e. proteins
c. electrolytes

B 13. The first recognized representative of pharmaceutical profession:


a. community pharmacist d. industrial pharmacist
b. hospital pharmacist e. clinical pharmacist
c. manufacturing pharmacist

A 14. A person who is engaged in managing, directing, and performing both professional and administrative services
associated with the operation of the hospital pharmacy department:
a. chief pharmacist d. pharmacy and therapeutics committee
b. staff pharmacist e. medical director
c. pharmacy supervisor

www.brex.us 91
HOSPITAL PHARMACY

C 15. Serve as a tool with which the pharmacist assures appropriateness, safety, and effectiveness of drug therapy:
a. medical staff
b. pharmacy & therapeutics committee
c. patient medication profile
d. physicians original order

A 16. The following person performs some of the pharmacist's technical functions except:
a. governing body d. secretary
b. clerk e. pharmacy aide
c. intern

E 17. These are administrative and management responsibilities of pharmacist, except:


a. preparation of reports
b. inventory control
c. plan and integrate professional services
d. budget

A 18. Men are not accepted in this department for treatment:


a. gynecology d. nephrology
b. urology e. cardiology
c. internal medicine

D 19. PSHP was established in what year?


a. 1930 d. 1962
b. 1942 e. 1970
c. 1950

A 20. Which is not a primary function of hospital?


a. preventive medicine c. providing health care
b. giving health information d. conduct scientific research

C 21. All of the following are minimum standards for hospital pharmacy, except:
a. facilities c. drug administration
b. research d. assuring rational drug therapy

C 22. The pharmacy department is not responsible for:


a. inspecting unused/expired drugs
b. preparing parenteral nutrition
c. distributing sterile linen
d. informing the hospital staff pertaining to medication

D 23. Primary source of income for private hospitals:


a. gift shop c. pharmacy dept.
b. canteen d. patient's bill

C 24. Which is not a clinical pharmacy service?


a. monitor patient response
b. conduct drug utilization review
c. prepare patient bed
d. obtain & document medication histories

B 25. Incumbent president of PSHP:


a. Lourdes Echauz d. Gloria Arroyo
b. Normita Leyesa e. Siopin Co
c. Eladio Tinio

C 26. Responsible for creating a hospital formulary:


a. pharmacy d. medical staff
b. medical director e. both b & c
c. board of trustees

A 27. Through a _______, the policies of the hospitals are implemented on a day-to-day basics:
a. chief executive officer d. doctors
b. corporation e. pharmacist
c. board of trustees

E 28. The act of a pharmacist in supplying one or more drug products to, or for patient, usually in response to an order
from a doctor”
a. administering d. giving
b. corporation e. dispensing
c. procuring

www.brex.us 92
HOSPITAL PHARMACY

B 29. It is a list of medications which are available only in that hospital:


a. MIMS d. RPS
b. Formulary e. both a and b
c. PDR

E 30. Most hospitals provide the following modalities:


a. surgery d. drug therapy
b. hypnosis e. a & d
c. acupuncture

C 31. The pharmacy department should have floor space for the ff., except:
a. office d. compounding & dispensing area
b. store room e. circulation
c. rest room

D 32. Emergency box in the hospital contains the following drugs, except:
a. Phenobarbital d. guaifenesin
b. epinephrine e. morphine
c. heparin

B 33. Pharmacy department is best classified under what division of hospital:


a. general c. administrative
b. clinical d. medical

B 34. During chickenpox, measles or smallpox outbreak, this department would be very busy:
a. dermatology c. pediatric
b. infectious disease d. neurology

A 35. This should be performed if the patient died while in the hospital:
a. autopsy c. biopsy
b. surgery d. physical examination

C 36. All of these should be taken into consideration in preparing dosage form not available commercially, except:
a. stability d. pharmacokinetic
b. quality assurance e. economic factor
c. availability of the patient

B 37. The following are general sections of a pharmacy procedural manual, except:
a. facilities d. services & activities
b. library e. personnel
c. organization

B 38. Who is not qualified to be a member of the Pharmacy and Therapeutics Committee?
a. cardiologist d. hospital administrator
b. pharmacy intern e. nurse
c. gastroenterologist

E 39. Drugs approved by the Pharmacy & Therapeutics Committee may be classified into the following categories,
except:
a. specialized formulary
b. investigational drug
c. drug approved on a conditional trial period
d. formulary drug
e. nonformulary drug

A 40. First President of PSHP:


a. Rosario Capistrano Tan d. Marcelia Itturalde
b. Lourdes Echauz e. Siopin Co
c. Violeta Alvarez

D 41. Nutritional solution prepared by hospital pharmacist for patients who has dysfunctional GIT:
a. IV mixture d. TPN
b. D5W e. NTP
c. NSS

C 42. Medical staff who are responsible for taking care of patient and who has direct involvement with the staff
organizational and administrative duties:
a. resident medical staff d. consulting medical staff
b. honorary medical staff e. honorary medical staff
c. attending medical staff

www.brex.us 93
HOSPITAL PHARMACY

E 43. Responsibilities of Pharmacy & Therapeutics Committee, except:


a. develop a mechanism for reporting & reviewing ADR
b. prepare policies governing the activities of medical representative
c. develop a book containing the list of medications available in the hospital
d. prepare list of emergency drugs
e. interview pharmacy staff

C 44. Information which is not necessary in an inpatient prescription:


a. date when the order was written
b. name and strength of medication
c. patient’s addressograph information
d. printed name of physician

B 45. Which will not help in ensuring the safety of a pharmacy department in the hospital
a. segregation of poisonous materials from non-poisonous materials
b. employment of a sufficient number of nonqualified personnel
c. policy with regards to the use of investigational drugs
d. availability of adequate and safe working space

C 46. Which may not lead to an adverse drug reaction?


a. error in compounding
b. use of the wrong drug
c. rashes due to sudden change in temperature
d. overdose
e. underdose

D 47. Pharmacy division responsible for improving formulations of existing products:


a. assay and quality control division
b. manufacturing and packaging division
c. department services division
d. pharmaceutical research division
e. administrative services division

A 48. Responsible for preparing the pharmacy newsletter:


a. drug information services division
b. education and training division
c. administrative services division
d. pharmaceutical research division
e. departmental services division

A 49. This is not a necessary information in drug evaluation for inclusion in the hospital formulary:
a. available sizes of drugs d. therapeutic indication
b. adverse reactions e. bioavailability
c. source of supply

C 50. Takes charge of interviewing medical sales representatives:


a. educational and training division
b. in-patient services division
c. purchasing and inventory control division
d. administrative services division
e. departmental services division

A 51. Drug recalls may emanate from the following agencies, except:
a. nursing department d. manufacturer
b. BFAD e. pharmacy department
c. Pharmacy & Therapeutic Committee

D 52. Nursing stations are inspected or visited by the pharmacist due to the ff. reasons, except:
a. to check labels for legibility c. to remove deteriorated drugs
b. to remove expired products d. to remove empty containers

A 53. Patient drug profile is maintained in the pharmacy department for the ff. purposes, except
a. to lower down the patient's expenses
b. to prevent potential drug interactions
c. to promote rational use of drugs
d. to detect drug-induced laboratory test abnormalities
e. to prevent drug allergies

B 54. Which one is not useful to the pharmacist in evaluating the quality of care provided to patient with a certain
disease state?

www.brex.us 94
HOSPITAL PHARMACY

a. proper choice of dosage form d. proper route of administration


b. proper price of medication e. proper strength of medication
c. proper choice of therapeutic agent

B 55. The process used to determine the “waiting period” of an outpatient for his prescription?
a. drug recall c. flow process chart
b. queuing theory d. activity chart

C 56. Which question is not of importance in the pharmacist's review of the patient's drug therapy record?
a. Were tests for predicting drug allergies done on patients with suspicious history?
b. Was the drug employed one with a special effect upon the diagnosed ailment?
c. Was the drug taken with water?
d. Did the patient receive his therapy via oral route when the topical route should have been employed?

B 57. An enteric-coated tablet that disintegrates upon passing through the stomach falls under what type of a defective
product?
a. faulty drug delivering apparatus c. inadequate packaging
b. defective dosage form d. deteriorated product

E 58. All of the following are direct responsibilities of the chief pharmacist, except:
a. schedules work hours for pharmacy staff
b. establishes and/or implements written policies & procedures for pharmacy operation
c. evaluates job performance of pharmacy staff
d. selects, hires and institutes disciplinary action & discharges pharmacy staff
e. performs packaging and labeling functions to promote product stability

C 59. Filling of prescription is lawfully the job of:


a. physician d. sales clerk
b. nurse e. patient
c. pharmacist

C 60. The following are pharmacist's responsibilities, except:


a. review patient's drug administration form to insure that all doses are being administered and charted
correctly
b. assist the physician in selecting dosage regimen schedules
c. transcribe accurately new drug administration forms
d. provide drug information to physicians, nurses and other health care personnel

B 61. The pharmacy department does not supply one of the following:
a. IV admixture d. light
b. sterile linen e. biological
c. narcotics

A 62. What is the most important storage condition for narcotic drugs?
a. security d. light
b. sanitation e. moisture
c. temperature

A 63. Which is not an acceptable method of drug information dissemination?


a. megaphone d. leaflets
b. pharmacy bulletin e. seminar
c. newsletter

C 64. This pharmacy division coordinates and control all drug delivery and distribution systems:
a. departmental services division c. purchasing & inventory control division
b. administrative services division d. central supply services division

D 65. Not a suitable location for the hospital pharmacy department:


a. first floor c. near the emergency room
b. near the information area d. basement

A 66. Control substance which is non-refillable:


a. schedule II c. schedule IV
b. schedule III d. schedule V

E 67. Which duty should not be assigned to a pharmacy technician?


a. maintain drug inventory records
b. clean prescription equipment
c. print labels for prepackaged product
d. weigh ingredients in bulk compounding
e. compound prescriptions

www.brex.us 95
HOSPITAL PHARMACY

E 68. A very important liaison between the hospital, patient and his community
a. administrator d. pharmacist
b. P & TC e. medical social service
c. priest

C 69. The service generally under the supervision of a licensed physician who has a basic interest in hematology:
a. pathology d. dietetic service
b. anesthesia e. morgue
c. blood bank
A 70. Psychiatric, AIDS and leprosy-victims must be attended by:
a. custodial hospital d. short-term hospital
b. general hospital e. long-term hospital
c. non-accredited hospital

B 71. Which of the following is not needed in the admission sheet?


a. patient's name and address d. occupation
b. monthly income e. family status
c. sex & date of birth

A 72. Which is not included in the laboratory report of a patient?


a. manufacturing data d. hematology data
b. microbiology data e. chemistry
c. pathology data

E 73. The hospital which is affiliated with a university:


a. Philippine Heart Center d. National Kidney Institute
b. Quezon Institute e. Philippine General Hospital
c. San Lazaro Hospital

C 74. This is one of the items in the medical record for the purpose of providing the physician with a chronologic picture
and analysis of the clinical course of the patient:
a. special examination d. pathologic findings
b. history of present illness e. physical examination
c. progress notes

E 75. This department enables the physician to assume the care of the patient at any time:
a. pharmacy d. nursing
b. medical social service e. medical records
c. central sterile supply

A 76. This type of hospital provides care to patients with various types of illnesses:
a. general hospital d. not-for-profit hospital
b. special hospital e. hospital with 50-99 beds
c. short-term hospital

D 77. This is not a chronic ailment:


a. asthma d. diarrhea
b. cancer e. diabetes
c. epilepsy

B 78. This is not a function of the dietetic service:


a. participate in ward rounds and conferences
b. recording medication histories of patients
c. plan patient's menu
d. interview patients regarding their food habits
e. counseling patients concerning normal or modified diet regimen

B 79. One of the responsibilities of the pharmacy department is to control all drugs used in the hospital. Which of the ff.
does not fall in this responsibility?
a. drug samples must be distributed only through the pharmacy
b. chief pharmacist should have a private office
c. maintenance of an up-to-date formulary
d. routine inspection of drug storage areas

A 80. Hospital maybe classified in different ways, except:


a. location d. type of service
b. ownership e. length of service
c. bed capacity

D 81. This group consists of medical practitioners of recognized professional ability:

www.brex.us 96
HOSPITAL PHARMACY

a. active d. consulting
b. associate e. honorary
c. courtesy

B 82. Serves as the secretary of the P & TC:


a. medical director c. nursing director
b. pharmacy director d. administrator

C 83. It serves as a guide to the hospital pharmacist in the development and execution of effective and proficient
pharmaceutical services in the hospital:
a. formulary system d. patient's medication profile
b. P & TC e. blue book
c. pharmacy procedural manual

B 84. The type of ambulatory care sought for by most outpatients:


a. tertiary care c. emergency case
b. primary care d. secondary care

B 85. Contains one discreet pharmaceutical dosage form:


a. unit dose d. multiple dose package
b. single unit package e. both b & c
c. unit dose package

B 86. A process of lifelong learning availed of by pharmacists for maintaining and improving professional competence:
a. undergrad studies c. graduate studies
b. continuing education d. intensive program

A 87. Represents the type of transaction that involves purchase of goods on specific volume and receiving certain
merchandise at no additional cost:
a. deals c. discount
b. volume contract d. group purchasing

C 88. A modern usage of the term ambulatory patient:


a. in-patient d. institutionalized patient
b. ward patient e. hospitalized patient
c. non-institutionalized patient

B 89. Medication administered to the wrong patient may be classified as:


a. wrong dose error c. wrong time error
b. unauthorized drug error d. omission error

A 90. Which of the following elements are common to ambulatory care but not to inpatient pharmaceutical service
program?
a. information on the proper use of the drug will be disseminated
b. medications dispensed will be completely labeled & packaged
c. the pharmacy program must be directed by a qualified pharmacist
d. drugs will be properly controlled

A 91. Which of the ff. does not correctly describe formulary system?
a. detrimental to the free enterprise system
b. appraises drug therapy
c. controls drug cost
d. assures quality of drug

B 92. A prescription label must contain all of the ff. except:


a. patient name c. patient age
b. patient address d. name of drug

D 93. This consists of routine systematic review of the body:


a. provisional diagnosis d. physical examination
b. special examination e. pathologic findings
c. personal family history

C 94. Medical staff is composed of:


a. pharmacists d. medical technologists
b. nurses e. both b & c
c. physicians

A 95. This ensures that drug orders are not inappropriately continued:

www.brex.us 97
HOSPITAL PHARMACY

a. automatic stop order d. professional care service


b. P & TC e. group purchasing
c. formulary system

A 96. The final decision as to which drugs should be included in the ward stock rests upon:
a. P & TC d. physician
b. pharmacist e. administrator
c. nurse

D 97. With respect to other drug distribution methods, the advantages of unit dose include the ff, except:
a. safer for the patient
b. more effective method of utilizing professional resources
c. less workload to the pharmacists
d. less manpower requirement
e. both a & b

B 98. The ff. are true, except:


a. two preparations each labeled with the same active chemical components are generic equivalents
b. Two preparations each labeled with different active ingredients are clinically effective equivalents
c. both high and poor quality pharmaceuticals are available under both generic names and brand names
d. a relationship exists between quality pharmaceuticals and therapeutic efficacy

D 99. Hospital Pharmacy


a. patient, physician, nurse, pharmacist relationship
b. practice of pharmacy in hospital setting
c. stepping stone to clinical pharmacy
d. all of the above

D 100. Purpose of P and TC, except:


a. develops a formulary of accepted drugs for used in the hospital
b. evaluates clinical data concerning drug for use in the a hospital
c. adds or deletes from the list of drugs accepted for used in the hospital
d. advises the doctor as to what brand of drug to use

D 101. Injections are often indicated as the correct form of administering a drug when
a. the drug is a suspension c. the drug is insoluble
b. the drug contains alcohol d. the drug is not easily absorbed from the GIT

D 102. Functions of the P and TC, except:


a. review adverse drugs reactions to drugs administered
b. to recommend addition and deletion of drugs
c. to serve as advisory to medical staff and pharmacists
d. approval of duplicate preparation to be listed in the formulary

A 103. Biological and other thermolabile medications should be kept in:


a. separate compartment in a refrigerator
b. locked cabinet
c. the display cabinet together with other drugs
d. none of the above

B 104. Basic function of hospital except:


a. education c. patient care
b. delivery of drugs to various outlets d. research

B 105. Medical staff primarily concerned with regular care of patient:


a. honorary medical staff d. residential medical staff
b. active medical staff e. consulting medical staff
c. associate medical staff

D 106. An authorized, structure and continuing program that reviews, analyzes and interprets pattern of drug usage in a
given health care delivery system against predetermined standards:
a. adverse drug reaction review c. P & TC
b. formulary system d. drug use review

C 107. Type of drug distribution system for government hospital:


a. individual prescription order system
b. unit dose
c. combination of individual prescription and floor stock
d. complete floor stock

D 108. Advantages of unit dose distribution:

www.brex.us 98
HOSPITAL PHARMACY

a. reduction in the incidents f medication error


b. decrease in the total cost of medication related activities
c. more accurate billings for drugs
d. all of the above

A 109. An extension of the main hospital pharmacy


a. satellite pharmacy c. both a & b
b. clinic d. none of the above

D 110. Institutional pharmacy is basically:


a. extended-care pharmacy c. manufacturing pharmacy
b. community pharmacy d. hospital pharmacy

D 111. Pyrogenic contamination may cause the patient to have


a. fever d. a & b
b. chills e. a & c
c. infection

C 112. Education and training division


a. coordinate programs of undergraduate pharmacy students
b. participate in hospital-wide educational programs involving doctors, nurses. etc.
c. both a and b
d. none of the above

D 113. Which is/are true about the DDB prescription form?


a. original copy-pharmacist copy c. triplicate copy – doctor copy
b. duplicate copy – patient’s copy d. all of the above

B 114. Any drug that has not yet been released for general use and has not been cleared for marketing by the BFAD.
a. prototype drug c. prodrug
b. investigational drug d. controlled drug

D 115. Responsible for specifications as to the quality, quantity, and source of drugs
a. purchasing agent c. medical staff
b. president d. pharmacist

A 116. A modern and reliable means of controlling the volume of purchases


a. computation o inventory turn-over c. all of the above
b. non-purchase d. none of the above

D 117. In inventory control, the turnover rate is calculated by which of the ff. formula?
a. annual purchases + annual inventory
b. annual purchases – annual inventory
c. annual purchases – annual inventory
d. annual purchases/annual inventory

C 118. The least acceptable method of transmitting prescription orders of physician:


a. by imputing the order into a computer terminal
b. by sending a carbon copy of the physician’s original medication order
c. transcribing by nursing personnel to a requisition form
d. the physician write the medication order on a separate blank and send it to the pharmacy

A 119. The leading and major organization of hospital pharmacists in the country:
a. PSHP c. ASHP
b. AMA d. PPHA

D 120. Injections are randomly tested for fever-producing agents which are called
a. analgesics c. histamines
b. antitussives d. pyrogens

D 121. Abilities required of hospital pharmacists, except:


a. through knowledge of drugs and their action
b. ability to develop and conduct a pharmaceutical manufacturing program
c. ability to conduct and participate in research
d. manage a hospital

A 122. Total parenteral nutrition is administered, through:


a. subclavian vein c. none of the above
b. femoral artery d. both a & bd.

www.brex.us 99
HOSPITAL PHARMACY

D 123. Which is not a part of patient’s medication history?


a. past and present medication
b. drug allergies
c. previous adverse effects associated with medication use
d. cost of medication

C 124. Fundamental functions of hospitals, except:


a. patient care d. research
b. education e. public health
c. housing

C 125. Medication orders (Chart Orders) differ from prescription in that they:
a. are intended for ambulatory use
b. contains only the generic name of the drug
c. may contain nonmedication instructions from the practitioner
d. contain the quantity of medication to be dispensed

C 126. A patient medication orders states that he is to receive ampicillin 500 mg.p.o.q8H. How many doses of ampicilln
250 capsules should be placed in the patient’s unit dose drawer daily?
a. 2 c. 6
b. 4 d. 10

B 127. Providing the correct medicine at the correct time in the correct dose and correct route of administration:
a. patient care c. research
b. rational drug therapy d. drug information

C 128. The primary goal of a hospital formulary is to:


a. strictly control prescribing in a hospital
b. save hospital money
c. insure that the patients receive the best drug therapy possible
d. influence medical staff to use approved drugs only

C 129. It shows the flow of authority and services on hand:


a. pharmacy procedural manual c. organizational chart
b. hospital pharmacy d. journal

A 130. Manner of affixing the label to an IV admixture preparation container:


a. upside down c. vertically
b. sideways d. normal way

C 131. The ideal drug distribution system


a. complete floor stock system c. unit dose system
b. individual prescription system d. none of the above

A 132. Additives of TPN are:


a. electrolytes and vitamins c. amino acids and anesthetics
b. antibiotics and fats d. none of the above

A 133. Continuous I.V. infusion means:


a. administration of a large volume of drug at a constant rate usually 100-150 mL/hr for 8-24 hrs.
b. rapid IV pushing less than 30 seconds
c. administration at a rate not to exceed 1 mL/min
d. none of the above

D 134. A separate sterile room is required for:


a. reconstitution of lyophilized injections
b. ophthalmic preparations
c. preparation of IV admixtures
d. all of the above

B 135. A sign to the pharmacist that the patient is overusing a certain product:
a. not purchasing c. healthy patient
b. frequency of refills d. discharge patient

C 136. Responsible for the procurement, distribution and control of all drugs used within the hospital:
a. purchasing agent c. pharmacist
b. medical staff d. lay personnel

D 137. Feature/s of the unit dose drug distribution system


a. maintains duplicate copy of the doctor’s order
b. maintains patient medication profile

www.brex.us 100
HOSPITAL PHARMACY

c. delivers medications in “unit of use” form


d. all of the above
e. a and c only

D 138. Pharmacist’s monitoring drug therapy would typically be checking all of the ff., except:
a. proper utilization of drug d. appropriate diagnosis
b. drug interactions e. none
c. allergy contraindications

A 139. True statements about D5W include all of the ff. except:
a. pH of 8 to 10
b. isotonic
c. often used in IV admixture
d. should be used with caution among diabetic patients
e. none
B 140. A method of administering IV fluids & medications that provides a slow, primary line infusion to maintain
therapeutic drug level or provide electrolyte replacement
a. IV push d. heparin lock
b. drip infusion e. none
c. piggyback

A 141. A health care supply described as a portable toilet.


a. commode d. rectal tube
b. truss e. none
c. bedpan

D 142. Which of the ff. is not a method of operating a unit dose dispensing program:
a. Centralized UDDS d. non-charge
b. decentralized UDDS e. none
c. combination of a and b

B 143. Attending staff is also known as:


a. associated medical staff d. honorary medical staff
b. reactive medical staff e. none
c. combination of a and b

B 144. A rubber, plastic, or glass tube inserted into the bladder in order to withdraw urine from patients unable to void
naturally.
a. rectal tube d. incontinence
b. catheter e. none
c. urinals

C 145. The injection of small amount of undiluted medication directly into the vein or through a heparin lock.
a. drip infusion d. IV piggyback
b. intermittent infusion e. none
c. IV bolus

B 146. A method of administering IV fluids and medication using a volume control device designed to administer small
amount of fluid over a specified time.
a. heparin lock d. IV push
b. additive set e. none
c. IV piggyback

C 147. Purpose of a pharmacy policy and procedure manual


a. ensure uniformity and consistency
b. the best defense in the event of litigation
c. both of the above
d. none of the above

C 148. Responsibilities often delegated to hospital pharmacy technicians include:


a. counseling patients d. pharmacokinetic monitoring
b. review physician’s order e. none
c. prepare IV admixture

C 149. All of the following are considered desirable functions of contemporary hospital pharmacy, except:
a. clinical consultation d. purchasing & inventory control
b. IV mixture programs
c. floor stock distribution

B 150. Pharmacist’s clinical function includes the ff. except:

www.brex.us 101
HOSPITAL PHARMACY

a. drug utilization review d. pharmaceutical care


b. drug dispensing e. drug informatratix
c. pharmacokinetic monitoring

B 151. Place or critically and seriously ill patients:


a. intermediates care unit d. all of the above
b. intensive care unit e. none of the above
c. long term care unit

A 152. IV administration of a large amount of fluid over a prolonged period of time:


a. infusion c. IV push
b. IV bolus d. piggyback

A 153. Under the administrative organization chart, the central sterile supply fall under the:
a. clinical services d. all of the above
b. ambulatory service e. none of the above
c. administrative services

A 154. It helps improve drug prescribing practice by promoting the safe and rational use of drug
a. drug utilization review d. all of the above
b. patient medication profile e. none of the above
c. adverse drug reaction

B 155. Area which offer the hospital great financial savings and pharmacist’s pride and prestige:
a. dispensing d. all of the above
b. extemporaneous compounding e. none of the above
c. drug consultant

B 156. Emergency drugs are considered as:


a. OR drug order d. automatic stop order
b. STAT order e. none of the above
c. follow up order

D 157. The risk of medication misuse or errors are often due to:
a. increase frequency of changes in medication use
b. increase number of medications for patient
c. increase potency of new drugs
d. all of the above
e. none of the above

D 158. Automatic stop-order policy for drugs:


a. when patient goes to the operating room
b. when the patient is transferred to another service
c. after 24 hours of administration
d. a and b only
e. all of the above

E 159. Best method of providing service when pharmacy if closed


a. provide an emergency cart for all unit
b. allow access of a nurse or doctor into the pharmacy
c. have a pharmacist on call
d. a & b only
e. a & c only

A 160. The responsible provision of drug therapy for the purpose of achieving definite outcomes that improve the quality
of life of a patient is:
a. pharmaceutical care d. all of the above
b. clinical pharmacy e. none of the above
c. traditional hospital pharmacy

C 161. Legal document which contains all the treatment done to the patient:
a. patient medical profile d. doctor’s sheet
b. therapeutic sheet e. all of the above
c. patient chart

E 162. In-charge of inspecting the contents of emergency box daily:


a. P & TC c. both a and c
b. pharmacist e. both b & c
c. nurse

D 163. Parts of formulary system, except:

www.brex.us 102
HOSPITAL PHARMACY

a. information of hospital policies and procedures concerning drugs


b. drug product listing
c. special information
d. special formularies

D 164. Three potential benefits of formulary systems, except:


a. economic c. education
b. therapeutic d. expensive

C 165. Advantages of unit dose drug distribution, except:


a. more efficient use of personnel c. increased floor stock
b. minimization of credits d. greater workload control

C 166. A potent analgesic emergency drug:


a. epinephrine c. pethidine
b. dopamine d. ephedrine

B 167. It encompasses the collection, organization, retrieval, interpretation, evaluation, and dissemination of information
pertaining to all aspects of medicine.
a. PTC c. LAFH
b. DIS d. EDL

B 168. The group established in the Phil. For hospital pharmacists


a. ASHP c. ASHP
b. PSHP d. PACOP

C 169. Which one is used as antiviral?


a. Terramycin c. Zovirax
b. Garamycin d. Vibramycin

B 170. Which of the following is not an antiasthma preparation?


a. terbutaline c. salbutamol
b. cimetidine d. prednisolone

E 171. Betadine is used as:


a. antifungal d. both a & b
b. antibacterial e. all of the above
c. antiinfective

D 172. These are acceptable colors of pharmacy wall, except:


a. white d. orange
b. cream e. baby pink
c. mint

B 173. The drug atropine sulfate is used as:


a. antiasthmatic c. anti anginal
b. anticholinergic d. anthelmintinc

C 174. Excessive rapidity of respiration:


a. Cytopenia c. tachypnea
b. Necrosis d. tachycardia

C 175. Surgical puncture of the heart


a. pericardium c. cardiocentesis
b. colostomy d. necroscopy

A 176. Functional disorder of the nervous system


a. neurosis c. meningitis
b. tachycardia d. phlebotomy

B 177. The drug used to paralyze the patient during surgical procedure:
a. penthotal sodium c. nubain
b. vecuronium d. Phenobarbital

E 178. These are examples of objective parameters, except:


a. blood level d. blood pressure
b. laboratory tests e. degree of pain

c. x-ray tests
B 179. NPO means:

www.brex.us 103
HOSPITAL PHARMACY

a. non-progressive operation d. nothing after six


b. nothing by mouth e. none of the above
c. with full stomach

D 180. The most accurate method in calculating children’s dose based on weight and height is:
a. Young’s rule c. Cowling’s rule
b. Fired’s rule d. BSA

C 181. The active ingredients of Desenex powder is:


a. tolnatrate c. undecylenic acid
b. salicylic acid

D 182. During eye examination, the following are noted except:


a. visual acuity c. EOM
b. papillary reflex d. presence of tears

A 183. The breath of an alcoholic patient may smell:


a. fruity c. musty
b. ammoniacal d. sweet

C 184. Cromolyn sodium is useful in asthma only in:


a. oral tabs c. inhalation
b. pills d. injectable

B 185. It is ideal and proper condition for aseptic handling of hyperalimentation and intravenous fluids:
a. HEPA filters c. TPN
b. LAFH d. ventilator

B 186. The normal drop rate in TPN therapy is:


a. 1 ml/min c. 1ml/min
b. 2 ml/min d. 5 ml/min

B 187. The starting dose of TPN per day is:


a. 6 L/day c. 5L/day
b. 2.5 L/day d. 2L/day

B 188. Patient who is not bedridden, who can take care of himself and administer his own medication:
a. patient care c. geriatric
b. ambulatory patient d. pediatric

D 189. Assessment of the musculoskeletal system includes:


a. assessment of posture and gait c. ROM testing
b. measurement of the extremities d. all of these

D 190. The skin is assessed for:


a. lesions c. mobility
b. hydration d. all of these

D 191. Inspection involves:


a. concentrated visualization of the patient
b. listening to sounds that may emanate from the patient
c. noting particular odors from the patient
d. all of these

B 192. Blood pressure is determined by:


1)palpation 2) percussion 3) auscultation 4) inspection
a. 1,2,3,4 c 2,4
b. 1,3 d. 1,2,3

D 193. During palpation, the ff. are used, except:


a. fingers c. back of the hands
b. palms d. fist

C 194. Borborygmi is:


a. discontinuous, bubbling abdominal sound of short duration
b. accurse abdominal sound that disappears on coughing
c. loud gurgling and tinkling sounds heard over the abdomen
d. also known as crepitation

C 195. It is a technique that delivers an amount of electrolyte solution over a period of several hours and is used to
replace electrolytes needed by the patient

www.brex.us 104
HOSPITAL PHARMACY

a. drop factor c. fluid blousing


b. metabolism d. loading dose

B 196. Drug which is an antineoplastic agent:


a. gentamycin c. furocemide
b. cisplexin d. plasil

A 197. Daily administration of aspirin can impair:


a. hearing c. both a and b
b. seeing

A 198. This service is responsible for the procurement, planning and preparation of food for the patient and hospital
staff:
a. dietary service c. record service
b. nursing service d. laboratory service

B 199. The purpose of laboratory examination and radiological services is for:


a. therapeutic c. treatment
b. diagnostic d. exercise

C 200. Powerful tool for the orderliness, cleanliness, and sanitary procedures in the hospital pharmacy department:
a. chief pharmacist d all of the above
b. assistant chief pharmacist e. none of the above
c. staff pharmacist

D 201. Objectives of drug information service:


a. answering inquiries d. a and b
b. publication e. all of the above
c. budget preparation and printing scheme

E 202. The formulary is very important so it should be:


a. complete d. a and b
b. concise e. all of the above
c. easy to use

A 203. A bulk supply of each drug product is stored on the nursing station
a. floor stock d. all of the above
b. individual prescription order e. none of the above
c. emergency cart

B 204. Which of the following solutions should be sterile when dispensed?


a. tinctures d. sprays
b. ophthalmic solution e. none of the above
c. spirit

E 205. Function of the Chief Pharmacist in Pharmacy and Therapeutic Committee


a. maintenance of an adequate up to date library and drug therapy references
b. interviewing and screening of professional medical representative
c. preparation and dissemination of accurate minutes of committee meeting
d. all of the above
e. a & c

C 206. One who shall initiate regulation pertaining to the professional policies of the Pharmacy Department:
a. pharmacy & therapeutic committee d. clinical pharmacist
b. therapeutic sheet e. all of the bill
c. patient chart

A 207. Relevant pharmaceutical services:


a. quality control, IV admixtures
b. drug information, education and training
c. extemporaneous compounding
d. drug dispensing
e. all of the above

A 208. Pharmacy records on all drugs dispensed to the patient are in:
a. nursing department c. patient bill
b. therapeutic sheet d. all of the bill
c. patient chart

B 209. A department that supplies sterile line, sterile kits, operating room packs, needles, syringes and other medical
surgical supplies:

www.brex.us 105
HOSPITAL PHARMACY

a. nursing department c. medical social department


b. central service department d. laboratory service dept.

C 210. Which of the ff. sulfonylureas has the longest duration of action?
a. acetazolamide d. tolazamide
b. glyburide e. acetohexamide
c. chlorpropamide

D 211. Insulin preparation is usually administered by:


a. intradermal injection c. intravenous bolus
b. intramuscular injection d. subcutaneous injection

E 212. Function/s of purchasing and inventory control:


a. maintain drug inventory control
b. purchase all drugs
c. receive, store and distribute drugs
d. coordinate and control all drug delivery and distribution system
e. all of the above

A 213. Levels of patient care requiring hospital setting


a. intermediate & intensive care c. self care & ambulatory care
b. self care and home care d. b & c only

D 214. To achieve the goals of pharmacy service department, the hospital pharmacist should:
a. perform technical task
b. practice in patient care areas
c. perform task that require professional judgment
d. b & c
e. all of the above

B 215. The following are the responsibilities of pharmacy and therapeutic committee, except:
a. develop formulary
b. update the policies of the pharmacy department
c. choice of drugs to be requisitioned
d. add and delete from the list of drug
e. none of the above

B 216. The most common side effect of erythromycin therapy is:


a. hysterical laughing c. weight gain
b. abdominal cramping d. drowsiness

C 217. These are H2 blockers, except:


a. cimetidine c. felodipine
b. nizatidine d. ranitidine

C 218. Prepare the hospital pharmacy newsletter:


a. administrative services c. pharmacy information service
b. education & training service d. Pharmaceutical research division

B 219. The final decision as to which drugs shall be placed on the pavilion in the selection of charge floor stock drugs
rests on:
a. chief pharmacist c. medical staff
b. pharmacy & therapeutic committee d. both a & d

A 220. Advantage/s of the floor stock system of drug distribution


a. minimized return of medication
b. increased nursing time in medication activities
c. decreased potential for drug misadventuring
d. all of the above
e. none of the above

B 221. Chlorazepate dipotassium


a. antiepileptic c. major tranquilizer
b. minor tranquilizer d. anesthetic

C 222. These are primary centers for clinical investigation on new drugs.
a. school d. a & b
b. manufacturing firm e. all of the above
c. hospital

A 223. The following are services that involve primary the professional care of the patient except:

www.brex.us 106
HOSPITAL PHARMACY

a. admitting section d. medical record service


b. nutrition service e. anesthesia service
c. pathology service

C 224. First hospital pharmacist in the United States


a. Charles Rice c. Jonathan Roberts
b. Jonathan Wilbert d. none of the above

B 225. The ff. brought about the increasing need of hospital formularies except:
a. increasing number of new drugs being marketed
b. unbiased advertising & unscientific drug literature
c. highly competitive marketing practices of the pharmaceutical industry
d. all of the above
e. none of the above

B 226. Drug distribution system used more frequently in small private hospital
a. floor stock d. all of the above
b. individual prescription number e. none of the above
c. unit dose

B 227. Pharmacy technician performs the following responsibilities except:


a. receives deliveries of medicine
b. compounds prescription and fill requisition
c. responsible for cleanliness of the pharmacy
d. do the lifting of heavy drugs

C 228. This provides the patient the best possible therapy and also provides guidelines to physicians and veterinarian
a. pharmacy and therapeutic committee
b. medical dictionary
c. hospital formulary
d. a & c
e. none of the above

C 229. Duphaston is a/an


a. cathartic c. infertility drug
b. laxative d. anti-TB drug

C 230. A generic substitute means:


a. therapeutic equivalent d. all of the above
b. bioequivalent e. both a & c
c. chemical equivalent

C 231. The hospital department which is responsible for the safe and proper distribution of drugs to all patients
a. purchasing c. pharmacy
b. medicine d. nursing

C 232. Disadvantage/s of the individual prescription order system of drug distribution:


a. increased potential for medication errors
b. increased drug inventory
c. delay in drug administration
d. all of the above
e. none of the above

D 233. All are policies of a hospital pharmacy, except:


a. prescription written by the physician who is not a member of the hospital staff should not be dispensed
by hospital pharmacy
b. only those orders & prescription within the hospital should be dispensed by hospital pharmacy
c. all employees of the hospital should be given a 36% discount
d. there should be no ongoing formal program of quality pharmaceutical service

A 234. Which of the following drugs is administered solely by inhalation?


a. beclomethasone dipropionate c. dexamethasone
b. epinephrine HCL d. none of the above

A 235. Commercially available amino acids include


a. Amynosyn c. Lyposin
b. Intralipid d. all of the above

D 236. A modern pharmaceutical library shall have the following reference material, except:
a. United States Pharmacopeia
b. National Formulary

www.brex.us 107
HOSPITAL PHARMACY

c. Pharmacology, toxicology & therapeutics


d. almanac
e. bacteriology

C 237. These are personnel utilized whenever possible but never in activities requiring knowledge of the pharmacist:
a. nurse c. pharmacy helper
b. dietician d. all of the above

A 238. One who prepares reports on the progress of the pharmacy department and submits them to the director of the
hospital:
a. chief pharmacist d. all of the above
b. asst, chief pharmacist e. none of the above
c. staff pharmacist

E 239. IV medications maybe administered as:


a. infusion d. saddleback
b. bolus e. a,b, & c
c. piggyback

E 240. Each entry to the drug products listing of the hospital formulary must include, except:
a. cost information d. all of the above
b. size/s stocked by the pharmacy e. none of the above

A 241. This section is the heart of the formulary and consists of one or more descriptive entries for each formulary item
plus one or more indexes facilitate use of formulary:
a. drug product listing c. inventory control
b. formulary system d. drug management & supply

C 242. Physiological antidote for narcotics.


a. atropine c. naloxone
b. ephedrine d. codeine

D 243. Allotment of space in the pharmacy should be based on:


a. establishment of formulary system to reduce the amount of storage space capacity
b. based on the ability of the director to haggle for more available space
c. basic functions to be carried on in the pharmacy
d. a & c only

D 244. Medication histories are taken by pharmacist of every patient admitted to the hospital or on ambulatory care
section and which are taken:
a. to detect drug-induced laboratory test abnormalities
b. to help improve drug prescribing practices
c. to help prevent potential drug toxicities
d. all of the above
e. none of the above

D 245. A hospital pharmacist must develop physical assessment skills to:


a. be able to decipher and interpret common findings in the physical examination
b. have at least a basic understanding of such skills
c. have a background for more direct patient-care responsibilities
d. all of these

C 246. The most relaxed and comfortable position for examination of a patient with abdominal pain is:
a. Sim’s position c. dorsal position
b. lithotomy position d. supine position

B 247. This enables the pharmacist to screen for drug reactions and incompatibilities, drug allergies of drug prescribed
a. nurses notes d. patient history
b. patient drug profile e. all of the above
c. pharmacy record book

E 248. In-patient pharmacist’s responsibilities:


a. keeps the central dispensing area neat and orderly
b. provides drug information as necessary to the pharmacy, medical and nursing staff
c. insure that good techniques are used in compounding IV mixture
d. a and b.
e. b and c

A 249. Another name of cost-plus rate method of pricing of goods in the hospital pharmacy.
a. Zugish system c. a and b
b. Storage system d. none of the above

www.brex.us 108
HOSPITAL PHARMACY

D 250. Automated drug delivery system:


a. carboys c. pneumatic tubes
b. dumbwaiters d. all of the above

A 251. The following can legally receive drug samples, except:


a. nurses c. dentists
b. pharmacists d. veterinarians

B 252. Chest percussion is done to:


a. identify the tactile fremitus
b. determine the amount of liquids in the lungs
c. assess the size of the lungs
d. all of these

A 253. TPN therapy is most frequently complicated by septicemia due to:


a. Candida c. Pseudomonas
b. Enterobacter d. E coli

D 254. The use of an investigational new drug within a particular institution requires that the
a. institution be certified by BFAD
b. licensed prescriber to be approved by the chief investigator like pharmacy and therapeutics committee
c. hospital administrator, licensed prescriber and pharmacy be certified by BFAD
d. institution and the physicians be certified by the BFAD

B 255. The otosccope is used to:


a. measure the angle of bone and muscle movement
b. inspect the internal ear and tympanic membrane
c. assess the cardiovascular function
d. elicit reflexes

C 256. Non-licensed personnel play an important role in the hospital pharmacy because
a. many of them aspire to be pharmacists, and working in the hospital pharmacy provides training
b. they can be trained to act in the place of a pharmacist within 3 months
c. they can perform certain functions under the supervision of a registered pharmacist and therefore free
other pharmacists to perform more professional duties

C 257. Administrative policies for a pharmacy unit are generally developed


a. with the approval of the pharmacy and therapeutics committee
b. with the approval of the medical and/or nursing staffs
c. with the approval of the administrative officers of the hospital
d. by the pharmacy unit alone

C 258. The pharmacy and therapeutics committee is not designed to:


a. serve an educational function
b. advise on professional policies involving the use of drugs in the hospital use
c. dispense drugs for hospital use
d. recommend in-service programs relating to drugs

B 259. Drug samples are used for the following purposes, except:
a. to encourage clinical use by the physician
b. to be sold to indigent patients
c. for wounded civilians
d. for medical outreach purposes

A 260. Which one of the following does not have to be in the form of a written policy and procedure in the pharmacy?
a. patient admission c. orienting new pharmacy employee
b. handling flammable materials d. handling of narcotics

D 261. The following are abnormal heart sounds, except


a. S3 and S4 c. murmurs and hums
b. rubs and gallops d. S1 and S2

D 262. The following information can be deducted from the assessments of the skin, except:
a. scabs and scars – old and recent wounds
b. reddish skin – recent exposure under the sun
c. yellowish stains on the second and third fingers-chronic smoker
d. produce sweating – dehydration

A 263. Hair is important to assess since


a. it is useful indicator of a patient’s health/emotional status
b. hair coloring gives the approximate age of the patient

www.brex.us 109
HOSPITAL PHARMACY

c. length of hair implies the presence or absence of lice


d. amount of hair implies proper grooming

B 264. The clinical functions of a hospital pharmacist may include all of the following, except:
a. monitoring drug interactions
b. administration of medications pharmacokinetic consultation
d. participation on a nutritional support team
e. making rounds with physicians

D 265. The clinical functions of a hospital pharmacist may include all of the following, except:
a. monitoring drug interactions
b. participation on a nutritional support team
c. making rounds with physicians
d. administration of medications

D 266. These are developing trends in pharmacists functions, except


a. use of generics
b. advise physician on dosage regimen
c. influence physician in choice of drugs
d. diagnosis

B 267. The focal point about which all activities of the hospital revolves:
a. physician c. governing board
b. patient d. administrator

C 268. All of the following are advantages of unit dose drug distribution system, except:
a. pharmacist review of medication errors
b. participation on a nutritional support team
c. prompt delivery of a new medication orders
d. reduced pharmacy costs

D 269. Information sources utilized by drug information centers may include all of the following, except:
a. medical journals c. on-line computer information sources
b. textbooks d. popular literature

B 270. All of the following are competencies required in institutional pharmacy practice, except:
a. assimilation and provision of comprehensive information on drugs and their actions
b. effective administration and management of hospital
c. development and conduct of product formulation and packaging program
d. conduct of and participation in research

A 271 Which of the following does not correctly describes formulary system?
a. detrimental of the free enterprise c. control drug cost system
b. appraise drug therapy d. assume quality of drug

B 272. Pharmacist’s responsibility


a. taking blood pressure c. laboratory testing
b. monitoring drug abuse d. diagnosis

B 273. Total parenteral nutrition contains the following compounds


a. carbohydrates, lipids and antibiotics
b. lipids, vitamins, electrolytes and amino acid
c. water and carbohydrates only
d. none of the above

B 274. A unit dose package can be defined as a


a. vial of 1gm of Keflin to be reconstituted by the nurse sent to the nursing unit
b. package containing the exact dose of drugs to be administered to a patient at a specific time
c. bottle containing enough medication for 24 hours
d. package containing enough medication for the next three doses

D 275. Responsibilities often delegated to hospital pharmacy technicians include


a. counseling of patients being discharged
b. review of physician’s medication orders
c. pharmacokinetic monitoring
d. affix pre-printed labels to containers of pre-packaged drugs

A 276. When a supervising pharmacist serves on the pharmacy and therapeutics committee, which of the ff. types of
detailed information should be supplied by him for consideration and evaluation?
a. information pertaining to the therapeutic, pharmaceutics and economic aspects of a drug
b. information pertaining to the economic aspects of a drug only

www.brex.us 110
HOSPITAL PHARMACY

c. information pertaining to the therapeutic and economic aspects of a drug

D 277. The risk of medication misuse or errors are often due to


a. increase frequency of change in medication use
b. increase number of medications for patient
c. increased potency of modern medication
d. all of the above

C 278. The major reason for drug use review


a. drug cost d. drug administration
b. patient drug history e. all of the above
c. quality of drug therapy

E 279. Life support techniques used to treat critically ill patient, except:
a. pulmonary wedge pressures d. hemodialysis
b. intraaortic ballons e. none of the above
c. extracorporeal membrane oxygenation

B 280. A trade name for chorpropamide is:


a. Tolinase d. Micronase
b. Diabenese e. Orinase
c. dimelor

E 281. In disease management, pharmacists are responsible for:


a. drug therapy d. both a & c
b. improving patient outcomes e. all of the above
c. decreasing the cost of drug therapy

D 282. The following statement is true about Drug Utilization Review


(DUR), except
a. DUR is a system for assessing drug distribution and medication system
b. DUR is a system for monitoring adverse drug reaction
c. DUR is expected to predict potential drug interaction
d. DUR recommends the use of brand names in prescribing medicines
e. all of the above are correct

E 283. The following are focus of DUR, except:


a. drugs d. health providers
b. patients e. diagnosis
c. research

E 284. It is considered as the backbone of any hospital staff


a. medical staff c. clinical department
b. administration d. all of the above

D 285. Treatment of hypoglycemia includes:


a. insulin d. both b & c
b. candy e. all of the choices
c. fruit juice

C 286. What extra label should be included on the Mycostatin prescription?


a. shake well d. apply early in the morning
b. avoid alcoholic beverages e. dissolve in water
c. do not take by mouth

A 287. Characteristic/s of a TPN preparation


a. complete source of nutrition d. contains great amount lipids
b. hypoosmolar e. all of the above
c. can be infused together with blood products

D 288. The purpose of medical records


a. serve as a basis for planning and continuity of patient care
b. to provide date for use in research education
c. to serve as a basis for interview and evaluation of the care
d. all of the above
e. none of the above

E 289. All of these are possible indications for the use of ibuprofen, except:
a. dental pain c. bursitis
b. dysmenorrheal d. acne

www.brex.us 111
HOSPITAL PHARMACY

A 290. A patient complains that his urine has become bright orange-red. Which of his current prescription medications
maybe responsible?
a. rifampicin d. INH
b. acyclovir e. Ethembutol
c. erythromycin

D 291. The smallest diameter of needle cannula is gauge:


a. 18 c. 25
b. 20 d. 27

B 292. Which of the following is not a minimum standard for pharmacies in hospitals:
a. facilities d. research
b. drug administration e. drug distribution & control
c. assuring rational drug therapy

D 293. Which of the following may increase the absorption of iron?


a. antacid d. ascorbic acid
b. folic acid e. milk
c. high protein meal

C 294. Which of the following iron preparations is not available in an oral dosage form?
a. ferrous fumarate c. iron dextran
b. ferrous sulfate d. ferrous gluconate

E 295. Another name for regular insulin is:


a. isophane insulin d. NPH
b. crystalline zinc insulin e. Semilente
c. protamine zinc insulin

C 296. Which vitamins and minerals should be supplemented routinely in the pregnant woman?
a. iron & pyridoxine d. calcium & pyridoxine
b. iron & thiamine e. calcium & vit. A
c. iron & folate

E 297. Total parenteral nutrition is administered through:


a. jugular artery d. carotid artery
b. antecubital vein e. subclavian vein
c. femoral artery

A 298. Used to treat infections associated with COPD


a. erythromycin 500 mg qid d. sulfamethoxasole 400 mg/day
b. ampicillin 100 mg bid e both b and

D 299. The maximum dose of theophyline in children under 9 years of age is 24 mg/kg/day, while the maximum in
adults is:
a. 40 mg/kg/day d. 16/mg/kg/day
b. 36 mg/kg/day e. 10 mg/kg/day
c. 100 mg/kg/day

B 300. The pharmacist has been asked to educate the patient about signs of Digoxin toxicity that require immediate
notification of the physician. Which require immediate notification of the physician? Which of the ff. is not a sign
of toxicity?
a. dyspnea d. drowsiness
b. bradycardia e. confusion
c. visual disturbance

www.brex.us 112

You might also like